Почему помутнел раствор баритовой воды: 1)Почему помутнел раствор баритовой воды? Напишите уравнение реакции. 2) почему белый порошок

Почему помутнел раствор баритовой воды: 1)Почему помутнел раствор баритовой воды? Напишите уравнение реакции. 2) почему белый порошок

Содержание

Практические работы по химии

Практическая работа № 1

«Обнаружение углерода и водорода в органическом соединении.

Цель:

    • научить определять углерод, водород в органических соединениях;

    • познакомить с качественными реакциями непредельных углеводородов.

Реактивы: парафин, оксид меди (II), сульфат меди (II), известковая или баритовая вода, медная проволока, хлороформ, гексан, гексен-1, 5%-й спиртовой раствор фенилацетилена, толуол, раствор перманганата калия, карбонат натрия, бромная вода, аммиачный раствор хлорида меди (I).

Посуда и оборудование: лабораторный штатив, штатив для пробирок, пробирки, пробка с газоотводной трубкой, спиртовка, спички.

Обнаружение углерода и водорода

Присутствие углерода в органических соединениях в большинстве случаев можно обнаружить по обугливанию вещества при осторожном его прокаливании.

Наиболее точным методом открытия углерода и одновременно с ним водорода является сожжение органического вещества в смеси с мелким порошком оксида меди. Углерод образует с кислородом оксида меди(П) углекислый газ, а водород — воду. Оксид меди восстанавливается до металлической меди, например:

С13Н28 + 40СuО — 13С02 + 14Н20 + 40Сu

Ход работы

Опыт № 1 Определение углерода и водорода в органическом соединении (парафин)

Соберите прибор, как показано на рисунке 44. Смесь 1—2 г оксида меди(II) и -0,2 г парафина хорошо перемешайте и поместите на дно пробирки. Сверху насыпьте еще немного оксида меди(II). В верхнюю часть пробирки введите в виде пробки небольшой кусочек ваты и насыпьте на нее тонкий слой белого порошка безводного сульфата меди(II). Закройте пробирку пробкой с газоотводной трубкой. При этом конец трубки должен почти упираться в комочек ваты с сульфатом меди(II). Нижний конец газоотводной трубки должен быть погружен в пробирку с баритовой водой (раствор гидроксида бария) или известковой водой (раствор гидроксида кальция). Нагрейте пробирку в пламени горелки. Если пробка плотно закрывает пробирку, то через несколько секунд из газоотводной трубки начнут выходить пузырьки газа. Как только баритовая вода помутнеет, пробирку с ней следует удалить и продолжать нагревание, пока пары воды не достигнут белого порошка сульфата меди(ІІ) и не вызовут его посинения.

После изменения окраски сульфата меди(ІІ) следует прекратить нагревание.

  1. Почему помутнел раствор баритовой воды? Напишите уравнение реакции.

  2. Почему белый порошок сульфата меди(ІІ) стал голубым? Напишите уравнение реакции.

Заполнить таблицу

№п/п

Исходные вещества

Наблюдение

Уравнение реакций

Вывод

Название

Формула

Выводы:

обнаружив образовавшиеся в результате реакции углекислый газ и воду, вы установили в исследованном веществе наличие углерода и водорода. Так как эти элементы не содержались в добавленном оксиде меди(ІІ), то они могли находиться только во взятом для анализа органическом веществе.

Практическая работа № 2

«Обнаружение галогенов (проба Бейльштейна)»

Цель:

    • научить определять хлор в органических соединениях;

    • познакомить с качественными реакциями непредельных углеводородов.

Реактивы:  медная проволока, хлороформ, оксид меди (II)

Посуда и оборудование: лабораторный штатив, штатив для пробирок, пробирки, пробка с газоотводной трубкой, спиртовка, спички.

Ход работы

Опыт № 1 Обнаружение галогенов (проба Бейльштейна)

Галогены можно обнаружить при помощи реакции окрашивания пламени, предложенную русским химиком Ф. Ф. Бейльштейном.

Для проведения опыта требуется медная проволока длиной около 10 см, загнутая на конце петлей и вставленная другим концом в небольшую пробку (рис. 45).

Держа за пробку, прокалите петлю проволоки до исчезновения посторонней окраски пламени. Остывшую петлю, покрывшуюся черным налетом оксида меди (ІІ), опустите в пробирку с хлороформом, затем смоченную веществом петлю вновь внесите в пламя горелки. Немедленно появляется характерная зеленовато-голубая окраска пламени, так как образующиеся при сгорании летучие галогениды меди окрашивают пламя горелки.

Выводы:

обнаружив образовавшиеся в результате реакции углекислый газ и воду, вы установили в исследованном веществе наличие углерода и водорода. Так как эти элементы не содержались в добавленном оксиде меди(ІІ), то они могли находиться только во взятом для анализа органическом веществе.

Практическая работа № 3

«Получение метана и изучение его свойств: горения, отношения к бромной воде и раствору перманганата калия»

Зафиксируйте в тетради каждый опыт по алгоритму:

  • записать название опыта;

  • открыть по ссылке электронный ресурс;

  • ознакомиться с описанием опыта, кратко записать ход опыта и уравнение реакции;

  • просмотреть видеоролик, записать признаки реакции;

  • сделать вывод о реакционной способности вещества или о способах получения.

Опыт 1. Получение метана

http://files.school-collection.edu.ru/dlrstore/ceac7808-399a-9ac6-db56-7ed204f861fa/index.htm

Опыт 2. Горение метана

http://files.school-collection.edu.ru/dlrstore/5e8f612a-b762-9f6b-de63-c5dc1d3e64c5/index.htm

Опыт 3. Отношение метана к раствору перманганата калия и бромной воде http://files.school-collection.edu.ru/dlrstore/73ccda2d-12a7-ccd3-233c-1ec5450c05f0/index.htm

Сделайте вывод о химической активности алканов — предельных углеводородов.

Ответьте на вопросы:

Какие реакции являются качественными реакциями на непредельные углеводороды.

Практическая работа № 4

«Получение этилена дегидратацией этилового спирта.

Цель работы: научиться получать этилен в лаборатории и проводить качественные реакции на непредельные углеводороды этиленового ряда.

Запишите в тетради тему практической работы. Затем последовательно зафиксируйте в тетради каждый опыт по алгоритму:

  • записать название опыта;

  • открыть по ссылке электронный ресурс;

  • ознакомиться с описанием опыта, кратко записать ход опыта и уравнение реакции;

  • просмотреть видеоролик, записать признаки реакции;

  • сделать вывод о реакционной способности вещества или о способах получения.

Опыт 1. Поучение этилена из этилового спирта

http://files.school-collection.edu.ru/dlrstore/e659f378-5a0f-07e2-5491-fe1ca9d05442/index.htm

Практическая работа № 5

«Взаимодействие этилена с бромной водой, раствором перманганата калия.»

Цель работы: научиться проводить качественные реакции на непредельные углеводороды этиленового ряда.

Запишите в тетради тему практической работы. Затем последовательно зафиксируйте в тетради каждый опыт по алгоритму:

  • записать название опыта;

  • открыть по ссылке электронный ресурс;

  • ознакомиться с описанием опыта, кратко записать ход опыта и уравнение реакции;

  • просмотреть видеоролик, записать признаки реакции;

  • сделать вывод о реакционной способности вещества или о способах получения.

Опыт 1. Взаимодействие этилена с бромной водой

http://files.school-collection.edu.ru/dlrstore/e920f54f-3088-7973-829d-909ad427fbc8/index.htm

Опыт 2. Взаимодействие этилена с раствором перманганата калия

http://files.school-collection.edu.ru/dlrstore/a2fde3eb-37b0-b327-af25-099e3fa00573/index.htm

Сделайте вывод о реакционной способности алкенов.

Вывод:

  1. При взаимодействии этилена с бромной водой, красно-бурый раствор бромной воды обесцвечивается. Эта реакция является качественной на двойную связь.

  2. При окислении этилена водным раствором перманганата калия образуется этиленгликоль. Заметно, что фиолетовая окраска раствора исчезает. Реакция является качественной на двойную связь.

Практическая работа № 6

«Сравнение пламени этилена с пламенем предельных углеводородов (метана, пропан-бутановой смеси)»

Цель работы: научиться сравнивать реакции горения предельных и непредельных углеводородов этиленового ряда.

Запишите в тетради тему практической работы. Затем последовательно зафиксируйте в тетради каждый опыт по алгоритму:

  • записать название опыта;

  • открыть по ссылке электронный ресурс;

  • ознакомиться с описанием опыта, кратко записать ход опыта и уравнение реакции;

  • просмотреть видеоролик, записать признаки реакции;

  • сделать вывод о реакционной способности вещества или о способах получения.

Опыт 1. Горение этилена

http://files.school-collection.edu.ru/dlrstore/80d12abd-df02-dd41-7770-f1cd1f794ccf/index.htm

Ответьте на вопросы:

  1. Что общее и в чем отличие химических свойств метана (алкан) и этилена (алкен)?

  2. Задание:

Смесь этана и этилена объёмом 0,8 л (н.у.) обесцветила 200 г бромной воды с массовой долей 1,6%. Определите объёмную долю каждого газа в смеси.

Вывод: В отличие от метана этилен горит светящимся пламенем, что обусловливается повышенным содержанием углерода.

Практическая работа № 7

«Изучение растворимости спиртов в воде.»

Цель: Изучить растворимость спиртов в воде.

Ход работы

Опыт № 1 Растворимость спиртов в воде

В отдельные пробирки прилейте по 1—2 мл этилового и изоамилового (изопентилового) спиртов. Добавьте к ним по 2—3 мл воды и взболтайте. Отметьте, что этиловый спирт полностью растворился в воде, а изоамиловый спирт отделяется при отстаивании в виде маслянистого слоя над водой.

Вопросы к опыту:

1.    В чем причина различного «поведения» спиртов в воде?
2.    Почему изоамиловый спирт отслаивается над водой, а не наоборот?
3.    Какие органические жидкие вещества при смешивании с водой будут отслаиваться над водой?

Практическая работа № 8

«Окисление спиртов различного строения хромовой смесью.»

Цель: Провести реакции окисления спиртов различного строения хромовой смесью.

Ход работы

Опыт № 1 Окисление этилового спирта хромовой смесью

В пробирке смешайте 2 мл 5% -ного раствора дихромата калия, 1 мл 20%-ного раствора серной кислоты и 0,5 мл этилового спирта. Отметьте цвет раствора. Осторожно нагрейте смесь на пламени горелки до начала изменения цвета. При этом ощущается характерный запах уксусного альдегида, образующегося в результате реакции.

Вопросы к опыту:

1.    Почему цвет раствора меняется с оранжевого до синевато-зеленого? Напишите уравнение реакции окисления этилового спирта.
2.    Можно ли заменить серную кислоту в данной реакции на соляную?

Практическая работа № 9

«Получение диэтилового эфира.»

Цель: Провести реакцию получения диэтилового эфира.

Ход работы

Опыт № 1 Получение диэтилового эфира

Соберите прибор, как показано на рис. 10. В пробирку А налейте 2 мл смеси этилового спирта и серной кислот и для равномерного кипения жидкости добавьте немного тертого кирпича. Пробирку А с помощью пробки с газоотводной трубкой присоедините к пробирке Б. В пробирку Б налейте 2‑3 мл насыщенного раствора хлорида натрия. Пробирку Б поместите в химический стакан с холодной водой и кусочками льда. Отверстие пробирки закройте ватным тампоном.

Реакционную смесь нагревайте в пробирке на слабом пламени спиртовки. При этом можно заметить, что в пробирке Б образуется тонкий слой эфира на поверхности раствора. Закончив нагревание, выньте ватный тампон. Ощущается ли запах эфира?

Задания: 1. Зарисуйте прибор, указав содержимое пробирок А и Б.

2. Напишите уравнение протекающей реакции, отметьте наблюдения.

3. Какова роль серной кислоты в реакции этерификации?

Все ответы на задания и результаты опытов записать в тетрадь.

Практическая работа № 10

«Получение глицерата меди»

Цель: Провести реакции получения глицерата меди.

Ход работы

Опыт № 1 Получение глицерата меди

В пробирку налейте около 1 мл 10% -ного раствора сульфата меди(II) и добавьте немного 10% -ного раствора гидроксида натрия до образования голубого осадка гидроксида меди(II).

К полученному осадку добавьте по каплям глицерин. Взболтайте смесь. Отметьте превращение голубого осадка в раствор темно-синего цвета.

Вопросы к опыту:

1.    Какая реакция лежит в основе получения гидроксида меди(II)? Напишите уравнение этой реакции.
2.    Почему при добавлении глицерина к осадку гидроксида меди(II) осадок растворяется? С чем связано интенсивное окрашивание раствора? Напишите уравнение реакции взаимодействия глицерина с гидроксидом меди(II).
3.    Будут ли этиловый и изоамиловый спирты реагировать с гидроксидом меди(II)?

Практическая работа № 11

«Изучение восстановительных свойств альдегидов: реакция «серебряного зеркала», восстановление гидроксида меди (II). Взаимодействие формальдегида с гидросульфитом натрия»

Цель работы: закрепить знания о свойствах альдегидов и с помощью качественных реакций распознавать альдегиды среди органических соединений.

Оборудование и реактивы приведены после условия каждого этапа  работы.

Правила техники безопасности

Осторожно обращайтесь с химическим оборудованием!

Ход работы:

Опыт №1. Реакция «серебряного зеркала».

Оборудование: штатив с пробирками, спиртовка, спички.

Реактивы:  10%-ный раствор NaOH , CuSO4, NH4OH AgNO3,формалин.

В пробирку налейте 2 мл раствора формалина и добавьте несколько капель аммиачного раствора нитрата серебра.

Пробирку нагрейте до появления серебра на стенках пробирки.

Запишите уравнение реакции.

Опыт № 2.Взаимодействие этаналя с гидроксидом меди (II).

Оборудование: штатив с пробирками, спиртовка, спички.

Реактивы:  10%-ный раствор NaOH , CuSO4, этаналь.

1.Налейте в пробирку 1 мл раствора этаналя и столько же раствора гидроксида натрия.

2. Затем добавьте несколько капель раствора сульфата меди (II). Пробирку с полученным раствором нагрейте. Что наблюдаете?

3.Напишите уравнение реакции между сульфатом меди (II) и гидроксидом натрия. Напишите уравнение реакции взаимодействия этаналя с полученным раствором гидроксидом меди (II).

Вывод: Как обнаружить альдегиды?

Запишите общий вывод по работе.

Практическая работа № 12

«Взаимодействие формальдегида с гидросульфитом натрия»

Цель работы: закрепить знания о свойствах альдегидов и с помощью качественных реакций распознавать альдегиды среди органических соединений.

Оборудование и реактивы приведены после условия каждого этапа  работы.

Правила техники безопасности

Осторожно обращайтесь с химическим оборудованием!

Ход работы:

Опыт №1. Окисление бензальдегида кислородом воздуха.

Оборудование: часовое или предметное стекло.

Реактивы:  10%-ный раствор бензойного альдегида.

1.На предметное стекло поместите 2 капли бензальдегида и оставьте на 30 минут. Наблюдайте образование белых кристаллов по краям капли. Происходит реакция окисления альдегида и образуется бензойная кислота. Запишите уравнение реакции.

Вывод: До каких продуктов окисляются альдегиды?

Опыт № 2. Получение ацетона из ацетата натрия.

Оборудование: штатив с пробирками, спиртовка, спички.

Реактивы: крист.ацетат натрия- СН3СООNa, конц. раствор HCI, Н2О.

1.В пробирку поместите порошок ацетата натрия и укрепите в лабораторном штативе. Закройте пробирку пробкой с газоотводной трубкой, конец которой опустите в пробирку с водой.

2.Пробирку с ацетатом натрия нагрейте на спиртовке и наблюдайте выделение газа в пробирке с водой. Чувствуется своеобразный запах ацетона.

3.После нагревания в пробирку, где был ацетат натрия, прилейте две – три капли конц. раствора HCI. Наблюдайте выделение газообразного вещества. Определите этот газ. Запишите уравнения реакций.

Вывод: Как можно в лаборатории получить ацетон?

Запишите общий вывод по работе.

Практическая работа № 13

«Растворимость различных карбоновых кислот в воде. Взаимодействие уксусной кислоты с металлами.»

Цель работы:

  1. Получить уксусную кислоту и изучить её свойства; закрепить знания о карбоновых кислотах,

Реактивы и оборудование: 

  1. ацетат натрия, серная кислота (конц.), магний (порошок), цинк, карбонат натрия, фенолфталеин, универсальная индикаторная бумага, гидроксид натрия, прибор для получения  и собирания кислоты, спиртовка, пробирку, вата, спички. 

  2. спиртовка, водяная баня, колба с обратным холодильником, стеклянная палочка, химические стаканы; концентрированная серная кислота,

  3. конические колбы с пробками, мерные цилиндры, пробирки, разбавленные растворы соляной или серной кислот,

  4. металлический штатив, спиртовка, газоотводная трубка с пробкой, пробирки, химический стакан, ватный тампон, тертый кирпич;

Ход работы

    1. Напишите возможные изомеры для соединений с формулой С5Н10О2

    2. С какими из перечисленных веществ будет реагировать уксусная кислота: оксид магния, гидроксид алюминия, сульфат бария, карбонат калия, формиат натрия, цинк? Запишите уравнения возможных реакций.

    3. Как осуществить превращения: этан – этилен – ацетилен – ацетальдегид – уксусная кислота – метиловый эфир уксусной кислоты.

Внимание!!! Работа с кислотами!! Соблюдайте ТБ! 

Опыт 1. Получение уксусной кислоты.

В пробирку с ацетатом натрия прибавить 1- 2 мл концентрированной серной кислоты. Закрыть пробирку  пробкой с газоотводной трубкой, конец которой опустите в другую пробирку вход в пробирку прикрыть ваткой, смотрите рисунок:

Смесь в пробирке осторожно нагревайте до тех пор, пока в приёмнике – пробирке не собёрётся 1 -2 мл жидкости. Прекратите нагревание, закройте спиртовку.

Опустите в пробирку с образовавшейся жидкости универсальную индикаторную бумагу. Как изменился цвет индикатора? Почему? Запишите уравнение диссоциации уксусной кислоты.

Опишите запах, образовавшейся жидкости? Соблюдайте осторожность при определении запаха! Составьте  уравнение данной химической реакции. 

Опыт 2. Взаимодействие уксусной кислоты с металлами. 

Посмотрите видео-опыт  «Взаимодействие уксусной кислоты с металлами» 

В  одну пробирку положите гранулу цинка, в другую порошок магния. В обе пробирки прилейте 1 мл уксусной кислоты. Что наблюдаете? Сравните скорость этих реакций? Запишите соответствующие уравнения химических реакций, назовите продукты, укажите тип реакции.

Все ответы на задания и результаты опытов записать в тетрадь.

Практическая работа № 14

«Получение изоамилового эфира уксусной кислоты.»

Цель работы:

1. Закрепить знания о карбоновых кислотах,

  1. Получить изоамиловый эфир уксусной кислоты, исследовать его физические свойства.

Реактивы и оборудование: 

1. уксусная кислота, спиртовка, пробирку, вата, спички, изоамиловый спирт. 

2.спиртовка, колба с обратным холодильником, стеклянная палочка, химические стаканы; изоамиловый спирт, концентрированная уксусная кислота, концентрированная серная кислота

3.конические колбы с пробками, мерные цилиндры,

4.металлический штатив, спиртовка, газоотводная трубка с пробкой, пробирки, химический стакан, ватный тампон, тертый кирпич;

Ход работы

    1. Запишите уравнение этерификации пропанола и масляной кислоты.

Внимание!!! Работа с кислотами!! Соблюдайте ТБ! 

Опыт 1. Получение изоамилового эфира уксусной кислоты, исследование его физических свойств.

Соберите прибор, как показано на рис. 10. В пробирку А налейте 2 мл смеси изоамилового спирта, уксусной и серной кислот и для равномерного кипения жидкости добавьте немного тертого кирпича. Пробирку А с помощью пробки с газоотводной трубкой присоедините к пробирке Б. В пробирку Б налейте 2‑3 мл насыщенного раствора хлорида натрия. Пробирку Б поместите в химический стакан с холодной водой и кусочками льда. Отверстие пробирки закройте ватным тампоном.

Реакционную смесь нагревайте в пробирке на слабом пламени спиртовки. При этом можно заметить, что в пробирке Б образуется тонкий слой эфира на поверхности раствора. Закончив нагревание, выньте ватный тампон. Ощущается ли запах эфира?

Задания: 1. Зарисуйте прибор, указав содержимое пробирок А и Б.

2. Напишите уравнение протекающей реакции, отметьте наблюдения.

3. Какова роль серной кислоты в реакции этерификации?

Все ответы на задания и результаты опытов записать в тетрадь.

Практическая работа № 15

«Сравнение степени ненасыщенности твердого и жидкого жиров.»

Цель работы:

1.Изучить некоторые физические и химические свойства жиров.

2.Получить сложные жиры карбоновых кислот и исследовать их физические свойства.

Реактивы и оборудование: 

1. спиртовка, водяная баня, колба с обратным холодильником, стеклянная палочка, химические стаканы; твердый жир, 15 % спиртовой раствор щелочи, этиловый спирт, концентрированная уксусная кислота, концентрированная серная кислота, раствор растительного масла (2 капли масла на 1 мл гексана или очищенного керосина), бромная вода, насыщенный раствор поваренной соли, 5 % раствор соды, раствор мыла, раствор белка, бензин, толуол, 5 % раствор КОН, 5 % раствор Na2CO3.

2.конические колбы с пробками, мерные цилиндры, навески мыла и синтетического моющего средства, пробирки, разбавленные растворы соляной или серной кислот, раствор гидроксида натрия или калия, раствор ацетата свинца, сульфат меди (II), фенолфталеин, жесткая вода.

Ход работы

1.Напишите уравнение получения триглицерида, если в его состав входят остатки пальмитиновой, стеариновой и олеиновой кислот.

Внимание!!! Работа с кислотами!! Соблюдайте ТБ! 

Опыт 1. Получение сложных жиров карбоновых кислот.

В пробирку налейте 1 мл этилового спирта, 1 мл концентрированной уксусной кислоты и 0,5-1 мл концентрированной серной кислоты. Смесь в пробирке осторожно перемешайте и нагрейте на водяной бане, не доводя до кипения. Окончание реакции определите по появлению запаха сложного эфира, отличного от запаха карбоновой кислоты и спирта, взятых для синтеза. Дайте жидкости остыть и вылейте ее в стаканчик с насыщенным раствором поваренной соли. Какой ощущается запах? Где собирается эфир? Какова его растворимость? Какую консистенцию он имеет? Для чего используется кислота в процессе синтеза сложного эфира?

Задания:

1. Составьте в структурном виде уравнение реакции получения сложного эфира, укажите условия ее протекания.

2. Опишите физические свойства полученного эфира, ответьте на вопросы, поставленные в тексте эксперимента.

Опыт 2. Физические свойства жиров.

а) Растворимость жиров в различных растворителях.

В 4 пробирки поместите 1-2 капли растительного масла. Прилейте в первую пробирку 1 мл этилового спирта, во вторую – 1 мл бензина, в третью – 1 мл воды, в четвертую – 1 мл толуола.

Взболтайте содержимое пробирок и дайте постоять. В каждой ли пробирке растворился жир? Какие вещества являются хорошими растворителями жиров, а какие – плохими? Почему?

Задания:

1. Результаты эксперимента оформите в виде таблицы.

2. Сделайте вывод о растворимости жиров на основании опыта.

б) Эмульгирование жиров.

Если жиры хорошо взболтать с водой, то они образуют эмульсию, т.е. систему, в которой мелкие капельки жира взвешены в воде. Эмульсия масла в воде быстро разрушается, т.к. капельки жира, сталкиваясь друг с другом, образуют крупные капли, создающие слой жира на поверхности воды. Есть вещества, которые, адсорбируясь на поверхности капель, не дают соединиться каплям жира в более крупные, т.е. повышают устойчивость эмульсии – эмульгаторы.

В 5 пробирок налейте по 3-4 капли растительного масла. Добавьте в первую пробирку 5 мл воды, во вторую – 5 мл 5 % раствора КОН, в третью – 5 мл 5 % раствора соды, в четвертую – 5 мл раствора мыла, в пятую – 5 мл раствора белка. Сильно встряхните содержимое каждой пробирки и наблюдайте образование эмульсии.

Задания:

1. Результаты эксперимента оформите в виде таблицы.

2. Сделайте вывод об эмульгирующих свойствах различных веществ.

Опыт 3. Выделение жирных кислот.

а) Рассчитайте необходимый объем дистиллированной воды для приготовления 1 % растворов из выданных навесок мыла и синтетического моющего средства (СМС). Приготовьте растворы.

б) Налейте в пробирку 1 мл приготовленного раствора мыла и прибавьте разбавленной соляной или серной кислоты до образования хлопьев. Что собой представляет этот осадок? Проверьте, растворяется ли осадок в растворе щелочи. Объясните это явление.

Задания:

1. Составьте уравнения соответствующих реакций, запишите наблюдения.

2. Ответьте на вопросы, поставленные в тексте эксперимента.

Опыт 4. Получение нерастворимых солей жирных кислот.

В две пробирки налейте по 1 мл мыльного раствора, добавьте в них соответственно растворы ацетата свинца Pb(CH3COO)2 и сульфата меди (II) CuSO4.

Задание: Объясните изменения, происходящие в каждой пробирке, запишите уравнения соответствующих реакций и наблюдения.

Все ответы на задания и результаты опытов записать в тетрадь.

Практическая работа № 16

«Омыление жира.»

Цель работы:

1.Закрепить знания о сложных эфирах и жирах.

2.Изучить некоторые физические и химические свойства жиров.

3.Получить сложные жиры карбоновых кислот и исследовать их физические свойства.

Реактивы и оборудование: 

1.спиртовка, водяная баня, колба с обратным холодильником, стеклянная палочка, химические стаканы; твердый жир, 15 % спиртовой раствор щелочи

2.конические колбы с пробками, мерные цилиндры, пробирки, разбавленные растворы соляной или серной кислот, раствор гидроксида натрия или калия,

Ход работы

1.Что такое мыло и какая реакция лежит в основе его производства? Напишите уравнение реакции.

Внимание!!! Работа с кислотами!! Соблюдайте ТБ! 

Опыт 6. Омыление жиров в водно-спиртовом растворе.

Соберите прибор, как показано на рис. 9. В круглодонную колбу вместимостью 20 мл поместите 3-4 г измельченного твердого жира и налейте 8 мл 15 % спиртового раствора NaOH.

Перемешайте смесь стеклянной палочкой, колбу со смесью закройте обратным холодильником, опустите в водяную баню, закрепив в лапке штатива, нагрейте в течение 15-20 минут. Омыление жира следует проводить до тех пор, пока содержимое колбы не станет однородным. Оно обычно заканчивается образованием плотной твердой пленки на дне колбы. Затем колбу извлеките из водяной бани, дайте ей остыть, и добавьте в нее воды, хорошо взболтайте. Сравните растворимость полученного вещества с растворимостью жира, из которого оно было получено.

Задания:

1. Составьте уравнение реакции щелочного омыления твердого тристеарина, укажите условия ее протекания и наблюдения.

2. Как доказать, что продуктом данной реакции является мыло?

Все ответы на задания и результаты опытов записать в тетрадь.

Практическая работа № 17

«Получение мыла и изучение его свойств: пенообразования, реакций ионного обмена, гидролиза, выделения свободных жирных кислот»

Цель работы:

1.Сравнить свойства мыла и синтетических моющих средств.

Реактивы и оборудование: 

1.спиртовка, водяная баня, колба с обратным холодильником, стеклянная палочка, химические стаканы; твердый жир, 15 % спиртовой раствор щелочи, насыщенный раствор поваренной соли, 5 % раствор соды, раствор мыла, раствор белка, бензин, толуол, 5 % раствор КОН, 5 % раствор Na2CO3.

2.конические колбы с пробками, мерные цилиндры, навески мыла и синтетического моющего средства, пробирки, разбавленные растворы соляной или серной кислот, раствор гидроксида натрия или калия, раствор ацетата свинца, сульфат меди (II), фенолфталеин, жесткая вода.

Ход работы

Внимание!!! Работа с кислотами!! Соблюдайте ТБ! 

Опыт 1. Сравнение свойств мыла и синтетических моющих средств.

а) Влейте по 2-3 мл растворов мыла и СМС в пробирки, добавьте к ним 2‑3 капли раствора фенолфталеина. Каков цвет раствора? Почему?

Задания:

1. Напишите уравнение реакции гидролиза мыла, запишите наблюдения.

2. Ответьте на вопросы, поставленные в тексте эксперимента.

б) В 2 пробирки влейте по 4-5 мл жесткой воды, содержащей ионы Са2+ и Mg2+. В первую пробирку при встряхивании добавьте раствор мыла, во вторую – раствор синтетического моющего средства. В каком случае приходится прибавлять больше раствора до образования устойчивой пены? Какой препарат не утрачивает моющего действия в жесткой воде? Почему?

Задания:

1. Напишите уравнение реакции, происходящей в растворе мыла.

2. Запишите наблюдения явлений, происходящих в обеих пробирках, объясните их причину.

На основании проделанных опытов сделайте вывод о преимуществах и недостатках мыла и СМС.

Все ответы на задания и результаты опытов записать в тетрадь.

Практическая работа № 11

«Реакция «серебряного зеркала» глюкозы. Взаимодействие глюкозы с гидроксидом меди (II) при различных температурах.»

Цель работы: Исследовать химические свойства углеводов и объяснить причины, обуславливающие эти свойства.

Оборудование и реактивы: металлический штатив, спиртовка, пробирки, стеклянная палочка, химический стакан вместимостью 50 мл, электроплитка, водяная баня; 1 % раствор глюкозы, 1 % растворы сахарозы, лактозы, фруктозы; крахмал, спиртовой раствор иода, раствор сульфата меди (II), раствор гидроксида натрия (10-12 %), раствор серной кислоты (1:5), аммиачный раствор оксида серебра (I).

Ход работы:

ОПЫТ 1. Свойства глюкозы.

Внесите в пробирку 3 капли раствора глюкозы, одну каплю раствора соли меди и прибавьте при взбалтывании несколько капель гидроксида натрия до образования светло-синего раствора (щелочь должна быть в избытке). Что доказывает появление такой окраски раствора? Вспомните реакцию образования глицерата меди.

Полученный раствор нагрейте. Что наблюдается? Наличие какой функциональной группы в молекуле глюкозы подтверждает этот опыт?

Задания:

1. Напишите уравнение реакции взаимодействия глюкозы с гидроксидом меди (II) при комнатной температуре. Укажите наблюдения. На наличие каких функциональных групп указывает эта реакция?

2. Напишите уравнение реакции взаимодействия глюкозы с гидроксидом меди (II) при нагревании. Что наблюдается? Наличие какой функциональной группы в молекуле глюкозы подтверждает этот опыт?

ОПЫТ 2. Взаимодействие сахаров с гидроксидом меди (II).

Опыт проводят одновременно с растворами различных сахаров.

К 2 мл раствора сахара добавьте 1 мл разбавленного раствора щелочи и 3‑4 капли раствора сульфата меди (II). Встряхните пробирку и перемешайте содержимое пробирки стеклянной палочкой до растворения осадка. Жидкость при этом окрашивается в интенсивно-синий цвет.

Затем поместите все пробирки в нагретую водяную баню. Если сахар окисляется, то, вынув пробирку через 2-3 минуты, вы увидите изменения окраски и появление красного или коричневого осадка.

Задание: Заполните следующие таблицы:

Результаты опыта, проведенного при нормальных условиях

Углевод

Что наблюдается?

Как объясняется?

Результаты опыта, проведенного при нагревании

Углевод

Что наблюдается?

Как объясняется?

ОПЫТ 3. Взаимодействие сахаров с аммиачным раствором оксида серебра (I).

Опыт проводят одновременно с растворами различных сахаров.

Налейте в тщательно вымытые и высушенные пробирки по 1 мл аммиачного раствора оксида серебра (I) и по 1 мл раствора сахара. Пробирки поместите на несколько минут в горячую водяную баню.

Задание: Заполните следующую таблицу:

Результаты опыта, проведенного при нагревании

Углевод

Что наблюдается?

Как объясняется?

Запишите уравнение соответствующей реакции для глюкозы.

Все ответы на задания и результаты опытов записать в тетрадь.

Практическая работа № 19

« Обнаружение лактозы в молоке.»

Цель работы: Исследовать химические свойства углеводов и объяснить причины, обуславливающие эти свойства.

Оборудование и реактивы: металлический штатив, спиртовка, пробирки, стеклянная палочка, химический стакан вместимостью 50 мл, электроплитка, водяная баня; 1 % раствор глюкозы, 1 % растворы сахарозы, лактозы, фруктозы; крахмал, спиртовой раствор иода, раствор сульфата меди (II), раствор гидроксида натрия (10-12 %), раствор серной кислоты (1:5), аммиачный раствор оксида серебра (I).

Ход работы:

ОПЫТ № 1: Обнаружение лактозы в молоке.

В молоке дисахарид лактозу обнаруживают реакцией Фелинга, содержащего комплексно связанные с виннокислой кислотой ионы Cu2+. В результате реакции образуется оксид меди (I), выделяющийся в виде красного осадка Cu2O.

Предварительно осаждают белки молока добавлением трихлоруксусной кислоты (ТХУ) и фильтруют. К 10 каплям фильтрата добавляют 10 капель дистиллированной воды, 10 капель NaOH и 6 капель реактива Фелинга. Смесь нагревают. Отмечают характер появляющегося окрашивания.

Задание: Запишите все происходящие наблюдения. Оформите наблюдения в виде схемы.

Все ответы на задания и результаты опытов записать в тетрадь.

Практическая работа № 20

«Образование солей анилина. Бромирование анилина.»

Цель работы:

1. Изучить химические свойства анилина.

2. Выполнить и запомнить качественные реакции на анилин.

Оборудование и реактивы: спиртовка, держатель для пробирок, пробирки, раствор гидроксида натрия (10-12 %), раствор сульфата меди (II) (0,5 моль/л), водный раствор яичного белка (готовится из расчета 1 мл белка на 5 мл насыщенного раствора поваренной соли), насыщенный раствор ацетата свинца, насыщенный раствор сульфата меди (II), концентрированная азотная кислота, концентрированная соляная кислота, концентрированная серная кислота, насыщенный раствор сульфата аммония, этиловый спирт, концентрированный раствор гидроксида натрия, концентрированный раствор аммиака, раствор ацетата свинца (0,5 моль/л).

ХОД РАБОТЫ

ОПЫТ 1. Образование солей анилина

В пробирку налейте 0,5 мл анилина и 3 мл дистиллированной воды. Взболтайте. Что наблюдаете? В пробирку добавьте соляной кислоты до полного растворения анилина в воде. Добавьте в пробирку 1—2 мл раствора щёлочи. Что наблюдаете?

Вопросы

1. Почему при добавлении соляной кислоты происходит растворение анилина? Напишите соответствующее уравнение реакции.

2. Почему при добавлении щёлочи анилин выделяется из водного раствора? Напишите уравнение реакции.

ОПЫТ 8. Бромирование анилина

В пробирку налейте 0,5 мл анилина и 0,5 мл дистиллированной воды. Прибавьте по каплям бромной воды до появления осадка.

Задания:

1. Почему обесцвечивается бромная вода?

2. Каково строение образующегося осадка? Напишите уравнение реакции.

Все ответы на задания и результаты опытов записать в тетрадь.

Практическая работа № 21

«Образование солей глицина. Получение медной соли глицина.»

Цель работы:

1. Изучить образование солей глицина.

Оборудование и реактивы: спиртовка, держатель для пробирок, пробирки, раствор гидроксида натрия (10-12 %), раствор сульфата меди (II) (0,5 моль/л), водный раствор яичного белка (готовится из расчета 1 мл белка на 5 мл насыщенного раствора поваренной соли), насыщенный раствор ацетата свинца, насыщенный раствор сульфата меди (II), концентрированная азотная кислота, концентрированная соляная кислота, концентрированная серная кислота, насыщенный раствор сульфата аммония, этиловый спирт, концентрированный раствор гидроксида натрия, концентрированный раствор аммиака, раствор ацетата свинца (0,5 моль/л).

ХОД РАБОТЫ

ОПЫТ 1. Амфотерные свойства аминокислот

В пробирку налейте 2—3 мл раствора карбоната натрия и всыпьте щепотку глицина. Что наблюдаете? Напишите уравнение реакции.

Поместите в пробирку немного кристалликов глицина, смочите их несколькими каплями соляной кислоты и нагрейте. Что наблюдаете? Вылейте несколько капель образовавшегося раствора на часовое стекло. Наблюдайте образование при охлаждении кристаллов соли глицина. Напишите уравнение реакции.

Задания:

1. Какие свойства глицина проявляются в каждой из этих реакций?

2. Сравните форму кристаллов глицина и гидрохлорида глицина. Чем они отличаются?

ОПЫТ 2. Получение медной соли глицина

В пробирку, содержащую 2 мл раствора глицина, добавьте 1 г порошка оксида меди (II) и нагрейте до кипения.

Задания:

1. Чем обусловлено появление голубой окраски раствора?

2. Каково строение образующейся соли?

Все ответы на задания и результаты опытов записать в тетрадь.

Практическая работа № 22

«Денатурация белка. Цветные реакции белков»

Цель работы:

1. Изучить химические свойства белков.

2. Выполнить и запомнить качественные реакции на белки.

Оборудование и реактивы: спиртовка, держатель для пробирок, пробирки, раствор гидроксида натрия (10-12 %), раствор сульфата меди (II) (0,5 моль/л), водный раствор яичного белка (готовится из расчета 1 мл белка на 5 мл насыщенного раствора поваренной соли), насыщенный раствор ацетата свинца, насыщенный раствор сульфата меди (II), концентрированная азотная кислота, концентрированная соляная кислота, концентрированная серная кислота, насыщенный раствор сульфата аммония, этиловый спирт, концентрированный раствор гидроксида натрия, концентрированный раствор аммиака, раствор ацетата свинца (0,5 моль/л).

ХОД РАБОТЫ

ОПЫТ 1. Свертывание белков при нагревании.

2-3 мл раствора белка налейте в пробирку и нагрейте в пламени спиртовки до кипения. Что при этом наблюдается? Чем можно объяснить это явление? Содержимое пробирки разбавьте водой. Растворяется ли осадок, если нет, то почему?

Задания:

1. Запишите в виде схемы ход эксперимента и соответствующие наблюдения.

2. Ответьте на вопросы, поставленные в тексте.

ОПЫТ 2. Осаждение белков солями тяжелых металлов.

В две пробирки налейте по 1-2 мл раствора белка и медленно, по каплям, при встряхивании прилейте в одну из них насыщенный раствор сульфата меди (II), а в другую – насыщенный раствор ацетата свинца. Что наблюдается? Затем содержимое пробирок разбавьте большим количеством воды. Что наблюдается при этом?

Задания:

1. Запишите в виде схемы ход эксперимента и соответствующие наблюдения.

2. Ответьте на вопросы, поставленные в тексте. Сделайте вывод о действии солей тяжелых металлов на белок.

ОПЫТ 3. Осаждение белков минеральными кислотами.

Налейте в одну пробирку 1 мл концентрированной азотной кислоты, в другую – 1 мл концентрированной соляной кислоты, в третью – 1 мл концентрированной серной кислоты. Каждую пробирку наклоните и осторожно влейте в нее по стенке 1-1,5 мл раствора белка так, чтобы он не смешивался с более тяжелым слоем кислоты, затем пробирку поставьте в штатив. Что наблюдается на границе раздела двух жидкостей?

Затем пробирки встряхните. Какие изменения происходят при этом в пробирках?

Задания:

1. Запишите в виде схемы ход эксперимента и соответствующие наблюдения.

2. Ответьте на вопросы, поставленные в тексте. Сделайте вывод об отношении белка к концентрированным кислотам.

ОПЫТ 4. Высаливание белков сульфатом аммония.

В пробирку налейте 1-1,5 мл раствора белка, прилейте равный объем насыщенного раствора сульфата аммония. Смесь слегка встряхните. Что наблюдается? Что происходит с белком?

Смесь разбавьте большим количеством воды. Что происходит с осадком? Какой вид свертывания белка наблюдается?

Задания:

1. Запишите в виде схемы ход эксперимента и соответствующие наблюдения.

2. Ответьте на вопросы, поставленные в тексте.

ОПЫТ 5. Осаждение белков этиловым спиртом.

В пробирку налейте 1-1,5 мл раствора белка, прилейте 2-3 мл этанола. Что наблюдается? Проверьте, растворяется ли осадок в воде. Какой вид свертывания белка наблюдается?

Задания:

1. Запишите в виде схемы ход эксперимента и соответствующие наблюдения.

2. Ответьте на вопросы, поставленные в тексте.

ОПЫТ 6. Цветные реакции на белки.

а) Ксантопротеиновая реакция.

К 1 мл раствора белка добавьте 5-6 капель концентрированной азотной кислоты до появления белого осадка или мути от свертывания белка. Реакционную смесь нагрейте до окрашивания осадка в желтый цвет. В процессе гидролиза, происходящем при этом, осадок может частично растворяться. Смесь охладите и добавьте к ней осторожно, по каплям, избыток концентрированного раствора аммиака. Окраска при этом переходит в оранжевую. Данная реакция является качественной на белки.

Задания:

1. Запишите соответствующие наблюдения.

2. Ответьте на вопрос: какие группы атомов, остатки молекул каких органических веществ позволяют обнаружить эта реакция?

б) Биуретовая реакция.

В пробирку налейте 1-2 мл раствора белка, равный объем концентрированного раствора щелочи и 2-3 капли разбавленного раствора сульфата меди (II). Содержимое пробирки тщательно перемешайте. Жидкость при этом окрашивается в ярко-фиолетовый цвет. Эта реакция также является качественной на белки.

Задания:

1. Запишите соответствующие наблюдения.

2. Ответьте на вопрос: какие группы атомов, остатки молекул каких органических веществ позволяют обнаружить эта реакция?

в) Сульфгидрильная реакция.

В пробирку налейте 0,5 мл раствора ацетата свинца и 2 мл раствора гидроксида натрия до растворения образовавшегося осадка гидроксида свинца (II). В результате получается плюмбит натрия Na2PbO2. В другую пробирку налейте 2 мл белка и прилейте из первой пробирки столько же плюмбита натрия.

Смесь нагрейте до кипения. При нагревании белок гидролизуется, и образуется сероводород, который вступает во взаимодействие с раствором Na2PbO2 с образованием черного или черно-коричневого осадка сульфида свинца PbS.

Задания:

1. Запишите все соответствующие наблюдения.

2. Напишите уравнения реакций образования гидроксида свинца (II) и плюмбита натрия.

3. Что позволяет обнаружить данная реакция в белке?

Все ответы на задания и результаты опытов записать в тетрадь.

Баритовая вода — Справочник химика 21





    Баритовая вода Белая сажа Белила [c.264]

    Получение нерастворимых карбонатов. I. В пробирку с 1—2 мл известковой или баритовой воды пропустите ток СО2 до образования осадка. Напишите уравнение реакции. [c.206]

    Через несколько минут баритовая вода мутнеет [c.169]

    Некоторые физические и химические свойства стереоизомерных форм сильно различаются между собой. Малеиновая кислота плавится прн 130°, фумаровая—при 287° первая легко растворима в воде и осаждается баритовой водой последняя трудно растворима и не осаждается баритовой водой. [c.346]










    Бесцветный Нет Вызывает помутнение баритовой воды СО2 Карбонаты [c.44]

    При нагревании смеси метилового спирта и уксусной кислоты с несколькими каплями концентрироваиной серной кислоты образовалось 2,22 г сложного эфира. При обработке того же количества исходной смеси водным раствором гидрокарбоната натрия и пропускании образовавшегося газа в избыток баритовой воды выпало [c.44]

    При нагревании с баритовой водой гуанидин гидролизуется до мочевины и аммиака  [c.290]

    БАРИТОВАЯ ВОДА — насыщенный водный раствор Ва (0Н)2, применяется для поглощения Oj в аналитической химии и в газовом анализе. [c.39]

    Бесцветный Резкий Вызывает помутнение баритовой воды, обесцвечивает иодкрахмальную бумагу SO2 Сульфиды, сульфиты, тиосульфаты, сульфа ты некоторых тяжелых металлов [c.44]

    При процессах гниения или при кипячении с баритовой водой холин теряет молекулу воды и образуется сиропообразное ядовитое вещество нейрин  [c.308]

    Гидроксиды стронция и бария Sr(0H)2 и Ва(0Н)2 представляют собой сильные основания, лучше растворимые в воде, чем гидроксид кальция один литр воды при 20° С растворяет 8 г гидроксида стронция и 38 г гидроксида бария. Насыщенный раствор гидроксида бария называется баритовой водой и часто применяется в качестве реактива. [c.393]

    ПО. Газообразные продукты сожжения смеси бензола и анилина пропустили в баритовую воду, в результате выделилось 59,4 г осадка. Определить процентный состав исходной смеси, если при обработке такого Hie количества смеси сухим хлороводородом может образоваться 2,59 г осадка. [c.49]

    В пробирку с газоотводной трубкой возьмите 2—3 микрошпателя древесного угля, прибавьте 2—3 мл концентрированной серной кислоты и слегка подогрейте. К концу газоотводной трубки поднесите синюю лакмусовую бумажку, смоченную водой. Соберите выделяющийся газ в пробирку (для этого опустите газоотводную трубку до дна пробирки) и внесите в него горящую лучинку. Пропустите ток газа через раствор баритовой воды и подкисленный раствор перманганата калия. Что наблюдается Какой кислотно-основной характер имеют образующиеся оксиды и каковы их свойства Напишите уравнения реакций. [c.58]

    Результат опыта. Через несколько минут после погружения в суспензию кровяного угля органического вещества баритовая вода в промывной склянке 7 начинает сильно мутнеть, что указывает на появление в системе довольно значительных количеств. СО2. В склянке 3 баритовая вода остается прозрачной. Следовательно, СО2 в системе появилась в результате сгорания органического вещества, вошедшего в соприкосновение с суспензией кровяного угля, играющего роль катализатора. Причем, это сгорание, как видим, происходит при низких температурах. [c.97]

    Продукты полного сгорания (в избытке кислорода) смеси, пропана и метиламина обработали избытком баритовой воды. При этом образовалось 137,9 г, осадка. Газообразные вещества, не поглотившиеся баритовой водой, были пропущены над раскаленной медной спиралью, после чего объем газа оказался в 2,5 раза меньше объема исходной смеси пропана и метиламина. Определить процентный состав исходной смеси газов. [c.51]










    Очищая от диоксида углерода, газ пропускают через поглотительные склянки с концентрированным раствором КОН, а также через колонки с натронной известью или с аскаритом (КОН на волокнистом асбесте). Полноту удаления СО2 контролируют баритовой водой. [c.38]

    Карбонат-ионы — по образованию белого осадка карбоната бария при взаимодействии диоксида углерода, выделяющегося из подкисленного раствора, с баритовой водой. Если раствор содержит сульфит- и тиосульфат-ионы, следует предварительно добавить к раствору 4—5 капель раствора пероксида водорода для превращения их в сульфаты. [c.221]

    Гетерогенная кислотно-основная реакция (растворение соли). Кусочек мрамора в пробирке обливают разбавленной соляной кислотой. У отверстия пробирки держат стеклянную палочку с висящей на ней каплей баритовой воды. В капле образуется муть от выпадающего ВаСОз. Если у отверстия пробирки будет находиться капля раствора ВаСЬ, помутнения не произойдет (почему ). [c.566]

    Газ из колонки с силикагелем поступал в стеклянную трубку, в которой имелась платиновая спираль, накаливаемая электрическим током. Поскольку через колонку после анализируемого газа пускался воздух, очищенный от СО2, то в трубке с платиновой спиралью происходило сожжение сначала метана, затем этана и т. д. При этом образовывался СО2, количество которого первоначально фиксиро-валвсь по помутнению баритовой воды, т. е. раствора гидроокиси [c.224]

    В пробирку с газоотводной трубкой внесите небольшое количество гидроксокарбоната меди (II) — Си2(ОН)2СОз. Конец газоотводной трубки держите над поверхностью баритовой (или известковой) воды. Нагрейте пробирку с солью. Наблюдайте изменение окраски вещества, конденсацию воды на холодных частях прибора, а также помутнение баритовой воды. Напишите уравнение реакции разложения. [c.57]

    Золи кремниевых кислот обладают гидрофильными свойствами. Прибавлением электролитов не всегда удается вызвать коагуляцию. В литературе имеются указания на то, что сравнительно быструю коагуляцию можно вызвать добавлением к золю кремниевых кислот баритовой воды или концентрированного раствора сульфата алюминия. [c.103]

    Выделяющийся при этом СОз реагирует с баритовой водой Ва(0Н)2 с образованием белого малорастворимого в воде осадка ВаСОд [c.167]

    При окислении 100 г раствора формальдегида и этилового спирта в воде перма-нгаиатом калия образовалось 30 г органической кислоты и. газообразно б вещество, которо е при пропускании в избыток баритовой воды дает 20 г осадка. 0предел ить процентную концентрацию формальдегида -и спирта в смеои. [c.42]

    Pi створ Ва(0Н)2 (баритовая вода) — лабораторный реактив для открытия С0а-Са(0Н)2 (известковое молоко, гашеная известь) применяется в качесгве дешевого растворимого основания. [c.481]

    Образовавшуюся углекислоту затем поглощают раствором щелочи или баритовой, водой. Можно вести также окисление окиси углерода кислородом на специальном катализаторе — гепталите (смесь активной Д1 уокиси марганца с окисью меди в отношении 6 4). Полученный углекислый газ поглош.ается титрованным раствором баритовой воды , по обратному титрованию неизрасходовааной щелочи рассчитывают содержание окиси углерода. [c.828]

    При сожже Нии смеси бензола и анилина образовалось 6,94 л газообразных продуктов, при пропускании которых в избыток баритовой воды образовалось 59,4 г осадка. Найти процентный состав смеси  [c.50]

    Гидролиз галоидалкилов при нагревании с водой не всегда протекает достаточно легко. Лучше применять для этой цели разбавленные растворы щелочи или карбонаты щелочных металлов, окись свинца и известковую или баритовую воду, при действии которых галоидалкилы иногда очень гладко омыляются до спиртов  [c.109]

    Полимеризация формальдегида под действием известковой или баритовой воды была осушествлена еще в 1861 г. А. М. Бутлеровым. Этим способом Бутлеров впервые получил синтетическим путем сахаристое вещество (мeтuлeaитaиJ, — Прим, редактора]. [c.211]

    Пулегон может быть получен из цитронеллаля (Тиман и Шмидт). При кипячении с уксусным ангидридом этот природный альдегид превращается в ацетат нзопулегола, который может быть окислен хромовой кислотой до изопулегона. При действии баритово ) воды изопулегон перегруппировывается в пулегон, причем в качестве промежуточного продукта, вероятно, образует гидрат, который затем легко отщепляет воду  [c.827]

    Почему при отсутствии иавестковои воды как реактива на O.j можно при )енять баритовую воду — водный раствор гидроокиси бария Ответ обосновать приведением уравнения соответствуюш,ей реа4[c.102]

    Помутнение известковой или баритовой воды происходит также при взаимодействии с диоксидом серы, например Са (ОН ), + SO, = aSO , + И,О [c.209]

    Образование мелких капель воды на стенках пробирки / и газоотводной трубки, а также посинение сульфата меди (образование Си504-5Н.20) укажут на присутствие в испытуемом веществе водорода, а помутнение известковой или баритовой воды — на присутствие углерода (образование осадка ВаСОз или СаСО )- [c.108]

    Вещество, имеющ-ее формулу СуНвО, при окислении целиком превратилось в омесь -веществ А (И Б. При -обработке одной из двух равных порций этой смеои избытком аммиачного раствора -окоида серебра образовалось 32,4 г осадка, а второй — избытком -водного раствора гидр-о-карб-оната натрия выдел(ился газ, при пропускании- которого в избыток баритовой -воды образовалось 9,85 г осадка. Каково -строение исходного вещества (и (вещ-еств А и Б) и сколько его было взято для оиисления  [c.47]










    Выполнение реакции. 5—6 капель исследуемого раствора помещают в одно из колен двухколенной пробирки (рис. 7) и прибавляют туда же несколько капель 2 н. раствора серной кислоты. В другое колено пробирки помещают 1 мл насыщенного водного раствора Ва(ОН)2 и быстро закрывают пробирку пробкой. В присутствии СОГ-ионов в пробирке, содержащей насыщенный раствор баритовой воды, образуется белый осадок. Аналогичную реакцию дают 50з — и ЗгОГ-ноны. Для устранения вредного влияния сульфитов и тиосульфатов их предварительно окисляют, прибавляя в колено с исследуемым раствором раствор окислителя (КМПО4 или КаСгаО,). [c.167]





Учебник общей химии (1981) — [

c.387



]

Приготовление растворов для химико-аналитических работ (1964) — [

c.29



]

Аналитическая химия (1994) — [

c.183



]

Курс аналитической химии (2004) — [

c.152



]

Курс неорганической химии (1963) — [

c.296



]

Рабочая книга по технической химии часть 2 (0) — [

c.40



]

Качественные микрохимические реакции по органической химии (1957) — [

c.17


,


c.30



]

Качественные микрохимические реакции по органической химии Издание 2 (1965) — [

c.22


,


c.231



]

Химические товары справочник часть 1 часть 2 издание 2 (1961) — [

c.107



]

Химические товары Справочник Часть 1,2 (1959) — [

c.107



]

Краткая химическая энциклопедия Том 1 (1961) — [

c.376



]

Качественный анализ (1951) — [

c.569



]

Качественный анализ 1960 (1960) — [

c.569



]

Курс качественного химического полумикроанализа 1962 (1962) — [

c.471



]

Общая химия 1982 (1982) — [

c.619



]

Общая химия 1986 (1986) — [

c.599



]

Учебник общей химии 1963 (0) — [

c.363



]

Неорганическая химия (1950) — [

c.271



]

Общая химия Издание 18 (1976) — [

c.611



]

Общая химия Издание 22 (1982) — [

c.619



]

Аналитическая химия (1980) — [

c.196



]

Курс аналитической химии (1964) — [

c.169


,


c.182



]

Неорганическая химия Том 1 (1971) — [

c.247



]

Курс аналитической химии Издание 2 (1968) — [

c.222



]

Курс аналитической химии Издание 4 (1977) — [

c.201



]

Краткая химическая энциклопедия Том 1 (1961) — [

c.376



]

Основы общей химии Том 2 (1967) — [

c.314



]

Основы общей химии Том 2 Издание 3 (1973) — [

c.160



]

Курс неорганической химии (1972) — [

c.264



]

Государственная фармакопея союза социалистических республик Издание 10 (1968) — [

c.873



]


План-конспектпрактической1 работы №1 «Качественный анализ органических соединений», химия 10 класс

План-конспект урока химии 10 класс.

Место урока: 10 класс Тема 2. Углеводороды и их природные источники

Тип урока: практическая работа

Цель работы: Образовательные

Закрепить теоретические навыки, полученные при изучении темы “Предельные углеводороды”. Уметь практически определять наличие углерода, водорода и хлора в органических веществах, проводить наблюдения, делать выводы; соблюдать правила техники безопасности при работе в химическом кабинете.

Развивающие

Развивать у учащихся умения сравнивать и анализировать сведения, полученные в ходе практической работы, логически излагать свои мысли.

Воспитывающие

Продолжить формирование химической картины мира, способствовать осознанию реальности существования атомов и молекул, и материального единства на основе этих представлений (показывая многообразие органических соединений).

Оборудование: Каждой группе выдается набор реактивов и лабораторных принадлежностей: лабораторный штатив, нагревательный прибор, простейший прибор для получения газов, держатель, фарфоровая чашка, парафин, оксид меди (II), известковая вода, прокаленный сульфат меди (II), медная проволочка (10 см), раствор тетрахлорметана, сахар, полимеры, содержащие х.э. хлор.

Ход урока

1.Организационный момент.

2.Актулизация знаний

Сегодня у нас необычный урок — практическая работа. С какими представителями предельных углеводородов вы сталкивались в своей жизни?

II. Постановка целей и задач урока . Мотивация учебной деятельности.

Какова тема практической работы?

Чем мы будем заниматься на уроке? Сформулируйте цель практической работы. (Практическим путём научиться распознавать основные группы катионов и анионов)

III. Формирование навыков на основе применения их в стандартных условиях.

Откроем учебники и посмотрим, какие опыты мы будем проводить ( изучение инструкции проведения опытов).

Определяя план действий.

Какая перед нами стоит задача?

Какие необходимы приборы и материалы для проведения опытов?

Какие правила техники безопасности мы должны соблюдать при выполнении работы?

IV. Формирование дифференцированных обобщенных умений.

Под руководством учителя формулируют тему и цель практической работы ( на основании инструкции) и записывают в тетради.

V. Анализ задачи.

Учащиеся получают таблицы для записей наблюдений:

При проведении практической работы мы должны заполнить таблицу

в виде

Проведение анализа

Отличительный

признак реакции

VI. Выполнение практической работы.

Опыт 1. Обнаружение углерода и водорода в органическом соединении

Выполнение работы:

Присутствие углерода в органических соединениях в большинстве случаев можно обнаружить по обугливанию вещества при осторожном его прокаливании.
Опыт 1. Обнаружение углерода и водорода в органическом соединении.
Условия выполнения работы:
Собрали прибор как показано на рис. 44 учебника. Насыпали в пробирку щепотку сахара и немного оксида меди (II) СuO. Положили в пробирку, где-то на уровне две трети её небольшой ватный тампон, потом насыпали немного безводного медного купороса CuSO4. Закрыли пробирку пробкой с газоотводной трубкой, так, чтобы нижний её конец был опущен в другую пробирку с предварительно налитым туда гидроксидом кальция Са(ОН)2. Нагрели пробирку в пламени горелки. Наблюдаем выделение пузырьков газа из трубки, помутнение известковой воды и посинение белого порошка CuSO4.
С12Н22О11 + 24CuO 12CO2 + 11H2O + 24Cu
Ca(OH)2 + CO2 CaCO3↓ + H2O
CuSO4 + 5H2O CuSO4 . 5H2O
Вывод: В исходном веществе присутствует углерод и водород, так как получили углекислый газ и воду в результате окисления, а в окислителе CuO они не содержались.

Опыт 2.
Наиболее точным методом открытия углерода и одновременно с ним водорода является сожжение органического вещества в смеси с мелким порошком оксида меди. Углерод образует с кислородом оксида меди(П) углекислый газ, а водород — воду. Оксид меди восстанавливается до металлической меди, например:

С13Н28 + 40СuО —> 13С02 + 14Н20 + 40Сu

Определение углерода и водорода в органическом соединении (парафин)

Соберите прибор, как показано на рисунке 44. Смесь 1—2 г оксида меди(II) и -0,2 г парафина хорошо перемешайте и поместите на дно пробирки. Сверху насыпьте еще немного оксида меди(II). В верхнюю часть пробирки введите в виде пробки небольшой кусочек ваты и насыпьте на нее тонкий слой белого порошка безводного сульфата меди(II). Закройте пробирку пробкой с газоотводной трубкой. При этом конец трубки должен почти упираться в комочек ваты с сульфатом меди(II). Нижний конец газоотводной трубки должен быть погружен в пробирку с баритовой водой (раствор гидроксида бария) или известковой водой (раствор гидроксида кальция). Нагрейте пробирку в пламени горелки. Если пробка плотно закрывает пробирку, то через несколько секунд из газоотводной трубки начнут выходить пузырьки газа.

Как только баритовая вода помутнеет, пробирку с ней следует удалить и продолжать нагревание, пока пары воды не достигнут белого порошка сульфата меди(ІІ) и не вызовут его посинения.

После изменения окраски сульфата меди(ІІ) следует прекратить нагревание.

1.    Почему помутнел раствор баритовой воды? Напишите уравнение реакции.

2.    Почему белый порошок сульфата меди(ІІ) стал голубым? Напишите уравнение реакции.

Опыт 3. Обнаружение галогенов
Условия выполнения работы:
Взяли медную проволоку, загнутую на конце петлёй щипцами, прокалили её в пламени до образования чёрного налёта оксида меди (II) СuO. Затем остывшую проволоку окунули в раствор хлороформа и вновь внесли её в пламя горелки. Наблюдаем окрашивание пламени в голубовато-зелёный цвет, так как соли меди окрашивают пламя.
5CuO + 2CHCl3 = 3CuCl2 + 2CO2 + H2O + 2Cu

VII. Самоконтроль выполнения работы.

Обучающиеся заполняют таблицу и делают выводы по каждому опыту

Качественный элементный анализ органических соединений

в виде

Проведение анализа

Отличительный

признак реакции

Хлор

Хлорида

меди (II)

Медную проволоку «смачивают»

исследуемым раствором и вносят в пламя горелки (проба Бейльштейна)

Зеленое пламя

Углерод

Диоксида

углерода

Покрывают вещество оксидом меди(II), нагревают, выделяющийся газ пропускают (недолго) через баритовую воду

Осадок белого цвета

Водород

Воды

Вещество покрывают слоем оксида меди(II), нагревают, помещают

каплю жидкости на бумагу, пропитанную хлоридом кобальта(II)

Изменение голубого цвета на светло-розовый

VIII. Итоги урока. Рефлексия.

Выводы: обнаружив образовавшиеся в результате реакции углекислый газ и воду, вы установили в исследованном веществе наличие углерода и водорода. Так как эти элементы не содержались в добавленном оксиде меди(ІІ), то они могли находиться только во взятом для анализа органическом веществе.

Галогены окрашивают пламени в голубовато-зелёный цвет

IX. Домашнее задание: закончить оформление практической работы в тетради

методические рекомендации к выполнению лпр | Методическая разработка по химии (10 класс):

ГОСУДАРСТВЕННОЕ БЮДЖЕТНОЕ ПРОФЕССИОНАЛЬНОЕ
ОБРАЗОВАТЕЛЬНОЕ УЧРЕЖДЕНИЕ
«Дзержинский технический колледж»

Рассмотрено                                                                                     Утверждаю

на заседании МК                                                                Зам. директора по СПО

математических и                                                                _______Р.Б.Силантьева

естественнонаучных                                                               «____»_______2018г                  

дисциплин

Председатель МК  Зайцева Е.А.

_______________________

«____»_______2018г

Перечень и содержание ЛПР

по дисциплине  « ХИМИЯ»

для обучающихся 1 курса

 в 2018 -2019 учебном году

Преподаватель:                                                                  Зайцева Е.А.

Критерии и нормы оценки знаний и умений обучающихся за практические и лабораторные работы.

 Оценка   «5» ставится, если обучающийся :

 1.  Правильной самостоятельно  определяет цель данных работ; выполняет работу в полном объёме с соблюдением необходимой  ‘ последовательности проведения опытов, измерений.

 2.  Самостоятельно, рационально выбирает и готовит для выполнения работ необходимое оборудование; проводит данные работы в условиях, обеспечивающих получение наиболее точных результатов.

 3.  Грамотно, логично описывает ход практических (лабораторных) работ, правильно формулирует выводы; точно и аккуратно выполняет все записи, таблицы, рисунки, чертежи, графики, вычисления.

 4.  Проявляет организационно-трудовые умения: поддерживает чистоту рабочего места, порядок на столе, экономно расходует материалы; соблюдает правила техники безопасности при выполнении работ.

Оценка   «4» ставится, если обучающийся:

 1.  Выполняет практическую (лабораторную) работу полностью в соответствии с требованиями при оценивании результатов на «5», но допускает в вычислениях, измерениях два — три недочёта или одну негрубую ошибку и один недочёт.

 2.  При оформлении работ допускает неточности в описании хода действий; делает неполные выводы при обобщении.

Оценка   «3» ставится, если обучающийся:

 1.1 Правильно выполняет работу не менее, чем на 50%, однако объём выполненной части таков, что позволяет получить верные результаты и сделать выводы по основным, принципиальным важным задачам работы.

 2.  Подбирает оборудование, материал, начинает работу с помощью преподавателя; или в ходе проведения измерений, вычислений, наблюдений допускает ошибки, неточно формулирует выводы, обобщения.

 3.  Проводит работу в нерациональных условиях, что приводит к получению результатов с большими погрешностями; или в отчёте допускает в общей сложности не более двух ошибок (в записях чисел, результатов измерений, вычислений, составлении графиков, таблиц, схем и т.д.), не имеющих для данной работы принципиального значения, но повлиявших на результат выполнения.

 4.  Допускает грубую ошибку в ходе выполнения работы: в объяснении, в оформлении, в соблюдении правил техники безопасности, которую обучающийся  исправляет по требованию преподавателя.

Оценка   «2» ставится, если обучающийся:

 1.  Не определяет самостоятельно цель работы, не может без помощи преподавателя подготовить соответствующее оборудование; выполняет работу не полностью, и объём выполненной части не позволяет сделать правильные выводы.

 2.  Допускает две и более грубые ошибки в ходе работ, которые не может исправить по требованию преподавателя; или производит измерения, вычисления, наблюдения неверно.

Оценка    «1» ставится в случае:

 1.      Нет ответа.

Практическая работа № 1

Тема: Приготовление раствора с определенной массовой долей растворенного вещества.

Цель: 1. научиться использовать весы для определения массы вещества,        

2. отмерять требуемый объем жидкости,

3.готовить  растворы определенной концентрации.

4. научиться использовать понятие «массовая доля растворенного вещества» для расчетов.

Оборудование: мерный цилиндр, рычажные весы и набор разновесов, колба, стеклянная трубочка, вода, поваренная соль.

порядок  работы:

  1. Отвесьте на весах навеску поваренной соли и поместите в колбу.
  2. Отмерьте требуемый объем дистиллированной воды и вылейте ее в колбу с солью.
  3. Содержимое колбы перемешайте до полного растворения соли.
  4. Рассчитайте число молекул поваренной соли в полученном растворе.

1 вариант

Номер группы

Номер задачи

Масса раствора, г

Масса растворенного вещества, г

Масса соды в растворе, г

Массовая доля растворенного вещества, %

1

1

2

3

?

400

500

40

?

?

160

?

300

?

50

?

2

1

2

3

?

50

400

30

?

?

270

?

320

?

10

?

3

1

2

3

?

200

50

50

?

?

150

?

45

?

5

?

4

1

2

3

?

300

200

20

?

?

380

?

180

?

30

?

Решить задачу:

  1. Для подкормки растений нужно приготовить 2% раствор аммиачной селитры массой 400г Сколько грамм йода и спирта надо взять для приготовления 500г 5% йодистой настойки?
  2. .  Как это сделать?
  3. При засолке огурцов на 1л воды берут соль массой 90г. Какова массовая доля соли в этом растворе?

Практическая работа № 1

Тема: Приготовление раствора с определенной массовой долей растворенного вещества.

Цель: научить использовать весы для определения массы вещества, отмерять требуемый объем жидкости, приготовлять растворы определенной концентрации. Уметь использовать понятие «массовая доля растворенного вещества» для расчетов.

Оборудование: мерный цилиндр, рычажные весы и набор разновесов, колба, стеклянная трубочка, вода, поваренная соль.

Ход работы:

  1. Отвесте на весах навеску поваренной соли и поместите в колбу.
  2. Отмерьте требуемый объем дистиллированной воды и вылейте ее в колбу с солью.
  3. Содержимое колбы перемешайте до полного растворения соли.
  4. Рассчитайте число молекул поваренной соли в полученном растворе.

2 вариант

Номер группы

Номер задачи

Масса раствора, г

Масса растворенного вещества, г

Масса соды в растворе, г

Массовая доля растворенного вещества, %

1

1

2

3

200

?

?

?

200

200

?

200

?

20

?

40

2

1

2

3

300

?

?

?

5

80

?

45

?

10

?

20

3

1

2

3

200

?

?

?

10

5

?

190

?

25

?

10

4

1

2

3

400

?

?

?

90

20

?

210

?

5

?

10

Решить задачу:

  1. Для подкормки растений нужно приготовить 2% раствор аммиачной селитры массой 400г. Как это сделать?
  2. При засолке огурцов на 1л воды берут соль массой 90г. Какова массовая доля соли в этом растворе?
  3. Сколько грамм йода и спирта надо взять для приготовления 500г 5% йодистой настойки?

Практическая работа № 2

Реакции ионного обмена. Гидролиз солей

Цель: научиться решать задачи на гидролиз.

Оборудование:

Порядок  работы

Опыт № 1: Реакции с образованием осадков.

1. В три пробирки налить по 2  мл растворов: в одну – сульфата натрия, в другую – сульфата цинка, в третью – сульфата аммония. В каждую из пробирок прибавить по такому же количеству раствора хлорида бария. Что наблюдаете? Какой цвет  имеет осадок?

        Написать молекулярные уравнения реакции и краткое ионное уравнение.  Каков цвета осадка?

2. В две пробирки налить по 2-3 мл растворов: в одну – сульфата меди (II), в другую – хлорида алюминия. В каждую из пробирок прибавить понемногу раствора щелочи. Что наблюдаете? Какой цвет  имеет осадок?

        Написать молекулярные и ионные уравнения реакции.

Внимание! Пробирки с осадками оставить для опыта 2!

Опыт № 2: Реакции с образованием малодиссоциирующих веществ.

1. Налить в пробирку 2-3 мл раствора щелочи. Прибавить 2-3 капли фенолфталеина. Как изменился цвет раствора? К полученному раствору добавить соляную кислоту до исчезновения окраски.

        Написать молекулярное и ионное уравнение реакции.

2. В пробирки с полученными осадками в опыте 1.2. добавить раствор серной кислоты до исчезновения осадков.

Написать молекулярное и ионное уравнение реакции.

Опыт № 3: Реакции с образованием газов.

1. В небольшом количестве воды взболтать щепотку растертого в порошок мела и прилить немного соляной кислоты. Какой газ выделяется при этом?

Написать молекулярное и ионное уравнение реакции.

2. В пробирку внести немного кристаллического карбоната натрия и добавить соляной кислоты. Какой газ выделяется при этом?

Написать молекулярное и ионное уравнение реакции.

Задание: сделать общий вывод, отметив, в каких случаях реакции ионного обмена «идут до конца».

Опыт № 4: Определение рН растворов.

1. Опустить в раствор уксусной кислоты полоску универсальной индикаторной бумаги. Сравнить цвет бумаги со шкалой рН.

2. Опустить в раствор аммиака полоску универсальной индикаторной бумаги. Сравнить окраску бумаги со шкалой рН.

Опыт № 5: Гидролиз солей.

В четыре пробирки налить по 2-3 мл: в первую –дистиллированной воды, во вторую – раствора карбоната натрия, в третью – раствора сульфата алюминия, в четвертую – раствор нитрата калия.

        Определить реакцию среды. Составить уравнения.

Опыт № 6: Реакции  ионного обмена.

1. Налить в две пробирки по 1-2 мл раствора желтой кровяной соли – K4[Fe(CN)6] и в одну из них добавить несколько капель раствора сульфата железа FeSO4 (+2), а в другую – хлорида железа FeCl3(+3).

        Наблюдать образование синего осадка берлинской лазури в пробирке, содержащей ионы Fe3+.

        Написать молекулярное и ионное уравнения реакции. Назвать комплексные соединения.

2. Опыт повторить, только вместо желтой кровяной соли взять красную кровяную соль – K3[Fe(CN)6].

        Наблюдать образование осадка турнбулевой сини в пробирке содержащей ионы Fe2+.

Лабораторная работа по химии № 3
Получение газов.

Цель работы. Получение газообразных неорганических веществ, их идентификация  с помощью качественных реакций, а также изучение некоторых свойств.

Оборудование:

Порядок работы. 1. Получение водорода. В пробирку поместите две гранулы цинка и прилейте 1-2 мл соляной кислоты. (Что наблюдаете? Напишите Уравнение реакции)

        2.Получение кислорода. Накройте пробирку с цинком пробиркой большего диаметра. Через 1-2 мин поднимите большую пробирку вверх и, не переворачивая ее, закрыв пальцем, поднесите к пламени спиртовки. (Что наблюдаете? Что можно сказать о чистоте собранного вами водорода? Почему водород собирали в перевернутую вверх дном пробирку?)

        3.Получение  кислорода. В пробирку объемом 20 мл прилейте 5—7 мл раствора пероксида водорода. Подготовьте тлеющую лучинку (подожгите ее и, когда она загорится, взмахами руки погасите). Поднесите к пробирке с пероксидом водорода, куда предварительно насыпьте немного (на кончике шпателя) оксида марганца (IV). (Что наблюдаете? Запишите уравнение реакции.)

        4. Получение аммиака. В пробирку прилейте 1-2 мл раствора хлорида аммония, а затем такой же объем раствора щелочи. Закрепите пробирку в держателе и осторожно нагрейте на пламени горелки. (Что наблюдайте? запишите уравнение реакции) Поднесите к отверстию пробирки влажную красную лакмусовую бумажку. (Что наблюдайте? Осторожно понюхайте выделяющийся газ. Что ощущаете?)

«Решение экспериментальных задач по теме «ТЭД»

Цель: закрепить знания о качественных реакциях на ионы, способах распознавания веществ и генетической связи, умения определять качественный состав и получать заданные вещества.

ТБ: соблюдать осторожность при работе с кислотами и щелочами, растворами солей меди, бария, серебра.

Ход работы

Цель опыта

Ход опыта

Наблюдения. Выводы. Уравнения реакций в молекулярном и ионном виде

      1. Опытным путем определите растворы сульфата натрия, хлорида натрия и серной кислоты.

Проба 1.

индикатор

1      2     3

Проба 2

BaCl2

        1         2

                                  Реактив

№ пробирки

Индикатор

BaCl2

Вывод

1

↓белый (б)

Вещество№1-

2

Вещество№2-

3

Изменение окраски (а)

↓белый (в)

Вещество№3-

Уравнения:

а) h3SO4 ═ 2 H+ + SO42-

б) Na2SO4+ BaCl2= ВаSO4↓+2NaСl

       Ba2+ + SO42- = BaSO4↓

в) h3SO4+ BaCl2= ВаSO4↓+2НСl

       Ba2+ + SO42- = BaSO4↓

2.Подтвердите качественный состав

а) соляной  кислоты

б) хлорида алюминия

в) сульфата меди (II)

    Лакмус              AgNO3

 1 (H+)    2(Cl-)

                 HCl

     

    NaOH                   AgNO3

 1(Al3+)         2(Cl-)

                    AlCl3

               ?                             ?

             1              2

                    CuSO4

 а) 1. Лакмус красный

        HCl ═  H+ + Cl-

   

    2. Белый осадок

  HCl + AgNO3 = AgCl↓+ НNO3

       Ag+ + Cl- = AgCl ↓

Вывод: исследуемое вещество…

  б) 1.

   

     

      2.

Вывод: исследуемое вещество…

  в) 1.

     

       2.

Вывод: исследуемое вещество…

3. Используя следующие вещества: гидроксид натрия, оксид углерода (VI), серную кислоту, хлорид бария и нитрат серебра, осуществите следующие превращения:

NaOH→Na2CO3

  Na2SO4 → NaCl

 NaNO3

1) NaOH +… → Na2CO3+…  

2) Na2CO3 +… →  Na2SO4 +…+…

 3)  Na2SO4 +…→ NaCl +…

                   

4) NaCl +…→ NaNO3 +…

Для реакции  составить уравнения в молекулярном и ионном виде.

Практическая работа №5

Качественный анализ органических соединений. Обнаружение углерода и водорода.

Цель: опытным путем определить наличие водорода и углерода в органических соединениях.

Оборудование: оксид меди(II), парафин, безводный сульфат меди(II).

Порядок работы.

Присутствие углерода в органических соединениях в большинстве случаев можно обнаружить по обугливанию вещества при осторожном его прокаливании.

Определение углерода и водорода в органическом соединении (Парафин)

Смесь 1-2г оксида меди(II) и 2 грамма парафина хорошо перемешайте и поместите на дно пробирки. Сверху насыпьте ещё немного оксида меди(II). В верхнюю часть пробирки введите в виде пробки небольшой кусочек ваты и насыпьте на неё тонкий слой белого порошка безводного сульфата меди(II). Закройте пробирку пробкой с газоотводной трубкой. При этом конец трубки должен почти упираться в комочек ваты с сульфатом меди(II). Нижний конец газоотводной трубки должен быть погружен в пробирку с  известковой водой (раствор гидроксида кальция). Нагрейте пробирку в пламени горелки. Если  пробка плотно закрывает пробирку, то через несколько секунд из газоотводной трубки начнут выходить пузырьки газа. Как только известковая вода помутнеет, пробирку с ней следует удалить и продолжать нагревать, пока пары воды не достигнут белого порошка сульфата меди(II) и не вызовут его посинения.  После изменения окраски сульфата меди(II) следует прекратить нагревание.

  1. Почему помутнел раствор баритовой воды? Напишите уравнение реакции.
  2. Почему белый порошок сульфата меди(II) стал голубым? Напишите уравнение реакции.

      Выводы: обнаружив образовавшиеся в результате реакции углекислый газ и воду, вы установили в исследованном веществе наличие водорода и углерода. Так как эти элементы не содержались в добавленном оксиде меди(II), то они могли находиться только во взятом для анализа органическом веществе.

Обнаружение галогенов  (проба Бейльштейна)

Галогены можно обнаружить при помощи реакции окрашивания пламени, продолженную русским химиком Ф.Ф. Бейльштейном.

Для проведения опыта требуется медная проволока длиной около 10 см, загнутая на конце петлёй и вставленная другим концом в небольшую пробку. Держа за пробирку, прокалите  петлю проволоки до исчезновения посторонней окраски пламени. Остывшую петлю, покрывшуюся чёрным налётом оксида меди(II), опустите в пробирку с хлороформом,  затем смоченную веществом петлю вновь внесите в пламя грелки. Немедленно появляется характерная зеленовато-голубая окраска пламени, так как образующиеся при сгорании летучие галогениды меди окрашивают пламя горелки.

Практическая работа № 6

Спирты

Цель: опытным путем изучить свойства спиртов (этиловый, изоамиловый), определить их различие.

Оборудование: этиловый, изоамиловый спирты, дихромат калия, раствор серной кислоты

Порядок работы.

Растворимость спиртов в воде.

В отдельные пробирки прилейте по 1-2мл этилового и изоамилового спиртов. Добавьте к ним по 2-3 мл воды и взболтайте. Отметьте, что этиловый спирт полностью растворяется в воде, а изоамиловый спирт отделяется при отстаивании в виде маслянистого слоя над водой.

  1. В чём причина различного «поведения» спиртов воде?
  2. Почему изоамиловый  спирт  отслаивается над водой, а не наоборот?
  3. Какие органические жидкие вещества при смешивании с водой будут отслаиваться над водой?

Окисление этилового спирта хромовой смесью.

В пробирке смешайте 2 мл 5%-ного раствора дихромата калия, 1мл 20%-ного раствора серной кислоты и 0,5 мл этилового спирта. Отметьте цвет раствора. Осторожно нагрейте смесь на пламени горелки до начала изменения цвета. При этом ощущается характерный запах уксусного альдегида, образующегося в результате реакции.

  1. Почему цвет раствора меняется с оранжевого до синевато- зелёного? Напишите уравнение реакции  окисления этилового спирта.
  2.  Можно ли заменить серную кислоту в данной реакции на соляную?

Практическая работа № 6

Углеводороды

Цель: научиться получать углеводороды и изучить их химические свойства.

Получение и свойства этена (этилена)

В пробирку поместите 2 мл концентрирова6ной  серной кислоты, 1мл этилового спирта (лучше, если используется смесь приготовленная учителем заранее) и несколько крупинок оксида алюминия (Al2O3) или маленький кусочек пемзы для равномерного кипения смеси при нагревании. Закройте пробирку пробкой м газоотводной трубкой и нагрейте пробирку в пламени горелки. Выделяющийся газ пропустите в отдельные пробирки с бромной водой и раствором перманганата калия. Убедитесь в том, что бромная вода и раствор перманганата калия быстро обесцвечиваются. Подожгите газ у конца газоотводной трубки. Отметьте цвет пламени. (Этен горит  светящимся пламенем.)

  1. Чем отличаются реакции горения этена и этана?
  2. Как получают этен и лаборатории и промышленности? Напишите уравнение реакции.
  3. Почему этен обесцвечивает растворы бромной воды и перманганата калия? Напишите уравнения соответствующих реакций.

Свойства Бензола

В две пробирки налейте по 5-6 капель бензола. В одну из них добавьте 1-2 мл бромной воды, а в другую- 1-2 мл раствора перманганата калия. Встряхните обе пробирки. Отметьте наблюдения.

  1. Почему растворы бромной воды и перманганата калия не обесцвечиваются при добавлении бензола?
  2. Почему в пробирке с бромной водой при добавлении бензола жёлтая окраска перешла в верхний бензольный слой, а в пробирке с перманганатом калия окрашенный остался нижний водный слой?

Практическая работа № 7

Амины. Аминокислоты. Белки.

Цель: научить определять белки, используя данные реактивы, учитывая их химические свойства.

Оборудование: анилин, раствор щелочи, бромная вода

Порядок работы.

Образование солей анилина

В пробирку прилейте 0,5 мл анилина и 3 мл воды. Взболтайте. Что наблюдается? В пробирку добавьте соляной кислоты до полного растворения анилина в воде. К раствору добавьте 1 – 2 мл раствора щелочи. Что наблюдаете?

Бромирование анилина

В пробирку налейте 0,5 мл анилина и 0,5 мл воды. Прибавьте по каплям бромной воды до появления осадка.

  1. Почему обесцвечивается бромная вода?
  2. Каково строение образующегося осадка? Напишите уравнение реакции.

Получение медной соли глицина

В пробирку, содержащую 2 мл раствора глицина, добавьте 1 г порошка оксида меди (II) и нагрейте до кипения.

      1.  Чем обусловлено появление голубой окраски раствора?

  1. Каково строение образующейся соли?

Денатурация белка

 Приготовьте раствор белка. Для этого белок куриного яйца растворите в 150 мл воды. В пробирку налейте 4 – 5 мл раствора белка и нагрейте на горелке до кипения. Отметьте помутнение раствора. Охладите содержимое пробирки. Разбавьте водой в 2 раза.

  1. Почему раствор белка при нагревании мутнеет?
  2. Почему образующийся при нагревании осадок не растворяется при охлаждении и разбавлении водой?

Осаждение белка солями тяжелых металлов

В две пробирки налейте по 1 – 2 мл раствора белка и медленно, при встряхивании, по каплям добавьте в одну пробирку насыщенный раствор медного купороса, а в другую – раствор ацетата свинца. Отметьте образование труднорастворимых солеобразных соединений белка. Данный опыт иллюстрирует применение белка как противоядия при отравлении солями тяжелых металлов.

Цветные реакции белков

Ксантопротеиновая реакция.  В пробирку налейте 2 – 3 мл раствора белка и прибавьте несколько капель концентрированной азотной кислоты. Нагрейте содержимое пробирки, при этом образуется желтый осадок. Охладите смесь и добавьте аммиак до щелочной реакции ( проба на лакмус ). Окраска переходит в оранжевую.

Биуретовая реакция. В пробирку налейте 2 – 3 мл раствора белка и 2 – 3 мл раствора гидроксида натрия, затем  1 – 2 мл раствора медного купороса. Появляется фиолетовое окрашивание.

Лабораторная работа по химии № 8

Изучение свойств пластмасс и волокон

Цель работы. Идентификация образцов пластмасс и волокон на основании их отношения к нагреванию и характера горения.

Оборудование:

Порядок работы.

Порядок выполнения. При изучении свойств пластмасс прежде всего следует уделить внимание внешнему виду, твердости, эластичности.
Однако окончательный вывод можно сделать, лишь изучив отношение образца к нагреванию, характер горения и природу продуктов разложения.
        Определить природу волокна по внешнему виду сложно. Одним из самых доступных способов является изучение характера горения, анализ запаха продуктов разложения и остатка после сгорания.
        Изучение характера горения материала и продуктов его сгорания следует проводить таким образом.

1. Возьмите щипцами образец пластмассы,  волокна или ткани и внесите его в верхнюю часть пламени спиртовки. Обратите внимание, плавится ли образец, как быстро он загорается.
2. После того как вещество загорелось, выньте его из пламени. Гаснет пламя или продолжает гореть?
3. К выделяющимся продуктам сгорания поднесите влажную лакмусовую бумажку, отметьте изменение цвета.
4. Движением руки направьте к носу газообразные продукты сгорания и попробуйте определить их запах.
5. Дождитесь, когда твердый остаток горения на керамической или стеклянной пластине полностью остынет. Рассмотрите его внешний вид, цвет.
6. Попробуйте растереть золу или спекшийся шарик между пальцами.
Изучая свойства пластмасс и волокон, воспользуйтесь данными таблицы, приведенной ниже:

Название пластмассы или волокна

Физические свойства

Отношение к нагреванию

Характер и продукты горения

Свойства пластмасс

Полиэтилен

Неокрашенный материал, полупрозрачен, молочного оттенка, эластичный, жирный на ощупь

Плавится; из расплавленного материала можно вытянуть нити

Горит синеватым пламенем с запахом горящей свечи. Продолжает гореть вне пламени, при этом с образца подают горящие капли (осторожно!)

Поливинил-хлорид

Эластичный материал, механически прочен, может иметь различную окраску

Плавится и начинает разлагаться

Горит коптящим пламенем; вне пламени гаснет. Выделяется хлороводород, который можно обнаружить по покраснению лакмусовой бумажки

Целлулоид

Твердый, гибкий материал, может иметь различную окраску

Не плавится

При внесении в пламя сразу загорается и быстро сгорает даже вне пламени. После сгорания остается серая зола

Свойства волокон

Хлопок

Быстро сгорает. При горении запах жженой бумаги. Остается серая зола

Шерсть, шелк

Горит медленно, образуя черный шарик, легко растирающийся в порошок. Запах «жженого рога»

Капрон

Плавится с образованием темного блестящего

Загорается при сильном нагревании с неприятным запахом. Продукты горения окрашивают лакмусовую бумажку в синий цвет

Лавсан

То же

Горит коптящим пламенем. Продукты горения имеют запах

Ацетатное волокно

Быстро сгорает, образуя нехрупкий темный шарик. Вне пламени постепенно гаснет.

Практическая работа №9

Карбоновые кислоты

Оборудование: уксусная, бензойная кислоты, гранулы цинка, раствор гидроксида натрия, концентрированная серная кислота, спиртовка

Порядок работы.

Растворимость карбоновых кислот в воде.

В одну пробирку прилейте 1-2 мл воды и добавьте 2-3 капли уксусной кислоты, взболтайте содержимое. В другую пробирку также прилейте 1-2 сл воды и добавьте 0,1-0,2 г бензойной кислоты, взболтайте содержимое. Отметьте различие в «поведении» уксусной и бензойной кислот. Пробирку с бензойной кислотой нагрейте на пламени газовой горелки до растворения. Охладите пробирку.  Добавьте к выпавшему осадку немного раствора гидроксида натрия.  

  1. В чём причина различной растворимости органических кислот в воде?
  2. Что наблюдается при добавлении гидроксида натрия к бензойной кислоте? Напишите уравнение реакции.

Взаимодействие уксусной кислоты с металлами

В пробирку прилейте 1-2 мл уксусной кислоты и добавьте 1-2 г гранул цинка. Если не наблюдается никаких изменений, пробирку слегка нагрейте на пламени горелки.

1.Что наблюдается при нагревании? Какой газ выделяется? Напишите уравнение реакции.

2.Какие металлы будут реагировать с  уксусной кислотой? А какие нет? Сделайте общий вывод об условиях протекания реакций с металлами как органических, так и неорганических кислот.

                  Получение сложного эфира.

В пробирку налейте 2мл изоамилового (изопентилового) спирта, 2мл уксусной кислоты и 0,5 мл концентрированной серной кислоты. Закройте пробирку газоотводной трубкой и нагрейте на водяной бане в течении нескольких минут. После охлаждения добавьте в пробирку несколько миллилитров воды. При  этом выделяется слой изоамилового эфира уксусной кислоты (изоамилацетата) с характерным запахом грушевой эссенции.

  1. Как называется реакция взаимодействия кислот со спиртами? Напишите уравнение реакции этилового спирта и изоамилового спирта с уксусной кислотой.
  2. Для чего в реакционную смесь, содержащую спирт и карбоновую кислоту, добавляют концентрированную серную кислоту?

Практическая работа  №10

Углеводы

цель работы

Оборудование: раствор глюкозы, сахароза,  аммиачный раствор оксида серебра, раствор гидроксида натрия, раствор сульфата меди, спиртовка

Порядок работы.

Действие аммиачного раствора

оксида серебра на глюкозу.

В пробирку, содержащую1-2 мл раствора глюкозы в воде, прилейте 1-2 мл аммиачного раствора оксида серебра и нагрейте пробирку на кипящей водяной бане.

Что наблюдается? Какая форма глюкозы (открытая или циклическая) даёт реакцию «серебряного зеркала»? Напишите уравнение реакции.

Действие гидроаксида меди(II)

На глюкозу

а) В пробирку прилейте 0.5мл раствора глюкозы и 2 мл раствора гидроаксида натрия. К полученной смеси добавьте 1мл раствора медного купороса.  Что наблюдаете?

б) К полученному раствору  аккуратно добавьте 1 мл воды и нагрейте на пламени горелки пробирку, укрепив её наклонно так, чтобы нагревалась только верхняя часть раствора. Прекратите нагревать, как только начнётся изменение цвета.

1. Почему образовавшийся вначале осадок гидроксида меди(II)  растворяется с образованием прозрачного синего раствора? Наличием каких функциональных групп в глюкозе обусловлена эта реакция? Напишите уравнение реакции.

2. Почему при нагревании происходит изменение цвета реакционной смеси с синего на оранжево-жёлтый цвет? Что представляет собой жёлто-красный осадок? Наличие, какой функциональной группы в глюкозе является причиной данной реакции? Напишите уравнение реакции.

Действие аммиачного раствора

оксида серебра на сахарозу.

а) В пробирку, содержащую 1-2сл раствора сахарозы в воде, прилейте 1-2 мл аммиачного раствора оксида серебра и нагрейте  пробирку на кипящей водяной бане. Что наблюдается?

б) В пробирку, содержащую 1-2мл раствора сахарозы в воде, добавьте несколько капель разбавленной серной кислоты и нагрейте на кипящей водяной бане в течении 5-10 мин. Затем охлаждённый раствор доведите до слабощелочной реакции (проба на лакмус), добавив в пробирку раствор щёлочи. К полученному раствору добавьте 1-2 мл аммиачного раствора оксида серебра и нагрейте пробирку на кипящей водяной бане.

1. Почему сахароза не даёт реакцию «серебряного зеркала»?

2. Какие процессы происходит с сахарозой при нагревании её расвтора с кислотой? Почему после нагревания проба с аммиачным раствором оксида серебра даёт положительный результат? Напишите уравнение реакции гидролиза сахарозы.  

Лабораторная работа по химии №11

Решение экспериментальных задач.

        Цель работы. идентификация неорганических веществ в растворах с помощью качественных реакций или путем выявления.

Оборудование: растворы хлорида натрия, карбоната натрия, сульфата натрия, ацетата натрия, хлорида аммония, хлорида бария, хлорида алюминия

Порядок работы.

        Порядок работы. 1. С помощью качественных реакций определите, в какой из выданных вам пробирок находятся раствором хлорида натрия, карбоната натрия, сульфата натрия, ацетата натрия. (Напишите уравнения реакций в молекулярной и ионной форах.)

        2. С помощью качественных реакций определите, в какой из выданных вам пробирок находятся растворы хлорида аммония, хлорида бария, хлорида алюминия.(Напишите уравнения реакций в молекулярной и ионной формах)

        3. C помощью индикаторной бумаги определите, в какой из выданных вам пробирок находятся растворы солей: карбонат натрия, нитрат аммония, сульфат калия. (Напишите уравнения реакций гидролиза в молекулярной и ионной формах.)

        4. Проведите химические реакции, позволяющие  осуществить следующие превращения:

медь — оксид меди(2) — сульфат меди(2) — гидроксид меди(2) — оксид меди(2)

        5. Опытным путем подтвердите качественный состав хлорида аммония.

        6.Получите гидроксид меди (2) реакцией обмена и осуществите реакции, подтверждающие его свойства.

Практическая работа по химии № 12

Идентификация органических соединений

Цель работы. Распознавание органических веществ с помощью качественных реакций.
Порядок работы. 1. В двух пробирках без этикеток содержатся следующие пары веществ:

а) растворы этилового спирта и муравьиной кислоты;
б) растворы этилового спирта и уксусной кислоты;
в) растворы формальдегида и глицерина
г) растворы глюкозы и глицерина;
д) растворы формальдегида и белка;
е) растительное и машинное масла;
ж) крахмальный клейстер и глицерин;
з) растворы глюкозы и этанола;
и) растворы сахарозы и глюкозы.
Предложите способ экспериментального определения содержимого каждой пробирки.

2. С помощью одного реактива докажите что глюкоза является веществом с двойственной функцией.

3. Вам выданы пробирки с растворами, в одной из которых содержится раствор глицерина, в другой – раствор формальдегида, в третьей –раствор глюкозы.С помощью одних и тех же реактивов определите каждое вещество.

4. Докажите опытным путем, что картофель, белый хлеб, пшеничная мука содержат крахмал.

Задача по химии — 576

2019-10-21   
Продукты электролиза водного раствора натриевой соли некоторой одноосновной карбоновой кислоты пропустили через промывные склянки с баритовой водой и раствором едкого натра. Газ, выделявшийся на аноде, вызывал помутнение известковой воды; газ, полученный на катоде, проходил через склянки, не меняя своего объема. После окончания электролиза объемы газов, собранных в цилиндрах, оказались одинаковыми. После сгорания газов на стенках сосудов появились капельки воды, а баритовая вода, добавленная к продуктам сгорания газов (после их очистки), помутнела только в случае газа, выделившегося на аноде.
а. Из каких элементов состоял газ, собранный после очистки газа, который образовался на аноде?
б. Какова формула кислоты, если плотность полученного газа в 1,037 раза больше плотности воздуха при тех же условиях, а масса углерода, вводящего в его состав, составляет 80 % от массы газа?

Решение:

В газе, выделяющемся на аноде, содержится двуокись углерода, которая поглощается растворами $Ba(OH)_2$ и $NaOH$. Кроме этого в нем содержится углеводород, при сгорании которого образуются вода и двуокись углерода, они и вызывают помутнение баритовой воды. На катоде выделяется водород.

Молекулярная масса углеводорода, полученного при анодном окислении анионов соли карбоновой кислоты, равна

$M = 29 \cdot 1,037 = 30,07 \approx 30$.

Известно, что содержание углерода в моле углеводорода составляет 80 %, т. е. $M_с = 30 \cdot 0,8 = 24$. Следовательно, в состав молекулы углеводорода входит 2 атома углерода и $30 — 24 = 6$ атомов водорода; его формула $C_2H_6$ — это этан.{-} — 2e \rightarrow R — R + 2CO_2$.

Анион кислоты содержит радикал $CH_3$ таким образом, для электролиза был взят ацетат натрия $CH_3COONa$, т. e. соль уксусной кислоты.

Задание 32 на ЕГЭ по химии

Задание 32 на ЕГЭ по химии (бывшее задание С2 «нового типа») содержит описание эксперимента, состоящего из последовательно проводимых химических реакций и лабораторных методов разделения продуктов реакций (мысленный эксперимент).

По моим наблюдениям, у многих учеников это задание вызывает сложности. В немалой степени это объясняется все более академическим характером преподавания химии в школах и на курсах, когда изучению особенностей работы в лаборатории и собственно проведению лабораторных экспериментов выделяется недостаточное количество внимания.

Поэтому я решил систематизировать и обобщить материал по т.н.  «лабораторной» химии. В этой статье рассматриваются примеры задания 32 в ЕГЭ по химии-2018 (бывшего задания С2), с подробным разбором и анализом решения.

Для выполнения этого задания необходимо хорошее понимание некоторых тем общей химии и химии элементов, а именно: основные классы неорганических веществ, химические свойства и получение оксидов, кислот, оснований и солей, и взаимосвязь между различными классами неорганических веществ; свойства простых веществ — металлов и неметаллов; гидролиз; электролиз; окислительно-восстановительные реакции (ОВР), основные окислители и восстановители, и их превращения в разных условиях, основные типы ОВР, химия щелочных металлов и их соединений, щелочноземельных металлов и соединений, углерода, кремния, азота, фосфора, серы, галогенов.

  1. Раствор, полученный при взаимодействии меди с концентри­рованной азотной кислотой, выпарили и осадок прокалили. Газооб­разные продукты реакции разложения полностью поглощены водой, а над твердым остатком пропустили водород. Напишите уравнения описанных реакций.

Анализ и решение.

«Ключевые слова» — концентрированная азотная кислота и медь.

Медь — малоактивный металл, проявляет свойства восстанови­теля.

Окислительные свойства азотной кислоты определяет азот в степени окисления +5, поэтому при взаимодействии как с метал­лами, так и с другими восстановителями выделяется не водород, а ве­щество, содержащее азот в более низких степенях окисления; кон­центрированная азотная кислота растворяет малоактивные металлы и восстанавливается до NO2 (уравнение 1).

«Ключевые слова» — … осадок прокалили. Нитраты металлов, на­ходящихся в ряду активности от магния до меди, разлагаются при нагревании на оксид металла, бурый газ и кислород (уравнение 2).

«Ключевые слова» — … пропустили водород. Восстановление ок­сидов металлов водородом — один из способов получения металлов (уравнение 3).

«Ключевые слова» — Газообразные продукты… поглощены во­дой…. При разложении нитрата меди выделяются NO2 и O2. Оксиду азота (IV) соответствуют азотистая и азотная кислоты, однако в присутствии  окислителя — молекулярного кислорода в растворе образуется только азотная кислота (уравнение 4).

1) Сu + 4HNO3(конц) = Cu(NO3)2 + 2NO2↑ +2Н2O

2)  2Cu(NO3)2 = 2СuО + 4NO2↑ + O2

3)  СuО + Н2 = Сu + Н2O

4)  4NO2 + O2+ 2Н2O = 4HNO3

  1. Простое вещество, полученное при нагревании фосфата каль­ция с коксом и оксидом кремния, сплавили с металлическим кальци­ем. Продукт реакции обработали водой, а выделившийся газ собрали и пропустили через раствор соляной кислоты. Напишите уравнения описанных реакций.

Анализ и решение

«Ключевые слова» — … нагревании фосфата кальция с коксом и оксидом кремния … . Первое из описанных превращений — про­мышленный способ получения фосфора (уравнение 1).

«Ключевые слова» — Простое вещество … сплавили с кальцием.

Простое вещество, образовавшееся в первом превращении, — фос­фор. Металлы (восстановители) взаимодействуют с неметаллами (окислителями) с образованием бинарных соединений (солей или оксидов) (уравнение 2).

«Ключевые слова» — Продукт … обработали водой …. При взаи­модействии фосфида Са3Р2 с водой происходит гидролиз (уравне­ние 3) и образуются гидроксид кальция Са(ОН)2 и фоcфин РН3, газ с чесночным запахом.

«Ключевые слова» — … газ … раствор соляной кислоты. Фосфин, являясь аналогом аммиака, проявляет основные свойства и реагирует с кислотой с образованием соли фосфония РН4Сl (уравнение 4).

1) Са3+5O4)2 + 5С0 + 3SiO2 = 3CaSiO3 + 2Р0 + 5C+2O↑

2) 2Р + ЗСа = Са3Р2

3) Са3Р2 + 6Н2O = 3Са(ОН)2 + 2РН3

4) РН3 + НСl = РН4Сl

  1. Осадок, полученный при взаимодействии растворов хлорида железа (III) и нитрата серебра, отфильтровали. Фильтрат обработа­ли раствором едкого кали. Выпавший осадок бурого цвета отдели­ли и прокалили. Полученное вещество при нагревании реагирует c алюминием с выделением тепла и света. Напишите уравнения описанных реакций.

Анализ и решение.

Ключевые слова: «… взаимодействии растворов хлорида железа (III) и нитрата серебра…». При взаимодействии двух растворимых солей протекает реакция ионного обмена, если в продуктах реакции есть малодиссоциирующее вещество (газ, осадок, вода и др.). При взаимодействии нитрата серебра и хлорида железа (III) выпадает белый творожистый осадок хлорида серебра (уравнение 1).

Далее осадок отфильтровали. Фильтрование — способ разделения смесей, при котором нерастворимые в воде вещества не проходят через тонкую пористую перегородку (фильтр), а растворимые в воде вещества с растворителем (фильтрат) переходят в отдельную емкость.

При фильтровании данного в задании раствора в осадке остается нерастворимый в воде хлорид серебра, а в фильтрат уходит растворимая в воде соль — нитрат железа (III).

Далее, ключевые слова: «… фильтрат обработали раствором едкого кали.«. При взаимодействии нитрата железа (III) с гидроксидом калия KOH протекает обменная реакция, с образованием нерастворимого в воде осадка — гидроксида железа (III)(реакция 2).

Далее, ключевые слова: «Выпавший осадок бурого цвета отдели­ли и прокалили«. Гидроксид железа (III) — нерастворимое в воде основание. Как правило, нерастворимые основания при прокаливании разлагаются на оксид металла и воду (реакция 3).

Далее «Полученное вещество при нагревании реагирует c алюминием с выделением тепла и света«. Данное описание соответствует окислительно-восстановительной реакции между оксидом железа (III) и алюминием. При нагревании смеси этих соединений происходит экзотермическая реакция восстановления железа до простого вещества-металла, при этом теплота выделяется также в форме света. Восстановление металлов из оксидов с помощью алюминия называют алюмотермией.

1) FeCl3 + 3AgNO3 = 3AgCl + Fe(NO3)3

2) Fe(NO3)+ 3KOH = Fe(OH)3 + 3KNO3

3)  2Fe(OH)3 = Fe2O3 + 3H2O

4)  Fe2O3 + 2Al = Al2O3 + 2Fe

4. Вещество, полученное на катоде при электролизе расплава хлорида натрия, сожгли в кислороде. Полученный продукт последо­вательно обработали сернистым газом и раствором гидроксида ба­рия. Напишите уравнения описанных реакций.

Анализ и решение.

Первая часть: «… при электролизе расплава хлорида натрия«. Электролиз — это химическая реакция, протекающая под действием тока. При электролизе расплавов солей на катоде происходит восстановление металлов до простых веществ, на аноде — окисление неметаллов до простых веществ (уравнение 1).

Далее, ключевые слова: «Вещество, полученное на катоде …  сожгли в кислороде«. При электролизе расплава хлорида натрия на катоде выделяется металлический натрий. При сжигании простое вещество натрий образует преимущественно пероксид (уравнение 2).

Ключевые слова: «Полученный продукт … обработали сернистым газом«. Пероксид натрия, как и пероксид водорода, проявляет двойственные свойства, и может выступать и как окислитель, и как восстановитель. Сернистый газ SO2типичный восстановитель. При взаимодействии с сернистым газом кислород пероксида натрия выступает в качестве окислителя и восстанавливается до степени окисления -2 (уравнение 3).  Подробнее про ОВР — в статье «Окислительно-восстановительные реакции».

Ключевые слова: » … Полученный продукт последо­вательно обработали сернистым газом и раствором гидроксида ба­рия«. Сульфат натрия, полученный в предыдущей стадии, вступает в реакцию ионного обмена с гидроксидом бария с выпадением осадка сульфата бария (уравнение 4).

1) 2NaCl = 2Na + Cl2

2) 2Na + O2 = Na2O2

3) Na2O2 + SO2 = Na2SO4

4) Na2SO4 + Ba(OH)2 = NaOH + BaSO4

5. Продукты разложения хлорида аммония последовательно про­пустили через нагретую трубку, содержащую оксид меди (II), а затем через склянку с оксидом фосфора (V). Напишите уравнения описан­ных реакций.

Анализ и решение.

Ключевые слова: «Продукты разложения хлорида аммония…». Хлорид аммония — соль, которая разлагается при нагревании твердой соли на газообразный аммиак и газ хлороводород (уравнение 1)

Далее, продукты разложения последовательно пропускают через нагретую трубку, содержащую оксид меди (II). Последовательно, значит, они реагируют по очереди. Оксид меди (II) — основный, при взаимодействии с кислотой HCl образует соль и воду (уравнение 2). Оксид меди (II) также проявляет окислительные свойства, при взаимодействии с аммиаком восстанавливается до простого вещества — меди, а аммиак окисляется также до простого вещества (уравнение 3). Подробнее про ОВР — в статье «Окислительно-восстановительные реакции».

Далее, продукты реакций 2 и 3 пропускают через емкость с оксидом фосфора (V). Анализируем возможность протекания химической реакции между веществами. Простое вещество медь химически малоактивно и не реагирует с кислотным оксидом фосфора. Простое вещество азот также химически малоактивно, с оксидом фосфора (V)  не реагирует. Зато с кислотным оксидом фосфора (V) отлично реагируют пары воды с образованием орто-фосфорной кислоты (уравнение 4).

1) NH4Cl = NH3 + HCl

2) CuO + 2HCl = CuCl2 + h3O

3) 3CuO + 2NH3 = 3Cu + N2 + 3h3O

4) 3H2O + P2O5 = 2H3PO4

6. К нерастворимой в воде соли белого цвета, которая встречается в природе в виде широко используемого в строительстве и архитек­туре минерала, прилили раствор соляной кислоты, в результате соль растворилась и выделился газ, при пропускании которого через из­вестковую воду выпал осадок белого цвета; осадок растворился при дальнейшем пропускании газа. При кипячении полученного раство­ра выпадает осадок. Напишите уравнения описанных реакций.

Анализ и решение.

Из школьного курса химии хорошо известно, что нерастворимой в воде солью белого цвета, которая встречается в природе в виде широко используемого в строительстве и архитек­туре минерала является карбонат кальция CaCO3. Нерастворимые соли растворяются под действием более сильных кислот, в данном случае, соляной кислоты (уравнение 1).

Образующийся газ пропускают через известковую воду Ca(OH)2. Углекислый газ — типичный кислотный оксид, который при взаимодействии с щелочью образует соль — карбонат кальция (уравнение 2). Далее осадок растворился при дальнейшем пропускании газа. Здесь рассматривается очень важное свойство: средние соли многоосновных кислот под действием избытка кислоты образуют более кислые соли. Карбонат кальция в избытке углекислого газа образует более кислую соль — гидрокарбонат кальция Ca(HCO3)2, который хорошо растворим в воде (уравнение 3).

Свойства кислых солей в значительной степени складываются из свойств образующих кислые соли соединений. Свойства гидрокарбоната кальция определяются свойствами образующих его соединений — угольной кислоты H2CO3 и карбоната кальция. Несложно вывести, что при кипячении гидрокарбонат будет разлагаться на карбонат кальция (раздагается при более высоких температурах, порядка 1200 градусов Цельсия), углекислый газ и воду (уравнение 4).

1) CaCO3 + 2HCl = CaCl2 + CO2 + H2O

2) CO2 +Ca(OH)2 = CaCO3 + H2O

3) CaCO3 + H2O + CO2 = Ca(HCO3)2

4)  Ca(HCO3)2 = CaCO3 + H2O + CO2

7. Вещество, полученное на аноде при электролизе раствора йодида натрия с инертными электродами, прореагировало с серово­дородом. Образовавшееся твёрдое вещество сплавили с алюминием и продукт растворили в воде. Напишите уравнения описанных ре­акций.

Анализ и решение.

Электролиз раствора йодида натрия с инертными электродами описывается уравнением:

1. 2NaI + 2H2O = 2NaOH + H2 + I2

Более подробно про электролиз можно прочитать в соответствующей статье. При этом на аноде выделяется йод. Йод реагирует с сероводородом. При этом йод окислитель, а сера — восстановитель:

2. I20 + H2S-2 = 2HI + S0

При этом образовалась твердая сера. Сера реагирует с алюминием при сплавлении, образуя сульфид алюминия. Большинство неметаллов реагируют с металлами с образованием бинарных соединений:

3. 3S0 + 2Al0 = Al2+3S3-2

Продукт реакции алюминия с серой — сульфид алюминия — при растворении в воде необратимо распадается на гидроксид алюминия и сероводород:

4. Al2S3 + 12H2O = 2Al(OH)3 + 3H2S

Такие реакции называют также реакциями необратимого гидролиза. Подробно случаи необратимого гидролиза рассмотрены в статье.

8. Газ, выделившийся при взаимодействии хлористого водоро­да с перманганатом калия, реагирует с железом. Продукт реакции растворили в воде и добавили к нему сульфид натрия. Более легкое из образовавшихся нерастворимых веществ отделили и ввели в ре­акцию с горячей концентрированной азотной кислотой. Напишите уравнения описанных реакций.

9. Сульфид хрома (III) обработали водой, при этом выделился газ и осталось нерастворимое вещество. К этому веществу прибави­ли раствор едкого натра и пропустили газообразный хлор, при этом раствор приобрёл жёлтое окрашивание. Раствор подкислили сер­ной кислотой, в результате окраска изменилась на оранжевую; через полученный раствор пропустили газ, выделившийся при обработке сульфида водой, и цвет раствора изменился на зелёный. Напишите уравнения описанных реакций.

Анализ и решение.

Ключевые слова: «Сульфид хрома (III) обработали водой, при этом выделился газ и осталось нерастворимое вещество«. Сульфид  хрома (III) под действием воды разлагается на гидроксид и сероводород. Подробно реакции гидролиза таких соединений рассмотрены в статье «Гидролиз». (реакция 1)

1) Cr2S3 + 6H2O = 2Cr(OH)3 + 3H2S

Ключевые слова: «… прибави­ли раствор едкого натра и пропустили газообразный хлор, при этом раствор приобрёл жёлтое окрашивание«. Под действием хлора в щелочной среде хром +3 окисляется до хрома +6. Хром +6 образует кислотный оксид и гидроксид, в растворе едкого натрия образует устойчивую соль желтого цвета — хромат натрия (реакция 2).

2)  2Cr+3(OH)3 + 3Cl02 + 10NaOH = 2Na2Cr+6O4 + 6NaCl + 8H2O

Далее, ключевые слова: «Раствор подкислили сер­ной кислотой, в результате окраска изменилась на оранжевую«. Соли-хроматы в кислой среде переходят в дихроматы. Желтый хромат натрия в кислой среде превращается в оранжевый дихромат натрия (реакция 3). Это не ОВР!

3) 2Na2CrO4 + H2SO4 = Na2Cr2O7 + Na2SO4 + H2O.

Далее: «… через полученный раствор пропустили газ, выделившийся при обработке сульфида водой, и цвет раствора изменился на зелёный«. Дихромат натрия — сильный окислитель, при взаимодействии с сероводородом восстанавливается до трехвалентной соли хрома. Соединения хрома (III) — амфотерные, в кислой среде образуют соли. Соли хрома (III) окрашивают раствор в зеленый цвет (реакция 4).

4) Na2Cr+62O7 + 3H2S-2 + 4H2SO4 = 3S0 + Cr+32(SO4)3 + Na2SO4 + 7H2O

10. Получение чёрно-белого изображения при фотографирова­нии основано на разложении соли неизвестного металла под дей­ствием света. При растворении этого металла в разбавленной азот­ной кислоте выделяется бесцветный газ, который на воздухе быстро изменяет свой цвет на бурый, и образуется соль, взаимодействующая с бромидом натрия с образованием творожистого осадка желтоватого цвета. Анионом в соли, используемой в фотографии, является анион кислоты, которая образуется одновременно с серной кислотой, при взаимодействии бромной воды и сернистого газа. Напишите уравне­ния описанных реакций.

11. В раствор, полученный при взаимодействии алюминия с раз­бавленной серной кислотой, по каплям добавляли раствор гидроксида натрия до образования осадка. Выпавший осадок белого цвета отфильтровали и прокалили. Полученное вещество сплавили с кар­бонатом натрия. Напишите уравнения описанных реакций.

Анализ и решение.

Разбавленная серная кислота при взаимодействии с металлами ведет себя, как обычная минеральная кислота. Металлы, находящиеся в ряду электрохимической активности левее водорода, при взаимодействии с минеральными кислотами вытесняют водород:

1. 2Al0 + 3H+2SO4 = Al+32(SO4)3 + 3H02

Далее, с гидроксидом натрия взаимодействует сульфат алюминия. В условии указано, что гидроксид натрия добавляли по каплям. Это значит, что гидроксид натрия был в недостатке, а сульфат алюминия -в значительном избытки. При таких условиях образуется осадок гидроксида алюминия:

2. Al2(SO4)3 + 6NaOH = 2Al(OH)3 + 3Na2SO4

Осадок белого цвета — гидроксид алюминия, нерастворимый в воде. При прокаливании нерастворимые гидроксиды разлагаются на воду и соответствующий оксид:

3. 2Al(OH)3 = Al2O3 + 3H2O

Полученное вещество — оксид алюминия — сплавили с карбонатом натрия. В расплаве менее летучие оксиды вытесняют более летучие из солей. Карбонат — это соль, которой соответствует летучий оксид, углекислый газ. Соответственно, при сплавлении карбонатов щелочных металлов с твердыми оксидами (кислотными и амфотерными) образуется соль, соответствующая этому оксиду,  и углекислый газ:

4. Al2O3 + Na2CO3 = 2NaAlO2 + CO2

12. Через раствор хлорида меди (II) с помощью графитовых элек­тродов пропускали постоянный электрический ток. Выделившийся на катоде продукт электролиза растворили в концентрированной азотной кислоте. Образовавшийся при этом газ собрали и пропусти­ли через раствор гидроксида натрия. Выделившийся на аноде газо­образный продукт электролиза пропустили через горячий раствор гидроксида натрия. Напишите уравнения описанных реакций.

13. Простое вещество, полученное при нагревании смеси фос­фата кальция с коксом и оксидом кремния, растворяется в растворе едкого кали. Выделяющееся газообразное вещество сожгли, продук­ты горения собрали и охладили, а в полученный раствор добавили нитрат серебра. Напишите уравнения описанных реакций.

14. Зловонную жидкость, образовавшуюся при взаимодействии бромистого водорода с перманганатом калия, отделили и нагрели с железной стружкой. Продукт реакции растворили в воде и добавили к нему раствор гидроксида цезия. Образовавшийся осадок отфильтро­вали и прокалили. Напишите уравнения описанных реакций.

15. Над поверхностью раствора едкого натра пропускали элек­трические разряды, при этом воздух окрашивался в бурый цвет, при­чём окраска через некоторое время исчезала. Полученный раствор осторожно выпарили и установили, что твёрдый остаток представ­ляет собой смесь двух солей. Выдерживание смеси солей на воздухе приводит к образованию одного вещества. Напишите уравнения описанных реакций.

16. Кальций растворили в воде. При пропускании через получен­ный раствор сернистого газа образуется осадок белого цвета, кото­рый растворяется при пропускании избытка газа. Добавление к по­лученному раствору щёлочи приводит к образованию осадка белого цвета. Напишите уравнения описанных реакций.

17. При сжигании на воздухе простого вещества жёлтого цвета образуется газ с резким запахом. Этот газ выделяется также при об­жиге некоторого минерала, содержащего железо, на воздухе. При действии разбавленной серной кислоты на вещество, состоящее из тех же элементов, что и минерал, но в другом соотношении, выде­ляется газ с характерным запахом тухлых яиц. При взаимодействии выделившихся газов друг с другом образуется исходное простое ве­щество. Напишите уравнения описанных реакций.

18. Газообразный продукт взаимодействия сухой поваренной соли с концентрированной серной кислотой ввели в реакцию с раст­вором перманганата калия. Выделившийся газ пропустили через рас­твор сульфида натрия. Выпавший осадок жёлтого цвета растворяется в концентрированном растворе гидроксида натрия. Напишите урав­нения описанных реакций.

19. Газ, образовавшийся при пропускании хлористого водорода через горячий раствор хромата калия, взаимодействует с железом. Продукт реакции растворили в воде и добавили к нему сульфид нат­рия. Более легкое из образовавшихся нерастворимых веществ отде­лили и ввели в реакцию с концентрированной серной кислотой при нагревании. Напишите уравнения описанных реакций.

20. Две соли содержат одинаковый катион. Термический распад первой из них напоминает извержение вулкана, при этом выделя­ется малоактивный бесцветный газ, входящий в состав атмосферы. При взаимодействии второй соли с раствором нитрата серебра обра­зуется белый творожистый осадок, а при нагревании её с раствором щёлочи выделяется бесцветный ядовитый газ с резким запахом; этот газ может быть получен также при взаимодействии нитрида магния с водой. Напишите уравнения описанных реакций.

21. К раствору сульфата алюминия добавили избыток раствора гидроксида натрия. В полученный раствор небольшими порциями прибавляли соляную кислоту, при этом наблюдали образование объ­ёмного осадка белого цвета, который растворился при дальнейшем прибавлении кислоты. В образовавшийся раствор прилили раствор карбоната натрия. Напишите уравнения описанных реакций.

22. Над поверхностью налитого в колбу раствора едкого натра пропускали электрические разряды, при этом воздух в колбе окра­шивался в бурый цвет, который исчезал через некоторое время. По­лученный раствор осторожно выпарили и установили, что твёрдый остаток представляет собой смесь двух солей. При нагревании этой смеси выделяется газ и остаётся единственное вещество. Напишите уравнения описанных реакций.

23. Оксид цинка растворили в растворе хлороводородной кисло­ты и раствор нейтрализовали, добавляя едкий натр. Выделившееся студенистое вещество белого цвета отделили и обработали избытком раствора щёлочи, при этом осадок полностью растворился. Нейтра­лизация полученного раствора кислотой, например, азотной, при­водит к повторному образованию студенистого осадка. Напишите уравнения описанных реакций.

24. Вещество, полученное на катоде при электролизе расплава хлорида меди (II), реагирует с серой. Полученный продукт обра­ботали концентрированной азотной кислотой и выделившийся газ пропустили через раствор гидроксида бария. Напишите уравнения описанных реакций.

25. Смесь ортофосфата кальция, кокса и песка нагревали в элек­трической печи. Один из продуктов этой реакции может самовоспла­меняться на воздухе. Твёрдый продукт горения этого вещества при нагревании растворили в воде и через полученный раствор пропусти­ли газообразный аммиак. Напишите уравнения описанных реакций.

26. Вещество, полученное на катоде при электролизе раствора хлорида железа (II), сплавили с серой и продукт этой реакции под­вергли обжигу. Образовавшийся газ пропустили через раствор гидро­ксида бария. Напишите уравнения описанных реакций.

27. В нагретую концентрированную серную кислоту внесли мед­ную проволоку и выделяющийся газ пропустили через избыток раст­вора едкого натра. Раствор осторожно выпарили, твёрдый остаток растворили в воде и нагрели с порошкообразной серой. Непрореагировавшую серу отделили фильтрованием и к раствору прибавили серную кислоту, при этом наблюдали образование осадка и выделе­ние газа с резким запахом. Напишите уравнения описанных реакций.

28. После кратковременного нагревания неизвестного порошко­образного вещества оранжевого цвета начинается самопроизвольная реакция, которая сопровождается изменением цвета на зелёный, вы­делением газа и искр. Твёрдый остаток смешали с едким кали и на­грели, полученное вещество внесли в разбавленный раствор соля­ной кислоты, при этом образовался осадок зелёного цвета, который растворяется в избытке кислоты. Напишите уравнения описанных реакций.

29. Две соли окрашивают пламя в фиолетовый цвет. Одна из них бесцветна, и при лёгком нагревании её с концентрированной серной кислотой отгоняется жидкость, в которой растворяется медь; послед­нее превращение сопровождается выделением бурого газа. При до­бавлении к раствору второй соли раствора серной кислоты жёлтая окраска раствора изменяется на оранжевую, а при нейтрализации полученного раствора щёлочью восстанавливается первоначальный цвет. Напишите уравнения описанных реакций.

30. Раствор хлорида железа (III) подвергли электролизу с графи­товыми электродами. Осадок бурого цвета, образовавшийся в каче­стве побочного продукта электролиза, отфильтровали и прокалили. Вещество, образовавшееся на катоде, растворили в концентриро­ванной азотной кислоте при нагревании. Продукт, выделившийся на аноде, пропустили через холодный раствор гидроксида калия. На­пишите уравнения описанных реакций.

31. Газ, выделившийся при взаимодействии хлористого водорода с бертолетовой солью, ввели в реакцию с алюминием. Продукт реак­ции растворили в воде и добавили гидроксид натрия до прекращения выделения осадка, который отделили и прокалили. Напишите урав­нения описанных реакций.

32. Неизвестная соль бесцветна и окрашивает пламя в жёлтый цвет. При лёгком нагревании этой соли с концентрированной сер­ной кислотой отгоняется жидкость, в которой растворяется медь; по­следнее превращение сопровождается выделением бурого газа и об­разованием соли меди. При термическом распаде обеих солей одним из продуктов разложения является кислород. Напишите уравнения описанных реакций.

33. Вещество, полученное на аноде при электролизе расплава йодида натрия с инертными электродами, выделили и ввели во взаи­модействие с сероводородом. Газообразный продукт последней реак­ции растворили в воде и к полученному раствору добавили хлорное железо. Образовавшийся осадок отфильтровали и обработали горя­чим раствором гидроксида натрия. Напишите уравнения описанных реакций.

34. Газы, которые выделяются при нагревании угля в концентри­рованных азотной и серной кислотах, смешали друг с другом. Про­дукты реакции пропустили через известковое молоко. Напишите уравнения описанных реакций.

35. Смесь железного порошка и твёрдого продукта, полученно­го при взаимодействии сернистого газа и сероводорода, нагрели без доступа воздуха. Полученный продукт подвергли обжигу на воздухе. Образовавшееся твёрдое вещество реагирует с алюминием с выделе­нием большого количества тепла. Напишите уравнения описанных реакций.

36. Вещество чёрного цвета получили, прокаливая осадок, который образуется при взаимодействии растворов гидроксида натрия и сульфата меди (II). При нагревании этого вещества с углем полу­чают металл красного цвета, который растворяется в концентриро­ванной серной кислоте. Напишите уравнения описанных реакций.

37. Простое вещество, смесь которого с бертолетовой солью используется в спичках и воспламеняется при трении, сожгли в из­бытке кислорода. Твёрдое вещество белого цвета, образовавшееся в результате сгорания, растворили в избытке раствора гидроксида натрия. Полученная при этом соль с раствором нитрата серебра об­разует осадок ярко-жёлтого цвета. Напишите уравнения описанных реакций.

38. Цинк растворили в очень разбавленной азотной кислоте и в полученный раствор добавили избыток щёлочи, получив про­зрачный раствор. Напишите уравнения описанных реакций.

39. Раствор, полученный при пропускании сернистого газа че­рез бромную воду, нейтрализовали гидроксидом бария. Выпавший осадок отделили, смешали с коксом и прокалили. При обработке продукта прокаливания хлороводородной кислотой выделяется газ с запахом тухлых яиц. Напишите уравнения описанных реакций.

40. Вещество, образующееся при добавлении цинкового порошка в раствор хлористого железа, отделили фильтрованием и растворили в горячей разбавленной азотной кислоте. Раствор упарили, твёрдый остаток прокалили и выделившиеся газы пропустили через раствор гидроксида натрия. Напишите уравнения описанных реакций.

41. Газ, выделившийся при нагревании раствора хлористого водо­рода с оксидом марганца (IV), ввели во взаимодействие с алюмини­ем. Продукт реакции растворили в воде и добавили сначала избыток раствора гидроксида натрия, а затем соляную кислоту (избыток). На­пишите уравнения описанных реакций.

42. Смесь двух бесцветных, не имеющих цвета и запаха, газов А и В пропустили при нагревании над катализатором, содержащим железо, и образующимся при этом газом В нейтрализовали раствор бромоводородной кислоты. Раствор выпарили и остаток нагрели с едким кали, в результате выделился бесцветный газ В с резким за­пахом. При сжигании газа В на воздухе образуются вода и газ А. На­пишите уравнения описанных реакций.

43. Сернистый газ пропустили через раствор перекиси водорода. Из образовавшегося раствора выпарили воду и к остатку добавили магниевую стружку. Выделяющийся газ пропустили через раствор медного купороса. Выпавший осадок чёрного цвета отделили и под­вергли обжигу. Напишите уравнения описанных реакций.

44. К нерастворимой в воде соли белого цвета, которая встреча­ется в природе в виде широко используемого в строительстве и архи­тектуре минерала, прилили раствор соляной кислоты, в результате соль растворилась и выделился газ, при пропускании которого через известковую воду выпал осадок белого цвета, который растворился при дальнейшем пропускании газа. При добавлении в полученный раствор избытка известковой воды выпадает осадок. Напишите урав­нения описанных реакций.

45. При обжиге некоторого минерала А, состоящего из двух эле­ментов, образуется газ, имеющий характерный резкий запах и обес­цвечивающий бромную воду с образованием в растворе двух силь­ных кислот. При взаимодействии вещества Б, состоящего из тех же элементов, что и минерал А, но в другом соотношении, с концен­трированной хлороводородной кислотой выделяется ядовитый газ с запахом тухлых яиц. При взаимодействии выделившихся газов друг с другом образуются простое вещество жёлтого цвета и вода. Напи­шите уравнения описанных реакций.

46. Вещество, выделяющееся на катоде при электролизе распла­ва хлорида натрия, сожгли в кислороде. Полученный продукт поме­стили в газометр, наполненный углекислым газом. Образовавшееся вещество добавили в раствор хлорида аммония и раствор нагрели. Напишите уравнения описанных реакций.

47. Азотную кислоту нейтрализовали пищевой содой, нейтраль­ный раствор осторожно выпарили и остаток прокалили. Образовав­шееся вещество внесли в подкисленный серной кислотой раствор перманганата калия, при этом раствор обесцветился. Азотсодержа­щий продукт реакции поместили в раствор едкого натра и добавили цинковую пыль, при этом выделился газ с резким характерным запа­хом. Напишите уравнения описанных реакций.

48. При взаимодействии раствора соли А со щёлочью было по­лучено студенистое нерастворимое в воде вещество голубого цвета, которое растворили в бесцветной жидкости Б с образованием раст­вора синего цвета. Твёрдый продукт, оставшийся после осторожного выпаривания раствора, прокалили; при этом выделились два газа, один из которых бурого цвета, а второй входит в состав атмосфер­ного воздуха, и осталось твёрдое вещество чёрного цвета, которое растворяется в жидкости Б с образованием вещества А. Напишите уравнения описанных реакций.

49. Белый фосфор растворяется в растворе едкого кали с выде­лением газа с чесночным запахом, который самовоспламеняется на воздухе. Твёрдый продукт реакции горения прореагировал с едким натром в таком соотношении, что в образовавшемся веществе белого цвета содержится один атом водорода; при прокаливании последнего вещества образуется пирофосфат натрия. Напишите уравнения опи­санных реакций.

50. На раствор хлорного железа подействовали раствором едкого натра, выпавший осадок отделили и нагрели. Твёрдый продукт реак­ции смешали с кальцинированной содой и прокалили. К оставшему­ся веществу добавил и нитрат и гидроксид натрия и длительное время нагревали при высокой температуре. Напишите уравнения описан­ных реакций.

Газ, выделившийся при взаимодействии хлористого водоро­да с перманганатом калия, пропустили через раствор тетрагидроксоалюмината натрия. Образовавшийся осадок отфильтровали, про­калили и твёрдый остаток обработали соляной кислотой. Напишите уравнения описанных реакций.

Азотоводородную смесь нагрели до температуры 500°С и под высоким давлением пропустили над железным катализатором. Про­дукты реакции пропускали через раствор азотной кислоты до его нейтрализации. Образовавшийся раствор осторожно выпарили, твёрдый остаток прокалили и выделившийся при этом газ пропу­стили над медью при нагревании, в результате образовалось твёрдое вещество чёрного цвета. Напишите уравнения описанных реакций.

Гидроксид трёхвалентного хрома обработали соляной кисло­той. В полученный раствор добавили поташ, выделившийся осадок отделили и внесли в концентрированный раствор едкого кали, в ре­зультате осадок растворился. После добавления избытка соляной кислоты был получен раствор зелёного цвета. Напишите уравнения описанных реакций.

Вещество, полученное на аноде при электролизе раствора йодида натрия с инертными электродами, ввели в реакцию с кали­ем. Продукт реакции нагрели с концентрированной серной кисло­той и выделившийся газ пропустили через горячий раствор хромата калия. Напишите уравнения описанных реакций.

Оксид двухвалентного железа нагрели с разбавленной азотной кислотой. Раствор осторожно выпарили, твёрдый остаток раствори­ли в воде, в получившийся раствор внесли железный порошок и че­рез некоторое время профильтровали. К фильтрату добавили раствор едкого кали, выпавший осадок отделили и оставили на воздухе, при этом цвет вещества изменился. Напишите уравнения описанных ре­акций.

Одно из веществ, образующихся при сплавлении оксида кремния с магнием, растворяется в щёлочи. Выделяющийся газ вве­ли в реакцию с серой, а продукт их взаимодействия обработали хло­ром. Напишите уравнения описанных реакций.

Твёрдое вещество, образующееся при взаимодействии серни­стого газа и сероводорода, при нагревании взаимодействует с алюми­нием. Продукт реакции растворили в разбавленной серной кислоте и в образовавшийся раствор добавили поташ. Напишите уравнения описанных реакций.

Неизвестный металл сожгли в кислороде. Продукт реакции, взаимодействуя с углекислым газом, образует два вещества: твёрдое, которое взаимодействует с раствором соляной кислоты с выделением углекислого газа, и газообразное простое вещество, поддерживающее горение. Напишите уравнения описанных реакций.

Продукт взаимодействия азота и лития обработали водой. Вы­делившийся в результате реакции газ смешали с избытком кислорода и при нагревании пропустили над платиновым катализатором; обра­зовавшаяся газовая смесь имела бурый цвет. Напишите уравнения описанных реакций.

Медную стружку растворили в разбавленной азотной кислоте и раствор нейтрализовали едким кали. Выделившееся вещество го­лубого цвета отделили, прокалили (цвет вещества изменился на чёр­ный), смешали с коксом и повторно прокалили. Напишите уравне­ния описанных реакций.

Фосфор сожгли в избытке хлора, образующееся твёрдое веще­ство смешали с фосфором и нагрели. Продукт реакции обработали водой, при этом выделялся бесцветный газе резким запахом. Раствор добавили к подкисленному серной кислотой раствору перманганата калия, который в результате реакции обесцветился. Напишите урав­нения описанных реакций.

Хлористое железо обработали при нагревании концентриро­ванной азотной кислотой и раствор осторожно выпарили. Твёрдый продукт растворили в воде, добавили к полученному раствору поташ и выпавший осадок отделили и прокалили. Над полученным веще­ством пропустили при нагревании газообразный водород. Напишите уравнения описанных реакций.

Неизвестная соль при взаимодействии с раствором нитрата серебра образует осадок белого цвета и окрашивает пламя горелки в жёлтый цвет. При взаимодействии концентрированной серной кислоты с этой солью образуется ядовитый газ, хорошо раствори­мый в воде. В полученном растворе растворяется железо, при этом выделяется очень легкий бесцветный газ, который используется для получения металлов, например, меди, из их оксидов. Напишите уравнения описанных реакций.

Силицид магния обработали раствором хлороводородной кислоты и выделяющийся газ сожгли. Твёрдый продукт реакции смешали с кальцинированной содой, смесь нагрели до плавления и выдержали некоторое время. После охлаждения продукт реакции (широко используется под названием «жидкое стекло») растворили в воде и обработали раствором серной кислоты. Напишите уравне­ния описанных реакций.

Газовую смесь аммиака и большого избытка воздуха пропу­стили при нагревании над платиной и продукты реакции через неко­торое время поглотили раствором едкого натра. После выпаривания раствора был получен единственный продукт. Напишите уравнения описанных реакций.

К раствору хлорного железа добавили кальцинированную соду и выпавший осадок отделили и прокалили. Над полученным ве­ществом пропустили при нагревании угарный газ и твёрдый продукт последней реакции ввели во взаимодействие с бромом. Напишите уравнения описанных реакций.

Продукт взаимодействия серы с алюминием (реакция проте­кает при нагревании) растворили в холодной разбавленной серной кислоте и в раствор добавили карбонат калия. Образовавшийся оса­док отделили, смешали с едким натром и нагрели. Напишите урав­нения описанных реакций.

Хлорид кремния (IV) нагревали в смеси с водородом. Продукт реакции смешали с магниевым порошком, нагрели и обработали во­дой; одно из образующихся веществ самовоспламеняется на воздухе. Напишите уравнения описанных реакций.

Через избыток раствора едкого кали пропустили бурый газ в присутствии большого избытка воздуха. В образовавшийся раст­вор добавили магниевую стружку и нагрели; выделившимся газом нейтрализовали азотную кислоту. Полученный раствор осторожно выпарили, твёрдый продукт реакции прокалили. Напишите уравне­ния описанных реакций.

Железную окалину растворили в концентрированной азотной кислоте при нагревании. Раствор осторожно выпари­ли и продукт реакции растворили в воде. К полученному раство­ру добавили железный порошок, через некоторое время раствор отфильтровали и фильтрат обработали раствором едкого кали, в результате выделился осадок светло-зелёного цвета, который бы­стро темнел на воздухе. Напишите уравнения описанных реакций.

К раствору кальцинированной соды добавили раствор хлори­да алюминия, выделившееся вещество отделили и внесли в раствор едкого натра. В образовавшийся раствор по каплям прибавляли раст­вор хлороводородной кислоты до прекращения образования осадка, который отделили и прокалили. Напишите уравнения описанных реакций.

В раствор нитрата ртути (II) добавили медную стружку. После окончания реакции раствор профильтровали и фильтрат по каплям прибавляли к раствору, содержащему едкий натр и гидроксид аммо­ния. При этом наблюдали кратковременное образование осадка, ко­торый растворялся с образованием раствора ярко-синего цвета. При добавлении в полученный раствор избытка раствора серной кислоты происходило изменение цвета. Напишите уравнения описанных ре­акций.

Продукт взаимодействия фосфида магния с водой сожгли и продукты реакции поглотили водой. Образовавшееся вещество ис­пользуется в промышленности для получения двойного суперфосфа­та из фосфорита. Напишите уравнения описанных реакций.

Соль, полученную при взаимодействии оксида цинка с сер­ной кислотой, прокалили при 800 °С. Твёрдый продукт реакции обра­ботали концентрированным раствором щёлочи и через полученный раствор пропустили углекислый газ. Напишите уравнения реакций описанных превращений.

К раствору хлорного железа добавили железный порошок и через некоторое время раствор профильтровали. К фильтрату до­бавили гидроксид натрия, выделившийся осадок отделили и обрабо­тали перекисью водорода. К полученному веществу добавили избы­ток раствора едкого кали и бром; в результате протекания реакции окраска брома исчезла. Напишите уравнения описанных реакций.

Оксид меди (I) обработали концентрированной азотной кис­лотой, раствор осторожно выпарили и твёрдый остаток прокалили. Газообразные продукты реакции пропустили через большое количе­ство воды и в образовавшийся раствор добавили магниевую стружку, в результате выделился газ, используемый в медицине. Напишите уравнения описанных реакций.

Сернистый газ пропустили через раствор перекиси водорода. Раствор упарили и в оставшуюся жидкость добавили медную струж­ку. Выделившийся газ смешали с газом, который образуется при взаимодействии сульфида железа (II) с раствором бромоводородной кислоты. Напишите уравнения описанных реакций.

При добавлении в раствор соли жёлтого цвета, окрашиваю­щей пламя в фиолетовый цвет, разбавленной соляной кислоты окрас­ка изменилась на оранжево-красную. После нейтрализации раствора концентрированной щёлочью цвет раствора вернулся к первоначаль­ному. При добавлении в полученный раствор хлорида бария выпадает осадок жёлтого цвета. Осадок отфильтровали и в фильтрат добавили раствор нитрата серебра. Напишите уравнения описанных реакций.

Силицид магния обработали раствором соляной кислоты, продукт реакции сожгли, образовавшееся твёрдое вещество смешали с кальцинированной содой и нагрели до плавления. После охлажде­ния расплава его обработали водой и к полученному раствору доба­вили азотную кислоту. Напишите уравнения описанных реакций.

Нерастворимое вещество, образующееся при добавлении в раствор хлористого железа едкого натра, отделили и растворили в разбавленной серной кислоте. В полученный раствор добавили цинковую пыль, выделившийся осадок отфильтровали и растворили в концентрированной соляной кислоте. Напишите уравнения опи­санных реакций.

Нитрат алюминия прокалили, продукт реакции смешали с кальцинированной содой и нагрели до плавления. Образовавшееся вещество растворили в азотной кислоте и полученный раствор ней­трализовали раствором аммиака, при этом наблюдали выделение объёмного студенистого осадка. Напишите уравнения описанных реакций.

Нитрид магния обработали избытком воды. При пропускании выделяющегося газа как через бромную воду или через нейтральный раствор перманганата калия, так и при его сжигании образуется один и тот же газообразный продукт. Напишите уравнения описанных ре­акций.

Хлорная вода имеет запах хлора. При подщелачивании запах исчезает, а при добавлении соляной кислоты — становится более сильным, чем был ранее. Напишите уравнения описанных реакций.

Твёрдое вещество, образующееся при нагревании малахита, нагрели в атмосфере водорода. Продукт реакции обработали концен­трированной серной кислотой и, после отделения от серной кисло­ты, внесли в раствор хлорида натрия, содержащий медные опилки, и результате образовался осадок. Напишите уравнения описанных реакций.

Фосфин пропустили через горячий раствор концентрирован­ной азотной кислоты. Раствор нейтрализовали негашёной известью, выпавший осадок отделили, смешали с коксом и кремнезёмом и про­калили. Продукт реакции, который светится на воздухе, нагрели в растворе едкого натра. Напишите уравнения описанных реакций.

Железный порошок растворили в большом количестве раз­бавленной серной кислоты и через полученный раствор пропустили воздух, а затем газ с запахом тухлых яиц. Образовавшуюся нераство­римую соль отделили и растворили в горячем растворе концентриро­ванной азотной кислоты. Напишите уравнения описанных реакций.

Бесцветные газы выделяются при выдерживании концентри­рованной серной кислоты как с хлоридом натрия, так и с йодидом натрия. При пропускании этих газов через водный раствор аммиака образуются соли. Напишите уравнения описанных реакций.

Магниевый порошок смешали с кремнием и нагрели. Про­дукт реакции обработали холодной водой и выделяющийся газ про­пустили через горячую воду. Образовавшийся осадок отделили, сме­шали с едким натром и нагрели до плавления. Напишите уравнения описанных реакций.

Один из продуктов взаимодействия аммиака с бромом — газ, входящий в состав атмосферы, смешали с водородом и нагрели в присутствии платины. Образовавшуюся смесь газов пропустили через раствор соляной кислоты и к полученному раствору добави­ли при небольшом нагревании нитрит калия. Напишите уравнения описанных реакций.

Соль, полученную при растворении меди в разбавленной азотной кислоте, подвергли электролизу, используя графитовые электроды. Вещество, выделившееся на аноде, ввели во взаимодей­ствие с натрием, а полученный продукт реакции поместили в сосуд с углекислым газом. Напишите уравнения описанных реакций.

Неизвестное вещество А растворяется в концентрированной соляной кислоте, процесс растворения сопровождается выделением газа с запахом тухлых яиц; после нейтрализации раствора щёлочью образуется объёмный осадок белого (светло-зелёного) цвета. При обжиге вещества А образуются два оксида. Один из них — газ, имею­щий характерный резкий запах и обесцвечивающий бромную воду с образованием в растворе двух сильных кислот. Напишите уравне­ния описанных реакций.

Магний нагрели в сосуде, наполненном газообразным аммиа­ком. Образовавшееся вещество растворили в концентрированном растворе бромоводородной кислоты, раствор выпарили и остаток нагревали до появления запаха, после чего добавили раствор щёлочи. Напишите уравнения описанных реакций.

К раствору сульфата трёхвалентного хрома добавили кальци­нированную соду. Выделившийся осадок отделили, перенесли в раст­вор едкого натра, добавили бром и нагрели. После нейтрализации продуктов реакции серной кислотой раствор приобретает оранжевую окраску, которая исчезает после пропускания через раствор серни­стого газа. Напишите уравнения описанных реакций.

Негашёную известь прокалили с избытком кокса. Продукт ре­акции после обработки водой используется для поглощения серни­стого и углекислого газов. Напишите уравнения описанных реакций.

На сульфид двухвалентного железа подействовали раствором соляной кислоты, выделяющийся газ собрали и сожгли в воздухе. Продукты реакции пропустили через избыток раствора едкого кали, после чего в образовавшийся раствор добавили раствор перманганата калия. Напишите уравнения описанных реакций.

Твёрдый продукт термического разложения малахита раство­рили при нагревании в концентрированной азотной кислоте. Раст­вор осторожно выпарили и твёрдый остаток прокалили, получив вещество чёрного цвета, которое нагрели в избытке аммиака (газ). Напишите уравнения описанных реакций.

Красный фосфор сожгли в атмосфере хлора. Продукт реакции обработали избытком воды и в раствор добавили порошкообразный цинк. Выделяющийся газ пропустили над нагретым оксидом двух­валентного железа. Напишите уравнения реакций описанных пре­вращений.

Серебристо-серый металл, который притягивается магнитом, внесли в горячую концентрированную серную кислоту и нагрели. Раствор охладили и добавили едкий натр до прекращения образо­вания аморфного осадка бурого цвета. Осадок отделили, прокалили и растворили в концентрированной соляной кислоте при нагрева­нии. Напишите уравнения описанных реакций.

Магниевую стружку нагревали в атмосфере азота и продукт реакции последовательно обработали кипящей водой, растворами серной кислоты и нитрата бария. Напишите уравнения описанных реакций.

При термическом разложении соли А в присутствии диокси­да марганца образовались бинарная соль Б и газ, поддерживающий горение и входящий в состав воздуха; при нагревании этой соли без катализатора образуются соль Б и соль высшей кислородсодержащей кислоты. При взаимодействии соли А с соляной кислотой выделя­ется жёлто-зелёный ядовитый газ (простое вещество) и образуется соль Б. Соль Б окрашивает пламя в фиолетовый цвет; при её взаимо­действии с раствором нитрата серебра выпадает осадок белого цвета. Напишите уравнения описанных реакций.

Осадок, полученный при добавлении в раствор сульфа­та алюминия каустической соды, отделили, прокалили, смешали с кальцинированной содой и нагрели до плавления. После обработ­ки остатка серной кислотой была получена исходная соль алюминия. Напишите уравнения описанных реакций.

Вещество, образующееся при сплавлении магния с кремни­ем, обработали водой, в результате образовался осадок и выделился бесцветный газ. Осадок растворили в соляной кислоте, а газ про­пустили через раствор перманганата калия, при этом образовались два нерастворимых в воде бинарных вещества. Напишите уравнения описанных реакций.

Вещество, полученное при нагревании железной окалины в атмосфере водорода, внесли в горячую концентрированную сер­ную кислоту и нагрели. Полученный раствор выпарили, остаток растворили в воде и обработали раствором хлорида бария. Раствор профильтровали и в фильтрат внесли медную пластинку, которая че­рез некоторое время растворилась. Напишите уравнения описанных реакций.

Негашёную известь «погасили» водой. В полученный раст­вор пропустили газ, который выделяется при кальцинировании гид­рокарбоната натрия, при этом наблюдали образование и последую­щее растворение осадка. Напишите уравнения описанных реакций.

Смесь азота и водорода последовательно пропустили над на­гретой платиной и через раствор серной кислоты. В раствор добавили хлорид бария и после отделения выпавшего осадка — известковое молоко и нагрели. Напишите уравнения описанных реакций.

Приведите примеры взаимодействия:

двух кислот

двух оснований

двух кислых солей

двух кислотных оксидов

Напишите уравнения описанных реакций.

Раствор средней соли, образующейся при пропускании сер­нистого газа через раствор щёлочи, на длительное время оставили на воздухе. Твёрдое вещество, образующееся после выпаривания раст­вора, смешали с коксом и нагрели до высокой температуры. При до­бавлении к твёрдому продукту реакции соляной кислоты выделяется газ с запахом тухлых яиц. Напишите уравнения описанных реакций.

К порошкообразному веществу чёрного цвета добавили раствор разбавленной серной кислоты и нагрели. В полученный рас­твор голубого цвета приливали раствор едкого натра до прекращения выделения осадка. Осадок отфильтровали и нагрели. Продукт реак­ции нагревали в атмосфере водорода, в результате чего получилось вещество красного цвета. Напишите уравнения описанных реакций.

Красный фосфор сожгли в атмосфере хлора и к продукту реакции добавили небольшое количество (несколько капель) воды. Выделяющееся вещество растворили в избытке воды, в полученный раствор добавили железный порошок и газообразный продукт реак­ции пропустили над нагретой, окисленной до оксида двухвалентной меди, медной пластинкой. Напишите уравнения реакций описанных превращений.

Раствор хлорида железа (III) подвергли электролизу с гра­фитовыми электродами. Осадок бурого цвета, образовавшийся при электролизе, отфильтровали и растворили в растворе гидроксида нат­рия, после чего добавили такое количество серной кислоты, которое необходимо для образования прозрачного раствора. Продукт, выде­лившийся на аноде, пропустили через горячий раствор гидроксида калия. Напишите уравнения описанных реакций.

В раствор кристаллической соды добавили хлорид алюми­ния, выделившийся осадок отделили и обработали раствором едко­го натра. Полученный раствор нейтрализовали азотной кислотой, выделившийся осадок отделили и прокалили. Напишите уравнения описанных реакций.

Аммиак смешали с большим избытком воздуха, нагрели в присутствии платины и через некоторое время поглотили водой. Медная стружка, добавленная в полученный раствор, растворяется с выделением бурого газа. Напишите уравнения описанных реакций.

При добавлении раствора кислоты А к диоксиду марганца происходит выделение ядовитого газа жёлто-зелёного цвета. Пропу­стив выделившийся газ через горячий раствор едкого кали, получают вещество, которое используется при изготовлении спичек и некоторых других зажигательных составов. При термическом разложении последнего в присутствии диоксида марганца образуется соль, из ко­торой при взаимодействии с концентрированной серной кислотой можно получить исходную кислоту А, и бесцветный газ, входящий и состав атмосферного воздуха. Напишите уравнения описанных ре­акций.

Продукт взаимодействия кремния с хлором легко гидролизуется. При сплавлении твёрдого продукта гидролиза как с каусти­ческой, так и с кальцинированной содой образуется жидкое стекло. Напишите уравнения описанных реакций.

К раствору, полученному при растворении железа в горячей концентрированной соляной кислоте, прибавили едкий натр. Выде­лившийся осадок отделили, на длительное время оставили на возду­хе, после чего растворили в разбавленной соляной кислоте. Напиши­те уравнения описанных реакций.

При нагревании вещества оранжевого цвета оно разлагается; среди продуктов разложения — бесцветный газ и твёрдое вещество зелёного цвета. Выделившийся газ реагирует с литием даже при не­большом нагревании. Продукт последней реакции взаимодействует с водой, при этом выделяется газ с резким запахом, который может восстанавливать металлы, например медь, из их оксидов. Напишите уравнения описанных реакций.

Газ с запахом тухлых яиц пропустили через концентрирован­ную серную кислоту при комнатной температуре. Образовавшийся осадок отделили и обработали горячей концентрированной азотной кислотой; выделяющийся газ растворили в большом количестве воды и в полученный раствор добавили кусочек меди. Напишите уравне­ния реакций описанных превращений.

Соль, полученную при растворении железа в горячей кон­центрированной серной кислоте, обработали избытком раствора гидроксида натрия. Выпавший бурый осадок отфильтровали и прокали­ли. Полученное вещество сплавили с железом. Напишите уравнения описанных реакций.

В концентрированную серную кислоту добавили металли­ческий цинк. Образовавшуюся соль выделили, растворили в воде и в раствор добавили нитрат бария. После отделения осадка в раст­вор внесли магниевую стружку, раствор профильтровали, фильтрат выпарили и прокалили. Напишите уравнения описанных реакций.

Неизвестное вещество красного цвета нагрели в хлоре и про­дукт реакции растворили в воде. В полученный раствор добавили щё­лочь, выпавший осадок голубого цвета отфильтровали и прокалили. При нагревании продукта прокаливания, который имеет чёрный цвет, с коксом было получено исходное вещество красного цвета. Напишите уравнения описанных реакций.

Йод нагревали с избытком фосфора и продукт реакции об­работали небольшим количеством воды. Газообразный продукт реак­ции полностью нейтрализовали раствором едкого натра и добавили в полученный раствор нитрат серебра. Напишите уравнения описан­ных реакций.

Железо сожгли в хлоре. Продукт реакции растворили в воде и в раствор внесли железные опилки. Через некоторое время раствор профильтровали и в фильтрат добавили сульфид натрия. Выделив­шийся осадок отделили и обработали 20%-ной серной кислотой, по­лучив почти бесцветный раствор. Напишите уравнения описанных реакций.

Газ, выделяющийся при нагревании твёрдой поваренной соли с концентрированной серной кислотой, пропустили через раствор перманганата калия. Газообразный продукт реакции поглотили холод­ным раствором едкого натра. После добавления в полученный раствор йодоводородной кислоты появляется резкий запах и раствор приобре­тает темную окраску. Напишите уравнения описанных реакций.

Через раствор, полученный при гашении извести, пропу­стили газ, который образуется при получении негашёной извести из известняка; в результате выделяется белый осадок. При действии уксусной кислоты на полученный осадок выделяется тот же газ, ко­торый образуется при прокаливании карбоната кальция. Напишите уравнения описанных реакций.

Вещество красного цвета, которое используется в производ­стве спичек, сожгли в избытке воздуха и продукт реакции при нагревании растворили в большом количестве воды. После нейтрализации полученного раствора пищевой содой в него добавили нитрат сереб­ра. Напишите уравнения описанных реакций.

Через раствор бромида натрия пропустили газ, выделяю­щийся при взаимодействии соляной кислоты с перманганатом ка­лия. После окончания реакции раствор выпарили, остаток раство­рили в воде и подвергли электролизу с графитовыми электродами. Газообразные продукты реакции смешали друг с другом и осветили, и результате произошёл взрыв. Напишите уравнения описанных ре­акций.

Газ, образующийся при сгорании кокса, длительное время соприкасался с раскалённым углем. Продукт реакции последова­тельно пропустили через слой нагретой железной руды и негашёную известь. Напишите уравнения описанных реакций.

К нагретой концентрированной серной кислоте добавили медную стружку и выделившийся газ пропустили через раствор ед­кого натра (избыток). Продукт реакции выделили, растворили в воде и нагревали с серой, которая в результате проведения реакции раст­ворилась. В полученный раствор добавили разбавленную серную кислоту. Напишите уравнения описанных реакций.

К растворам веществ Л и Б, которые окрашивают пламя в жёлтый цвет, добавили хлороводородную кислоту: При взаимодей­ствии раствора вещества А с хлороводородной кислотой выделяется бесцветный газ с неприятным запахом, образующий осадок чёрного цвета при пропускании через раствор нитрата свинца (II). При на­гревании раствора вещества Б с хлороводородной кислотой окраска раствора сменяется с жёлтой на зелёную и выделяется ядовитый газ жёлто-зелёного цвета с характерным резким запахом. При добавле­нии к раствору вещества Б нитрата бария выпадает осадок жёлтого цвета. Напишите уравнения описанных реакций.

К пиролюзиту осторожно прибавили раствор соляной кис­лоты и выделяющийся газ пропустили в химический стакан, наполо­вину наполненный холодным раствором едкого кали. После оконча­ния реакции стакан накрыли картонкой и оставили, при этом стакан освещали солнечные лучи; через некоторое время в стакан внесли тлеющую лучинку, которая ярко вспыхнула. Напишите уравнения описанных реакций.

Осадок, полученный при взаимодействии раствора соли алюминия и щёлочи, прокалили. Продукт реакции растворили в концентрированном горячем растворе щёлочи. Через полученный раствор пропустили углекислый газ, в результате чего образовался осадок. Напишите уравнения описанных превращений.

Чёрный порошок, который образовался при длительном нагревании металла красного цвета в избытке воздуха, растворили в 10%-ной серной кислоте и получили раствор голубого цвета. В раст­вор добавили щёлочь и выпавший осадок отделили и растворили в из­бытке раствора аммиака. Напишите уравнения описанных реакций.

К твёрдому веществу, которое образуется при сжигании фос­фора в избытке хлора, добавили фосфор и смесь нагрели. Продукт реакции обработали небольшим количеством горячей воды и в по­лученный раствор добавили подкисленный серной кислотой раствор перманганата калия. Напишите уравнения описанных реакций.

Через баритовую воду пропускали углекислый газ. В полу­ченный раствор добавили гидроксид бария, продукт реакции отде­лили и растворили в ортофосфорной кислоте. Напишите уравнения описанных реакций.

Нитрат цинка прокалили, продукт реакции при нагревании обработали раствором едкого натра. Через образовавшийся раствор пропустили углекислый газ до прекращения выделения осадка, после чего обработали избытком концентрированного нашатырного спир­та, при этом осадок растворился. Напишите уравнения описанных реакций.

В двух сосудах находятся растворы неизвестных веществ. При добавлении к раствору первого вещества хлорида бария выпадает осадок белого цвета, нерастворимый в воде и кислотах. Осадок бело­го цвета выпадает также и при добавлении раствора нитрата серебра к пробе, отобранной из второго сосуда. При нагревании пробы пер­вого раствора с гидроксидом натрия выделяется газе резким запахом. При взаимодействии второго раствора с хроматом натрия выпадает осадок жёлтого цвета. Напишите уравнения описанных реакций.

Сернистый газ растворили в воде и раствор нейтрализовали, добавляя едкий натр. В образовавшийся раствор добавили перекись водорода и после окончания реакции — серную кислоту. Напишите уравнения описанных реакций.

Цинк растворили в очень разбавленной азотной кислоте, по­лученный раствор осторожно выпарили и остаток прокалили. Про­дукты реакции смешали с коксом и нагрели. Составьте уравнения реакций описанных превращений.

Вещества, выделяющиеся на катоде и аноде при электролизе раствора йодида натрия с графитовыми электродами, реагируют друг с другом. Продукт реакции взаимодействует с концентрированной серной кислотой с выделением газа, который пропустили через раст­вор гидроксида калия. Напишите уравнения описанных реакций.

Вещество, которое образуется при электролизе расплава боксита в криолите, растворяется как в растворе соляной кислоты, так и в растворе щёлочи с выделением одного и того же газа. При смешивании полученных растворов образуется объёмный осадок бе­лого цвета. Напишите уравнения описанных реакций.

К оксиду свинца (IV) при нагревании добавили концентри­рованную соляную кислоту. Выделяющийся газ пропустили через нагретый раствор едкого кали. Раствор охладили, соль кислородсо­держащей кислоты отфильтровали и высушили. При нагревании по­лученной соли с соляной кислотой выделяется ядовитый газ, а при нагревании се в присутствии диоксида марганца — газ, входящий и состав атмосферы. Напишите уравнения описанных реакций.

Бурый осадок, полученный при взаимодействии сульфи­та натрия с водным раствором перманганата калия, отфильтровали и обработали концентрированной серной кислотой. Выделяющийся газ при нагревании реагирует с алюминием, а образующееся веще­ство — с раствором соляной кислоты. Напишите уравнения реакций описанных превращений.

Кальций нагрели в атмосфере водорода. Продукт реакции обработали водой, выделяющийся газ пропустили над нагретым ок­сидом цинка, а в раствор добавили кальцинированную соду. Напи­шите уравнения описанных реакций.

Нитрат серебра прокалили и твёрдый продукт реакции на­грели в кислороде. Образовавшееся вещество растворяется в избытке концентрированного нашатырного спирта. При пропускании через полученный раствор сероводорода образуется осадок чёрного цвета. Напишите уравнения описанных реакций.

Твёрдое вещество, которое образуется при нагревании фос­фора и пятихлористого фосфора, растворили в большом количестве воды. Часть полученного раствора добавили в подкисленный серной кислотой раствор перманганата калия, при этом последний обесцве­тился. Напишите уравнения описанных реакций.

Несколько гранул цинка внесли в сосуд с концентрирован­ной серной кислотой. Выделяющийся газ пропустили через раствор ацетата свинца (II), осадок отделили, подвергли обжигу и образовав­шийся газ ввели во взаимодействие с водным раствором пермангана­та калия. Напишите уравнения описанных реакций.

Несколько гранул цинка растворили при нагревании в раст­воре едкого натра. В полученный раствор небольшими порциями добавляли азотную кислоту до образования осадка. Осадок отдели­ли, растворили в разбавленной азотной кислоте, раствор осторожно выпарили и остаток прокалили. Напишите уравнения описанных реакций.

Газ, выделяющийся при растворении меди в горячей концен­трированной азотной кислоте, может взаимодействовать как с газом, выделяющимся при обработке меди горячей концентрированной серной кислотой, так и с медью. Напишите уравнения описанных реакций.

Раствор хлорида железа (III) подвергли электролизу с графи­товыми электродами. Выделившийся осадок бурого цвета (побочный продукт электролиза) отфильтровали, прокалили и сплавили с ве­ществом, образовавшимся на катоде. Другое, также выделившееся на катоде, вещество ввели в реакцию с продуктом, выделившим­ся при электролизе на аноде; реакция протекает при освещении и со взрывом. Напишите уравнения описанных реакций.

Нерастворимую в воде соль белого цвета, которая встреча­ется в природе в виде широко используемого в строительстве и архи­тектуре минерала, прокалили при 1000°С. К твёрдому остатку после охлаждения добавили воду и через полученный раствор пропустили газообразный продукт реакции разложения, в результате образовался осадок белого цвета, который растворился при дальнейшем пропу­скании газа. Напишите уравнения описанных реакций.

Главная

Итоги демонстрационного экзамена

студентов БПОУ ВО «Сокольский ЛПТ»

по компетенции «Поварское дело»

Демонстрационный экзамен по стандартам Ворлдскиллс — эта форма экзамена, которая предполагает оценку компетенций путём наблюдения за выполнением трудовых действий в условиях, приближенных к производственным.

С 08 по 10 июня 2021 года студенты БПОУ ВО «Сокольский ЛПТ» успешно сдали демонстрационный экзамен по компетенции №34 «Поварское дело» на базе Центра проведения демонстрационного экзамена БПОУ ВО «Вологодский колледж сервиса».

17 студентам группы №42-17 по профессии 43.01.09 Повар, кондитер было предложено приготовить блюда в соответствии со стандартами WorldSkills Russia.

За выполнением заданий следили: председатель государственной экзаменационной комиссии: Дарёшина Надежда Александровна, зав. производством ресторана «Прага» ИП Макаренкова М.Н. г. Сокол; заместитель председателя государственной экзаменационной комиссии: Шилова Ольга Анатольевна, заведующий отделением БПОУ ВО «Сокольский ЛПТ», члены комиссии: Алексикова Людмила Петровна, преподаватель БПОУ ВО «Сокольский ЛПТ», Халуева Александра Ивановна, мастер производственного обучения БПОУ ВО «Сокольский ЛПТ», Маслова Надежда Леонидовна, преподаватель БПОУ ВО «Сокольский ЛПТ».

Независимую экспертную оценку выполнения заданий осуществляют эксперты, прошедшие обучение по программе повышения квалификации «Эксперт демонстрационного экзкамена» в Академии Союза «Молодые профессионалы» (Ворлдскиллс Россия) члены экспертной группы государственной экзаменационной комиссии: главный эксперт: Прокофьева Анастасия Сергеевна, БПОУ ВО «Вологодский колледж сервиса»; эксперты, с правом участия в оценке демонстрационного экзамена по стандартам WorldSkills Черяпкина Светлана Александровна, заведующая столовой БПОУ ВО «Вологодский колледж сервиса»; Русанова Светлана Александровна, мастер п/о БПОУ ВО «Вологодский колледж сервиса»; Ефанова Анна Александровна, преподаватель БПОУ ВО «Вологодский колледж сервиса»; Шумихина Ксения Андреевна, мастер п/о БПОУ ВО «Вологодский колледж сервиса»; Кезимова Татьяна Викторовна, преподаватель БПОУ ВО «Вологодский колледж сервиса»; Веселова Ирина Сергеевна, преподаватель БПОУ ВО «Вологодский технический колледж.

Для проведения экзамена ЦПДЭ был предоставлен учебный кулинарный цех (оснащенный в соответствии с требованиями инфраструктурной листа аналогичным оборудованием для 6 рабочих мест), комнаты для работы экспертов, участников, волонтеров, складские помещения для хранения тулбоксов продуктов и инвентаря, выставочная зона, для демонстрации блюд.

Волонтерами выступили студенты нашего техникума. На площадке находились 6 волонтеров, которые помогали участникам мыть посуду и инвентарь, осуществляли совместно с участниками, подачу блюд для экспертной оценки, помогали в организации рабочих мест для следующей смены.

Участники Демонстрационного экзамена продемонстрировали различный уровень практической подготовки. Многие участники умеют спланировать и организовать рабочий процесс, расставляют приоритеты при выполнении работы, могут ориентироваться в производственном процессе, имеют представление о работе на новых и дополнительных видах оборудования. Участники работали уверенно, без лишней суеты. Подача блюд была осуществлена вовремя, согласно регламента.

В целом, во время сдачи экзамена у студентов не возникло ни каких проблем и внештатных ситуаций. Все обучающиеся справились с заданием в полном объеме.

Из 17 студентов сдали экзамен на «4» — 16 чел., «3»- 1 чел.

По результатам демонстрационного экзамена студенты получат Паспорта компетенций (Skills Passport) – документы, подтверждающие овладение навыками по стандартам Ворлдскиллс Россия по компетенции «Поварское дело».

Поздравляем студентов и наставника мастера производственного обучения Халуеву Александру Ивановну. Желаем ребятам успехов в избранной профессии!

Благодарим наших коллег из БПОУ ВО «Вологодский колледж сервиса» и БПОУ ВО «Вологодский технический колледж» за сотрудничество!

Barite — обзор | ScienceDirect Topics

Обогащение барита, флюорита и бастнезита из месторождения Донг Пао во Вьетнаме

Эта руда представляет собой сильно выветрившуюся руду, более 30% бастнезита содержится во фракции –7 мкм. Основными минералами-хозяевами, присутствующими в этой руде, являются барит и флюорит. В таблице 24.6 показаны химические анализы руды, использованные в различных исследованиях.

Таблица 24.6. Химический анализ руды Донг Пао

Элемент Анализы (%)
Всего REO 8.72
Церий (Ce 2 O 3 ) 3,76
Лантан (La 2 O 3 ) 3,18
Барит (BaSO 4 ) 62,5
Флюорит (CaF 2 ) 5,54
Кремнезем (SiO 2 ) 8,85
Глинозем (Al 2 O 3 ) 0,97
Железо (Fe 2 O 3 ) 2.69
Кальций (CaO) 0,15
Натрий (Na 2 O) 0,54
Калий (K 2 O) 0,11
Титан (TiO 2 ) 0,09
Фосфор (P 2 O 5 ) 0,13
Марганец (MnO) 0,64
Хром (Cr 2 O 3 ) 0.22
Ванадий (V 2 O 5 ) 0,03
LOI 10,6

Рудное месторождение находится в провинции Лай Чан во Вьетнаме и было разработано Sumitomo Металлургическая компания (Япония).

Поскольку в этой руде было много барита и флюорита, прямая флотация бастнезита из руды была невозможна. Следует отметить, что флюорит имеет такие же флотационные свойства, как и бастнезит, и депрессия флюорита во время флотации бастнезита затруднена.

Была проведена обширная исследовательская работа [7] по этой руде, направленная на разработку промышленного процесса переработки, который позволил бы получить высококачественный концентрат REO. В результате были разработаны уникальная технологическая схема и схема реагентов.

Технологическая схема, разработанная для обогащения руды Донг Пао, включает последовательную барит-флюорит-бастнезитовую флотацию. Технологическая схема представлена ​​на рисунке 24.8.

Рисунок 24.8. Разработана технологическая схема обогащения руды Донг Пао.

Перед измельчением руда была промыта и очищена от шлама. Мелочь от промывки содержала более 30% всего бастнезита, присутствующего в руде.

Измельченная руда была сначала подвергнута флотации баритом, а затем флотацией флюорита. Путем плавания барита и флюорита перед бастнезитом около 70% общего веса было удалено из бастнезитового флотационного сырья. Сырье для флотации бастнезита было увеличено с 8,5% REO до примерно 30% REO.

Схема реагентов, разработанная в ходе обширных лабораторных испытаний, представлена ​​в Таблице 24.7. Эта схема реагентов уникальна тем, что коллектор и количество задействованных депрессантов состоят из ряда химикатов, которые обеспечивают улучшенную селективность во время последовательной флотации барита и флюорита из бастнезита.

Таблица 24.7. Схема реагентов, разработанная для обогащения руды Донг Пао

9 0019 —

Реагент Добавки (г / т)
BaSO 4 контур CaF 2 контур REO контур
Ro Cl Ro Cl Ro Cl
Депрессанты и модификаторы
Na 2 SiO 3 2500 1200 1100
BaCl 2 500 400
NaF 300 400
Al 2 (SO 4 ) 3 600 400
MESB 20 200
Na 2 CO 3 4000 1400
Лимонная кислота 1000 3500
MM4 1000 1300
Коллекторы
SR82 850
AKF2 300
KV3 900 200
Мазут 200

Для флотации барита силикат натрия использовался в качестве депрессора, а хлорит бария — в качестве активатора барита.Коллектор барита SR82 состоял из смеси сульфоната нефти, алкилсульфата натрия и сукцинамата. Сборщик был избирательным как в отношении флюорита, так и бастнезита. Более 96% барита было извлечено в относительно высококачественный концентрат.

Во время флотации флюорита использовалась смесь квебрахо и лигнина сульфоната (MESB) с коллектором, состоящим из смеси олеиновой кислоты и сложного эфира фосфорной кислоты. Коллекторы, используемые для флотации бастнезита, включали жирную кислоту таллового масла, модифицированную тремя этилентетрааминами.Депрессант MM4 представлял собой смесь сульфоната лигнина с молекулярной массой от 9000 до 20000.

Результаты, полученные при непрерывных испытаниях в режиме блокировки, приведены в Таблице 24.8. Основным загрязнителем бастнезитового концентрата был флюорит. Полная флотация флюорита невозможна без больших потерь бастнезита во флюоритовом концентрате.

Таблица 24.8. Результаты испытаний с непрерывным замкнутым циклом

900

Продукт Вес (%) Анализы (%)% Распределение
BaSO 4 CaF 2 REO BaSO 4 CaF 2 REO
BaSO 4 Cl концентрат 62.83 95,8 0,67 0,61 96,8 7,3 4,6
CaF 2 Cl концентрат 7,36 3,11 44,4 7,57 0,4 56,3 6,7
Концентрат РЭО Cl 13,72 8,55 14,8 45,9 1,9 35,0 75,6
РЭО комбинированный хвост 16.09 3,47 0,50 6,79 0,9 1,4 13,1
Головка (расч.) 100,00 62,2 5,80 8,33 100,0 100,0 100,0
BaSO 4 Концентрат Cl 62,83 95,8 0,67 0,61 97,2 7,1 4,7
CaF 2 Концентрат Cl 7.36 3,11 44,4 7,57 1,4 54,9 6,7
Концентрат REO Cl 12,97 6,55 16,6 48,4 1,4 36,210 76,1
РЭО комбинированное хвостовое оперение 16,84 4,00 0,61 6,13 1,1 1,7 12,5
Головка (расч.)100.00 62,0 5,94 8,25 100,0 100,0 100,0

Барит — обзор | ScienceDirect Topics

Хелатирующие агенты

Незначительные количества хелатирующих агентов, таких как ЭДТА, лимонная кислота или глюконовая кислота, могут снизить эффективность ингибиторов образования отложений [15]. Концентрация ионов кальция и ионов магния влияет на ингибирование сульфата бария [52]. Были изучены пентафосфонатные, гексафосфонатные, фосфино-поли (карбоновые кислоты) (PPCA) соли и ингибиторы образования отложений PVS.Нитчатые хелатирующие агенты, приведенные в таблице 11.3, также стабилизируют покрытие инкапсулированных составов [51]. Некоторые хелатирующие агенты на основе иминокислот показаны на рис. 11.1.

Таблица 11.3. Хелатирующие агенты для стабилизации покрытий [51].

N 9019 3-HPIDA

90 (2-гидроксипропил) имино- N , N -диауксусная кислота

1 — N , N -диуксусная кислота

1. Хелатирующие агенты.

EDTA

Обычный растворитель накипи для баритовой накипи состоит из концентрированного раствора карбоната калия, гидроксида калия и калиевой соли EDTA. С другой стороны, карбонатные отложения можно растворить с помощью простых минеральных кислот, таких как [53]. Кроме того, поверхностно-активные вещества полезны для регулирования вязкости жидкостей. В качестве поверхностно-активного вещества предложен N -эруцил- N , N -бис (2-гидроксиэтил) — N -метиламмоний хлорид [53].

Эти поверхностно-активные вещества могут образовывать червеобразные мицеллы при смешивании с рассолом. Структура мицелл вносит значительный вклад в вязкоупругость жидкости, и вязкоупругость быстро теряется, когда жидкость контактирует с углеводородами, что приводит к изменению структуры мицелл или их распаду.

Разница в вязкости жидкости при контакте с углеводородами и водой позволяет выборочно размещать средство для очистки от накипи. В результате окалина может быть предпочтительно удалена из зон, несущих углеводороды.Это может привести к интенсификации добычи углеводородов без существенного увеличения обводненности добываемых флюидов [53].

ЭДТА можно регенерировать подходящим способом. Следующие реакции иллюстрируют растворение и последующее выделение отложений сульфата бария и регенерацию ЭДТА в упрощенной форме [54]:

(11.1)

Фосфонаты

Предыдущие исследования убедительно показали, что разновидности ингибиторов на основе метиленфосфоновой кислоты, такие как такие как пентафосфонат и гексафосфонат, значительно менее термически стабильны, чем полимерные частицы, такие как PVS и частицы S-Co.Таким образом, разновидности на основе фосфонатов были признаны менее пригодными для использования в высокотемпературных резервуарных системах.

Однако более поздние исследования ингибиторов с соединениями на основе различных аминов метиленфосфоновых кислот показали, что некоторые соединения термически стабильны при температурах, превышающих 160 ° C [55].

Ряд ингибиторов образования отложений на основе фосфонатов подвергали термическому старению при 160 ° C. После старения ингибиторы образования отложений все еще были способны предотвращать образование отложений карбоната в динамических испытаниях.Однако эффективность некоторых фосфонатных соединений в отношении сульфатной накипи снизилась из-за термического старения [56].

Этерифицированные фосфоно или фосфинокислоты с длинноцепочечным спиртом эффективны в качестве маслорастворимых ингибиторов образования отложений, а также в качестве ингибиторов парафина или асфальтенов или диспергаторов при добыче нефти. Сложные эфиры могут быть получены азеотропной конденсацией фосфинокислот со спиртом. Альтернативно, сложные эфиры получают теломеризацией сложного эфира ненасыщенной карбоновой кислоты с фосфитным или гипофосфитным телогеном [57].

Напротив, лабораторные исследования продемонстрировали явный потенциал для значительного увеличения продолжительности лечения за счет перехода с фосфонатного на ингибитор образования отложений на основе винилсульфонатного сополимера [5].

Твердый инкапсулированный ингибитор образования отложений (поли (фосфат) кальция и магния) был разработан и всесторонне испытан для использования при гидроразрыве пласта [49,58,59]. Ингибитор совместим со сшитыми боратом и цирконием жидкостями для гидроразрыва и вспененными жидкостями из-за покрытия.

Покрытие оказывает кратковременное влияние на профиль скорости высвобождения. Состав твердого производного оказывает наибольшее влияние на его долгосрочный профиль скорости высвобождения.

Сульфаты щелочноземельных металлов

В исследованиях растворения барита с использованием хелатирующих агентов на основе ЭДТА и диэтилентриаминпентауксусной кислоты было подтверждено, что присутствие добавок дикарбоновой кислоты, таких как ионы оксалата, улучшает характеристики хелатирующих агентов. Однако другие родственные добавки, такие как малонат и сукцинат, снижают эффективность.

Оксалат-ионы катализируют поверхностную реакцию комплексообразования между хелатирующим агентом и поверхностью барита путем образования двухлигандного поверхностного комплекса. Неблагоприятный эффект, наблюдаемый для других дикарбоновых кислот, как полагают, возникает из-за стерических эффектов, которые предотвращают образование двухлигандного поверхностного комплекса.

В расширенных исследованиях других отложений, связанных с баритом, таких как целестит (), гипс () и ангидрит (), было обнаружено, что растворители накипи, оптимизированные по своей эффективности против одного типа отложений, таких как барит, могут не быть наиболее эффективным против других шкал [60].

Более чем красивая вспышка в небе

Июль 2005 г.

Введение

Многие из ослепительных вспышек цвета, которые освещают ночное небо четвертого июля, приносят вам комплименты химии хлора. Хлорид бария, BaCl2, ответственный за сверкающие зеленые всплески, является основным хлорным соединением в этом месяце. Но хлорид бария — это гораздо больше, чем красивая вспышка в небе.

Производство хлорида бария

Фотография барита из Института минеральной информации, любезно предоставлена ​​Смитсоновским институтом.

Химический элемент барий — тяжелый, мягкий, серебристо-белый металл, который, как и элемент хлор, всегда встречается в природе в сочетании. Благодаря своей реакционной способности барий и хлор естественным образом образуют соединения.Наиболее распространенной формой бария в природе является соль сульфата бария, BaSO4, также известная как минеральный барит. Это также главный
руда бария. Хлорид бария, другая соль бария, может быть получен путем взаимодействия сульфата бария с соляной кислотой, HCl:

.

Обратите внимание, что барий и водород переключаются между «химическими партнерами» между реагентами и продуктами в уравнении.Этот тип реакции известен как реакция двойной замены и может быть представлен в забавной мультяшной форме как:

Хлорид бария: сульфатный детектив

Хлорид бария выполняет несколько важных функций, но одна из них хорошо знакома химикам — это роль «детектива сульфатов».«Сульфат, SO4 -2 — это отрицательно заряженная химическая единица, состоящая из атома серы, связанного с четырьмя атомами кислорода. Эти отрицательно заряженные единицы известны как отрицательно заряженные.
ионы или анионы.

Ключом к роли хлорида бария в отслеживании сульфатов является тот простой факт, что хлорид бария растворяется в воде, в то время как сульфат бария практически не растворяется в воде.Представленный жидкостью на водной основе неизвестного химического состава, химик может добавить в раствор хлорид бария и наблюдать за появлением густого белого «осадка» сульфата бария, указывающего на присутствие сульфата.

Хотя термин «осадок» звучит так, как будто в химическом стакане может идти дождь, осадок — это просто твердое вещество, которое образуется в жидком растворе в результате химической реакции. Поскольку он практически не растворяется в жидкости раствора, осадок становится видимым.Химический осадок сначала превращает жидкость в мутную, и в конечном итоге из-за силы тяжести мутность исчезает, когда твердое вещество оседает на дно емкости.

Другие роли хлорида бария

Помимо того, что хлорид бария является важным химическим индикатором, он может быть исходным химическим веществом для производства бария и хлора. Он также используется в производстве алюминиевых сплавов, в пигментах и ​​красителях и в качестве смягчителя воды.

Использование физических свойств бария

Барий, название элемента с атомным номером 56 на
Периодическая таблица элементов происходит от греческого слова «барыс», что означает «тяжелый».«Барий — относительно тяжелый элемент, потому что в его ядре 56 протонов (по сравнению, например, с хлором, у которого их всего 17). Такое же количество электронов уравновешивает электрический заряд.

Соединения бария тяжелые и плотные. Сульфат бария, например, используется при рентгеновских исследованиях для визуализации пищеварительной системы человека.Пациент выпивает «молочный коктейль с барием», суспензию сульфата бария, которая заполняет пищеварительный тракт и создает препятствие (в силу множества электронов) для рентгеновских лучей. Блокируя путь рентгеновских лучей, барий очерчивает тракт, позволяя врачам определять проблемные области.

Высокая плотность сульфата бария также играет важную роль при разведке нефти.Его добавляют в «буровой раствор», суспензию, добавляемую в буровые скважины, чтобы контролировать давление на подземные запасы нефти и газа. Сульфат бария помогает снизить риск выброса нефти и газа из буровой скважины. Прорыв может привести к пожару или взрыву.

Дополнительная деятельность:

  1. Следующая химическая реакция является примером единственной реакции замещения с участием калия (K), хлора (Cl) и йода (I):

    Чем эта реакция отличается от реакции двойного замещения? Сделайте карикатурное изображение одной реакции замещения.

  2. Составьте список соединений хлоридов металлов, которые используются в фейерверках, и их цветов.

Чтобы просмотреть список предыдущих функций «Хлорное соединение месяца», щелкните
здесь.

Baryte | Музей наук о Земле

Статьи о камнях и минералах

Кэти Фейк

Имя Барит происходит от греческого слова «Барыс», что означает «тяжелый». Фактически, альтернативное название барита — «тяжелый лонжерон». Он имеет необычно высокий удельный вес для неметаллического минерала.

Барит. Cole Co., штат Висконсин. Собрание музея наук о Земле Университета Ватерлоо.

Барит химически инертен и нерастворим, но он мягкий, его твердость составляет всего 3–3,5 по шкале твердости Мооса. Его можно найти практически любого цвета в зависимости от примесей, захваченных в структуре, хотя обычно он белый или бесцветный. Обычно он образует большие пластинчатые кристаллы, агрегаты в форме розетки или расходящиеся пластинки, известные как гребенчатые бариты.

Среда образования:

Большая часть барита добывается из слоев осадочной породы, образовавшейся при осаждении барита на дно океанского дна.Некоторые небольшие рудники используют барит из жил, которые образовались при осаждении сульфата бария из горячих подземных вод. В некоторых случаях барит является побочным продуктом добычи свинца, цинка, серебра или других металлических руд, особенно в гидротермальных жилах.

Баритовый флюорит. Бланшар Майн Н.М., Собрание Музея наук о Земле Университета Ватерлоо.

Проблемы со здоровьем:

Хотя барит содержит тяжелый металл барий, он не влияет на здоровье человека.Барий не радиоактивен, чрезвычайно нерастворим и не может легко усваиваться организмом человека.

Использует:

  • Буровые растворы потребляют около 85% мирового потребления барита
  • Как руда элементарного бария
  • Краска среднего и высшего сорта
  • Инженерный пластик
  • Химические вещества для приготовления лекарственных препаратов
  • Резина
  • Производство бумаги
  • Керамика
  • Косметика
  • Раскислитель меди
  • Baryte из-за высокой плотности делает его непрозрачным для рентгеновских лучей.Если его дать пациенту в виде питья или клизмы, с его помощью можно визуализировать форму внутренних органов с помощью рентгеновского излучения

Барит. Рудник Марморатон, Мармора, Онтарио. Собрание Музея наук о Земле Университета Ватерлоо.

населенных пунктов:

Месторождения барита известны во всем мире, но многие из них нерентабельны. Его добывают примерно в 20 странах мира; с Китаем как крупнейшим производителем.

Ссылки:

Rock Currier — Очистка кварца

Очистка кварца
Те же методы, описанные ниже, также можно использовать для очистки других твердых прочных минералов, таких как берилл, шпинель, турмалин, хризоберилл и т. Д.

Вот несколько фотографий образцов кристаллов кварца, только что из шахты в Арканзасе, которые необходимо очистить. Это типичные примеры образцов кварца, окрашенного железом, которые можно найти во многих частях мира.

Оскорбительное «окрашивание железом» на вышеуказанных образцах — это мелкозернистые минералы железа, которые обычно связаны с кристаллами кварца, когда их выкапывают из земли. Кристаллы кварца — довольно требовательные покупатели, и их можно довольно агрессивно чистить как механическими, так и химическими способами.Под этим я подразумеваю, что вы можете обработать большинство из них жесткой щеткой и чистящим порошком. Кроме того, вы можете поместить их в любую сильную кислоту, не травляя их или не делая естественного блеска, который многие кристаллы кварца имеют на своей поверхности (исключением является фтористоводород, который обсуждается ниже). Вы даже можете использовать щетки со стальной или алюминиевой щетиной, если хотите, поскольку кварц тверже этих металлов. Однако вы можете стереть часть металла с кристаллов кварца, которые вам, возможно, придется очистить.

Есть ли исключения из этого? Да, если кристаллы кварца неплотно соединены вместе, как кластер слегка сросшихся кристаллов кварца, изображенный ниже из Херкимера, Нью-Йорк. Грубая очистка кристаллов таких образцов кристаллов кварца может привести к тому, что кристаллы отделятся друг от друга там, где они прикреплены друг к другу.

Если кристаллы кварца тонкие и призматические, подобные этим, и сросшиеся, как два образца ниже, из карьера Джеффри, Джеффри, Пуласки Ко.В Арканзасе, США или в Уароне, Перу, грубое обращение может привести к поломке некоторых.

Если кварц относится к разновидности «водный кварц» (фенстер / жакари / элестиал), кристаллы могут содержать жидкие включения с пузырьками. Тепловое воздействие на них, например, при отрицательных температурах, или их бросание в горячую воду может привести к растрескиванию.

Кристаллы кварца из одних мест более чувствительны к тепловому удару, чем кристаллы из других. Кварц из Херкимера, Нью-Йорк, похоже, является одним из них.Когда их извлекают из земли, некоторые коллекционеры научились быстро заворачивать их, грязь и все такое, и помещать в ледяной ящик, чтобы они могли привыкнуть к жизни вне кармана, в котором они образовались. Затем, через день или около того, они тщательно очищаются водой той же температуры, что и кварц. Невыполнение этого требования может привести к разрушению кристаллов или возникновению внутренних трещин в некоторых из часто прекрасно чистых «херков». Кристаллы из некоторых населенных пунктов округа Херкимер кажутся более чувствительными к тепловому шоку, чем другие.Это исключения. Большинство кристаллов кварца устойчиво к значительным перепадам температуры. Тем не менее, об этом следует подумать при чистке кристаллов кварца. Если вы очистите много кварца в определенной местности, вы быстро узнаете, что они могут терпеть, а что нет. Наконец, ваши возможности очистки могут быть ограничены, если ваши образцы кварца связаны с хрупкими минералами. Эти хрупкие минералы могут быть повреждены некоторыми химическими веществами, которые в противном случае могли бы использоваться для их очистки.Типичным примером может быть кварц, связанный с кальцитом. Если вы погрузите образец в кислоту, кальцит растворится в кислоте с выделением большого количества пузырьков углекислого газа. Помимо этих исключений, вы можете свободно иметь их.

Имейте в виду, что в большинстве случаев «грязь», которую вы хотите очистить со своего кристалла (ов) кварца, также является минералом (ами). Таким образом, когда вы «очищаете» образцы, вы фактически удаляете минералы из образца, помещенного туда природой. Некоторые коллекционеры и минералоги считают, что, удаляя эту «грязь», вы уничтожаете данные, которые лучше сохранить для будущих поколений.Исторически сложилось так, что эта грязь не всегда остается нетронутой из-за того, что правильная очистка приносит прибыль. Если вас это беспокоит, вы можете оставить часть первоначальной «грязи» на образце в таком месте, где она не будет отвлекать от его «сдерживающей привлекательности». К счастью, кварц — один из самых распространенных минералов на Земле, и на каждый образец, который вы «разрушаете» очисткой, остается еще много другого, что нужно изучить.

В течение нескольких лет в течение 1980-х годов я и несколько сотрудников управляли небольшой кварцевой шахтой в штате Минас-Жерайс в Бразилии на холмах недалеко от городка Жоакуам Фелисио.Эта шахта находилась не в большом гранитном регионе штата, который также производит множество мелких кристаллов кварца, а скорее в той части, которая составляла часть обширной формации песчаника Итаколоми. Это была одна из многих сотен небольших шахт, которые были запущены во время Второй мировой войны для производства кварца для военных нужд. Он производил в основном одиночные кристаллы кварца длиной менее шести дюймов, но многие из них содержали красивые зеленые фантомы.

Большинство кристаллов были не очень блестящими и имели некоторые повреждения.Мы обнаружили, что способ их успешного продвижения на рынке заключался в шлифовании поверхностей с помощью быстро вращающейся шлифовальной ленты с последующей полировкой с помощью войлочного полировального круга и полировального порошка из оксида олова. Они хорошо продавались. У меня было несколько коллекционеров, которые просили меня принести некоторые из натуральных неполированных кристаллов, что я и сделал, но даже когда я оценил их меньше, чем полированные кристаллы сопоставимого качества, они не очень хорошо продавались. Наконец, я просто сказал нашему цеху по огранке и полировке их всех отполировать, хотя некоторые сохранил в их естественном состоянии.Итак, я полагаю, что урок, который я усвоил, состоит в том, что на самом деле очень немногие люди озабочены сохранением естественной «грязи» или естественным травлением кристаллов кварца. Но ради будущих поколений вы должны убедиться, что все собранные вами образцы помечены названием шахты, штатом / провинцией и страной. Поскольку существует так много разных месторасположений кварца и часто невозможно узнать местонахождение образца, просто взглянув на него, вам действительно следует наклеить этикетку прямо на образец, которая указывает местонахождение.Также не помешает сохранить некоторые в их естественном состоянии.

Чаще всего человек, желающий очистить кристаллы кварца, пытается удалить коричневые «пятна железа» или белый налет с кристаллов. Кристаллы кварца из пегматитовых сред часто покрыты мелкозернистой слюдой или различными глинистыми минералами. Кварц из осадочных сред, таких как песчаники из Арканзаса, США или из песчаника в Минас-Жерайс, Бразилия, часто «окрашен железом». В этих и многих других примерах кварц покрыт или окрашен другими минералами, которые ухудшают красивую блестящую, сверкающую поверхность кристаллов.Те из нас, у кого есть практические наклонности, надеются, что прямо под оскорбительной грязью скрываются красивые блестящие кристаллы кварца. В большинстве случаев вам не нужно точно знать, что это за минералы, чтобы их очистить. Однако, если вы можете узнать, что такое минерал (ы), эта информация иногда может помочь вам узнать, как его очистить. Часто эти мелкозернистые «железные пятна» или белые покрытия нелегко охарактеризовать минералогически, и не стоит прилагать усилия для их анализа.

Выбор метода уборки обычно зависит от ваших финансов и места проживания.Это определит доступные вам инструменты и химические вещества. Я являюсь крупным оптовым продавцом минералов в Лос-Анджелесе, штат Калифорния, и имею легкий доступ к широкому спектру чистящих инструментов и химикатов. За прошедшие годы я испробовал большинство различных методов очистки, описанных в этой статье. Мы рассмотрим первую очистку механическими методами, прежде чем перейдем к химическим веществам.

1. Методы механической очистки кристаллов кварца
В определенной степени ваш метод очистки будет зависеть от того, сколько кварца вы должны очистить.Если вы работаете на кварцевой шахте в Арканзасе или выкапываете аметист из разложившейся базальтовой почвы недалеко от Артигаса, Уругвай, вам, вероятно, придется почистить пикапы, полные кварца. В этом случае первое, что вам нужно сделать, это как можно больше вымыть их водой. Если ваши образцы сильно покрыты грязью, например, аметист, выкопанный из базальтовой почвы возле Артигаса, лучший способ начать — разместить образцы на земле (предпочтительнее бетон или асфальт) или, возможно, на прямоугольные сетки, прибитые к деревянным каркасам 2х4 фута. .Облейте их водой, используя как можно большее давление. Вода из шланга — это нормально, но лучше из электрической или газовой мойки высокого давления. Не нажимайте слишком сильно, иначе вы повредите их, столкнув их друг с другом, или заставите их катать их по земле, заставляя их скалываться и треснуть. Я слышал о некоторых людях, у которых нет машины для мытья под давлением, которые брали свои образцы в одну из автомойок с оплатой монетами и использовали там мойки высокого давления для очистки своих образцов.Если на образцах много густой грязи, после первой стирки их оставляют просохнуть. Это обычно приводит к растрескиванию и усадке оставшейся грязи и позволяет при последующей промывке удалить больше грязи. Этот процесс повторяется до тех пор, пока не станет ясно, что повторные циклы стирки и сушки будут непродуктивными. Если вам нужно очистить только несколько образцов, просто потрите их щеткой с мылом и водой или воспользуйтесь одним из небольших тканевых пистолетов для очистки под высоким давлением, описанных ниже, чтобы счистить как можно больше грязи.

После того, как вы удалили как можно больше крупной грязи / грязи и почувствуете, что образец (образцы) нуждаются в дальнейшей очистке, внимательно осмотрите его и поцарапайте любую оставшуюся «грязь» лезвием ножа. Посмотрите, сможете ли вы удалить оскорбительный материал. Если нужно, используйте острие лезвия. Не беспокойтесь о том, чтобы повредить кристалл кварца, поскольку он намного тверже, чем сталь вашего ножа. Вместо этого беспокойтесь о том, что лезвие соскользнет и порежет вашу руку. В идеале вы должны делать это под бинокулярным микроскопом в отраженном свете, но также могут подойти увеличительное стекло или острые глаза.Вы делаете это, чтобы точно видеть, что происходит, когда вы царапаете поверхность кристаллов, и действительно ли вы делаете какие-либо успехи в удалении проблемного покрытия. Не бойтесь по-настоящему добиваться этого. Если вы вообще можете очистить любую часть поверхности, даже если это только крошечный участок, есть большая вероятность, что вы сможете очистить кварц механически. Если вы не можете очистить какую-либо поверхность, вероятно, поверхность кварца травмирована естественным образом, или проблемный материал врастает в поверхность кристалла или находится прямо под поверхностью.Если материал, который вы хотите удалить, срастается с поверхностью кристалла или находится чуть ниже него, вы утонули, и вы мало что можете сделать, чтобы улучшить образец, кроме шлифовки поверхности и полировки с использованием обычных лапидарных процедур. Эти гранильные операции обычно трудозатратны, и вы не можете просто «отполировать» кристаллы и заставить их сиять, как это делается с латунью или медью. Я не знаю химического метода, который заставил бы тусклый кристалл кварца сиять. Ну погоди! Я не должен так говорить.Раньше я видел довольно непривлекательные кристаллы кварца, которые помещали в большие нагретые автоклавы из нержавеющей стали, наполненные концентрированным раствором щелочи, которые используются для выращивания синтетического кварца, а затем поверх этих кристаллов выращивали новый кварц, и они выглядели довольно красиво. Но это единственное исключение, о котором я знаю. Если вы не можете сделать вмятину на том, что вы хотите удалить острием лезвия ножа, возможно, еще остался крошечный осколок надежды, но я расскажу об этом ниже.Это будет для настоящих приверженцев, которые кое-что знают о химических веществах и о том, как их безопасно использовать.

2. Мыло и вода Для очистки кварцевых образцов первым хорошим шагом является очистка одного или двух из них водой с мылом. По возможности используйте жидкое моющее мыло, а не какую-либо другую теплую воду. Это удалит любую легко удаляемую грязь и часто может дать вам представление о том, что делать дальше. Если в ваше моющее средство добавлены духи или лимонный аромат, ваши образцы будут иметь приятный запах.Часто опытные чистильщики пропускают этот шаг и приступают к струйной обработке своих образцов водой под высоким давлением с использованием тканевых пистолетов, чтобы посмотреть, удалит ли это вредное вещество. Самый простой способ сделать это — купить один из маленьких портативных пистолетов для чистки, которые называются пистолетами для чистки тканей. Они обычно используются в химической чистке для удаления пятен с ткани. Вы можете купить их менее чем за 75 долларов. Если вы погуглите «пистолет для точечной очистки», вы найдете много таких, предлагаемых в продаже. Эти маленькие «зуммеры» стоили несколько сотен долларов, но многие дилеры покупали их, потому что они прекрасно очищали образцы и приносили дилерам такие большие деньги.Часто опытные коллекционеры и дилеры все еще часто выбирают старые образцы, которые не были хорошо очищены, потому что грязь была в трещинах, и их бывшие владельцы не могли хорошо очистить их водой с мылом и щеткой. Часто пять минут или меньше работы над ними с одним из этих новых маленьких тканевых пистолетов увеличивают стоимость прекрасного старого образца на сотни или даже тысячи долларов. Внешний вид и доступность этих маленьких «пушек» для чистки были революционными для тех, кто хочет чистить образцы.Его влияние на очищающие минералы похоже на разницу между копированием книги вручную с использованием перо и чернил и печатью на лазерном принтере.

Эти маленькие «ружья» имеют короткий ствол. Чуть ниже находится лампочка, похожая на пластиковый резервуар, который пользователь периодически откручивает и наполняет водой (горячей и мыльной, если хотите). Когда вы нажимаете на спусковой крючок этих удобных маленьких устройств, маленький поршень с пружинным приводом быстро раскачивается взад и вперед и выталкивает из сопла небольшие, но мощные струи воды.Он выглядит непрерывным, но на самом деле он прерывистый. Если вы поместите палец прямо перед соплом, поток часто бывает достаточно мощным, чтобы загнать воду под кожу, но сила потока быстро уменьшается с расстоянием от ствола из-за турбулентности. Примерно в ярде от сопла ручей превращается в туман. Вы можете быстро определить, какое усилие вы прилагаете к образцу, держа руку как можно дальше от сопла, а затем поднеся ее ближе.Довольно скоро вы почувствуете струю воды на своей руке, и когда вы подойдете достаточно близко, она начнет жалить. На этом стоит остановиться. Выполнив это небольшое упражнение, вы сможете быстро определить, на каком расстоянии от образца вам нужно будет держать пистолет, чтобы приложить желаемое усилие. Обычно вы будете держать «пистолет» в одной руке, а образец — в другой. С кварцем обычно можно взорвать сколько душе угодно. Возможно, вам придется быть осторожным с тонкими образцами, имеющими множество крошечных тонких иголок кварца, потому что, если они не плотно прикреплены к образцу, вы можете иногда их сразу сдувать.Однако вы обнаружите, что кристаллы зачастую оказываются более прочными, чем вы думаете. Накопив немного опыта, вы обнаружите, что можете поцарапать кварц острием ножа и узнать, можно ли его очистить водой под высоким давлением.

Когда вы используете эти маленькие пистолеты для очистки пятен, вы должны использовать что-нибудь для защиты глаз. Я считаю, что очков, которые я обычно ношу, достаточно, чтобы защитить мои глаза от маленьких кусочков камня и грязи, которые пистолет для очистки пятен срывает с грязных образцов.Однако, если вы не носите очки, защитные очки в порядке. Если вы собираетесь чистить несколько экземпляров, вы также обнаружите, что пластиковый дождевик защитит вашу одежду от намокания и попадания на нее множества крошечных кусочков камня и грязи. Я иногда использовал их возле своей кухонной раковины, а позже обнаружил, что куски камня и грязи обильно разбросаны по раковине, столешницам, брызговикам, стенам и окнам и всему остальному, что находится поблизости.

Иногда вода под высоким давлением очищает образец лишь частично, и для его обработки может потребоваться более агрессивный метод механической очистки, а именно использование абразивного инструмента.Однако обычно, если мы считаем, что покрытие действительно трудно соскоблить, мы пропускаем воду под высоким давлением и сразу переходим к одному из наших воздушных абразивных агрегатов.

3. Воздушно-абразивный инструмент и воздушные скребки.
Если струйная обработка кристаллов кварца под высоким давлением не удаляет вредный материал, есть еще одно механическое средство очистки, которое почти наверняка сработает, при условии, конечно, что вы можете соскрести немного проблемного материала лезвием ножа, как описано выше.Это делается с помощью абразивного инструмента. Некоторые называют эти пескоструйные аппараты. Они работают, направляя поток воздуха под высоким давлением, который переносит абразивную среду на образец. Абразивный материал действует как чистящий порошок, удаляя или стирая то, против чего он был направлен. Хитрость заключается в том, что вам нужно использовать менее твердый абразивный материал, чем материал, который вы пытаетесь очистить, чтобы избежать повреждения образца. С кварцем я рекомендую вам использовать крошечные стеклянные шарики, а не кварцевый или гранатовый песок.Стеклянные шарики не будут визуально повреждать поверхность кварца при условии, что давление воздуха не слишком высокое, а стеклянные шарики относительно чистые и не содержат большого количества острых предметов из битого стекла или других твердых примесей. Кварцевый или гранатовый песок обычно удаляет блестящую поверхность кристаллов кварца и делает их тусклыми. Если у вас нет доступа к воздушно-абразивному инструменту, посмотрите, есть ли в одном из ваших местных гаражей или металлообрабатывающих предприятий инструмент, который они используют для очистки свечей зажигания или снятия заусенцев с металлических деталей, и посмотрите, сможете ли вы занять на это немного времени.Но убедитесь, что вы не используете ничего более агрессивного или более твердого, чем стеклянные бусины. Начните с давления воздуха около 60 фунтов и при необходимости увеличьте его. Часто с помощью такого оборудования вы можете очистить небольшие образцы кварца (размером с ладонь) за несколько минут. Воздушно-абразивные инструменты обычно используются с различными видами абразивных порошков для очистки и подготовки окаменелостей. Мы годами использовали большие и маленькие версии этого оборудования с различными видами абразивных сред, и они многократно окупились.

Если у вас нет доступа к подобному оборудованию, вы можете сделать это по старинке и просто чистить руками ручными инструментами. Из использованных зубочисток получаются прекрасные инструменты для такого рода работ, а маленькие ручные электрические шлифовальные машины, такие как инструменты с гибким стержнем, могут быть вам полезны. Но независимо от того, сколько времени вы тратите на чистку кварца вручную, результаты редко будут такими хорошими, как вы можете получить с помощью оборудования для абразивной обработки воздуха, и это займет у вас в десять-сто раз больше времени.

Другой современный инструмент, который часто бывает полезен при «чистке» кварцевых образцов, — это воздушный скрайбер.Это небольшой миниатюрный ручной отбойный молоток, работающий на сжатом воздухе. Они немного похожи на толстые карандаши, и их можно использовать для придания формы вашему образцу или для удаления кусочков вредного материала, который может расти на ваших кристаллах кварца. Если вы потратите достаточно времени на работу с воздушно-абразивным инструментом, вы часто сможете удалить большие толстые массы материала, но часто воздушный скрайбер удалит его за секунды, а не за минуты или часы. Воздушные скребки также обычно используются для удаления следов пил с образцов, которые были обрезаны до нужного размера алмазными дисками.У нас есть некоторые, произведенные Chicago Pneumatic. Существуют разные виды, одни предназначены для удаления небольшого количества матрицы, а другие — для удаления гораздо большего количества.

4. Ультразвуковые очистители
До появления тканевых пистолетов ультразвуковые очистители часто использовались для очистки образцов. Эти чистящие средства бывают разных размеров, от тех, в которых есть небольшая чашка чистящего раствора, обычно воды с небольшим количеством моющего мыла, до гигантов, в которых можно почти принять ванну.Обычно они изготавливаются из нержавеющей стали и приводятся в действие преобразователями из пьезоэлектрических материалов, таких как цирконат-титанат свинца (PZT), титанат бария и т. Д.), Но иногда их изготавливают из магнитострикционных материалов, приклеенных к внешним стенкам резервуаров. Они сильно вибрируют чистящий раствор, что вызывает образование крошечных пузырьков (кавитация), а схлопывание этих микропузырьков создает много энергии и очищающего действия. В более крупных и мощных моделях вода будет нагреваться по мере использования устройства, что также улучшает процесс очистки.Очищаемый материал находится во взвешенном состоянии в резервуаре. Если вы поместите образцы на дно резервуара, это часто снизит эффект очистки, а иногда и резко, потому что это снижает количество кавитации и, следовательно, эффективность очистки устройства. Часто очистка происходит в течение нескольких минут. Однако это устройство, как правило, не очень хорошо очищает глубокие трещины или находящиеся ниже перекрывающиеся кристаллы или образцы с большим количеством грязи или хорошо затвердевшей грязи. Кроме того, большие устройства иногда бывают довольно шумными, а некоторые издают пронзительный пронзительный визг с высокой тональностью.Хорошие устройства и те, которые достаточно велики, чтобы вместить более крупные образцы, могут стоить от нескольких сотен до нескольких тысяч долларов каждый. Мы быстро прекратили использование этих единиц вскоре после того, как получили наши первые тканевые ружья.

Химическая чистка
Возможно, самая распространенная причина, по которой люди хотят чистить кварц, — это удаление коричневых «железных пятен». Эти «пятна от железа» вызваны в основном двумя минералами оксида железа: гематитом и гетитом. Обычно они ржаво-коричневого цвета, но могут проявлять диапазон цветов от черного до красного.Термин лимонит часто используется для их общего названия. Есть ряд других минералов, которые относятся к этой цветовой гамме и могут потребовать химической обработки, отличной от описанной ниже. Большая часть этого вредного материала может быть удалена механическими средствами, часто легко с помощью воды под высоким давлением или с помощью воздушного абразивного инструмента. Но если у коллекционера нет доступа к этим устройствам, он часто надеется на какую-то волшебную жидкость, в которую он может окунуть свой образец, чтобы удалить оскорбительный материал.Я думаю, что успех некоторых коммерческих чистящих средств, таких как Tarnex (раствор для чистки серебра), когда пользователь окунает потускневшее серебро в раствор, и оно волшебным образом становится ярким, является основной причиной этого желания. Для очистки кварца такого волшебного решения не существует.

Если у вас есть «окрашенные железом» кристаллы кварца, их струйная очистка водой под высоким давлением и / или абразивным инструментом удалит много следов железа, но почти наверняка часть пятен останется в трещинах.Чтобы полностью или почти полностью удалить его, вам понадобится химия. Перед тем, как использовать химические вещества, я бы посоветовал вам очистить образец так же хорошо, как вы можете с помощью описанных выше методов. Это позволит вам быстрее химически очистить образцы и использовать для этого меньшее количество химикатов. Существует три основных способа химического удаления «железных пятен» с кварца. Для этого используются: 1. Раствор Валлера (Iron Out), 2. Щавелевая кислота или 3. Соляная кислота. Эти три химических метода удаляют «пятна железа»: (гематит и гетит).Я должен заявить, что на данный момент практически невозможно удалить хорошо развитые кристаллы гематита и гетита с помощью химических веществ, обсуждаемых ниже, поскольку эти химические вещества эффективно удаляют только мелкозернистые эквиваленты этих минералов.

Эти три химических реагента особенно полезны для удаления гематита (оксида железа) и гетита (гидроксида железа), поэтому, если ваши «пятна от железа» вызваны другими минералами, эти три химических метода могут не сработать для вас. Если вы точно знаете, какие минеральные вещества присутствуют в ваших образцах кварца, и имеете некоторые познания в химии, то выбор химических чистящих средств будет гораздо более очевидным.В идеале, человек, желающий очистить некоторые образцы кварца, проанализирует «грязь» и определит, какой именно минерал (ы) составляет «грязь», а затем выберет соответствующий метод химической очистки. На практике обычно легче экспериментировать с легкодоступными химическими веществами, чем тратить силы на выполнение необходимого анализа, который часто бывает непростым или понятным, особенно при работе с мелкозернистыми смесями различных минералов. Итак, поскольку природа того, что вы хотите удалить, может вызывать сомнения, лучший совет, который я могу вам дать, — это попробовать один из этих трех химических реагентов на не очень ценном образце и посмотреть, добьется ли выбранный вами реагент того, что вы хотите. .

5. Раствор Уоллера (Iron Out)
Если вы решите использовать химические вещества для очистки кварца, я определенно рекомендую вам сначала попробовать использовать раствор Уоллера, потому что (1.) химические вещества в этом растворе обычно несложны. получить по крайней мере в Соединенных Штатах и, что более важно (2.), как правило, менее вредны, чем щавелевая или соляная кислота. Раствор Валлера представляет собой забуференный раствор дитионата натрия. Проще, чем покупать химические вещества и смешивать их самостоятельно, вы можете купить в Wal-Mart продукт под названием Super Iron Out.Вы покупаете его в пластиковых бутылках. Это мелкий белый порошок, который нужно смешать с водой в соответствии с указаниями на бутылке. Если рядом с вами нет Wal-Mart, попросите кого-нибудь купить его и отправить вам. Вы также можете погуглить название Iron Out или использовать http://www.summitbrands.com/summit/ и купить его по почте. Мэгги Уилсон, один из наших постоянных сотрудников Миндатера, сообщает нам, что в Соединенном Королевстве Iron Out известен как Rust Out и распространяется компанией Aqua Cure. Телефон: 01704 516916 Веб-сайт: www.aquacure.co.uk. Адрес: Aqua Dosa, Саутпорт, Англия. PR90SE

Иногда «пятна от железа» проникают глубоко в трещины в кварце, и вам, возможно, придется замачивать образец в течение нескольких дней или недель, чтобы растворы химических реагентов растворили и удалили пятна. В редких случаях вы не можете удалить их все. Раствор дитионата натрия не очень стабилен, поскольку вступает в реакцию с кислородом воздуха. Поэтому не стоит ожидать, что он пригодится через несколько дней. Мы перешли на этот метод удаления пятен железа почти исключительно с помощью других химических методов, потому что он быстрый и простой, и нам не нужно его нейтрализовать, а проблемы с удалением минимальны.

Франклин Робертс из Остина, штат Техас, знающий постоянный сотрудник Mindat, предоставляет нам следующую полезную информацию для людей, которые хотят создать свое собственное решение Iron Out. Это может оказаться особенно полезным для тех, кто живет в странах, где невозможно купить коммерчески доступные чистящие средства, такие как Iron Out или Super Iron Out.

Рецепт приготовления раствора Уоллера:
33 г дитионита натрия, также известного как гидросульфит натрия
28 г NaHCO3 (бикарбонат натрия)
59 г цитрата натрия
Добавьте около 800 кубических сантиметров / миллилитров воды, перемешайте до тех пор, пока химические вещества не станут химическими. растворите и добавьте воды, достаточной для получения литра объема.

Если вы предпочитаете сделать галлон раствора, просто умножьте все на четыре. Он работает медленно при комнатной температуре и быстрее при нагревании, но не превышает 60 ° C. Другие соединения, помимо дитионита, служат для создания буферных растворов / комплексов и могут предотвратить осаждение темно-зеленовато-черного покрытия (пирита) на вашем образце (ах).

В паспорте безопасности материалов (MSDS) перечислены активные ингредиенты Super Iron Out как:
Метабисульфит натрия 20-65%
Гидросульфит натрия 20-65%

В данном случае префикс «мета», что по-гречески означает « после «относится к тому факту, что метабисульфит является разновидностью, которая идет после бисульфита натрия (дитионита) в цепи продуктов окисления, идущей от дитионита натрия -> метабисульфита натрия -> дитионата натрия.Эти три соединения также известны как гидросульфит натрия, пиросульфит натрия и бисульфат натрия соответственно. Довольно запутанно, не правда ли? Причина, по которой Super Iron-Out перечисляет широкий диапазон процентных соотношений для двух ингредиентов, заключается не в том, что они не хотят, чтобы вы знали, сколько каждого из них содержится в их продукте; это потому, что они не знают себя. Фактическое соотношение — это движущаяся цель. Помните, чистый дитионит натрия — мощный восстановитель, способный улавливать два атома кислорода из воздуха, воды или любого другого места, где они могут их получить.Как только он захватывает первый кислород, он становится метабисульфитом натрия и может захватить только еще один атом кислорода, прежде чем превратиться в полностью окисленный дитионат натрия, который бесполезен в качестве восстановителя оксида железа. Однако, если у вас есть бассейн, он отлично подходит для снижения pH. Такие продукты, как супер-железо, обычно производятся из химических веществ промышленного класса, которые содержат намного больше примесей, чем реагенты или продукты высокой чистоты. Причина использования этого сырья заключается в том, что химикаты промышленного класса продаются по несколько долларов за тонну, в то время как чистые вещества могут стоить несколько долларов за фунт.Если все, что вам нужно сделать, это удалить пятна железа с дорожки (или минералов), промышленные или технические средства подойдут. Исходное сырье, используемое для получения сверхжелезда, вероятно, изначально представляло собой умеренно чистый дитионит натрия (гидросульфит), но во время производства и хранения он постепенно поглощает кислород из воздуха, а часть его окисляется, превращаясь в метабисульфит натрия (пиросульфит). Поскольку пиросульфит по-прежнему является хорошим восстановителем, его можно оставить в смеси, но получить точную оценку пропорций практически невозможно.Со временем все больше дитионита превратится в метабисульфит, а затем в дитионат. В конце концов, вся партия разложится до раствора дитионата натрия, и дни, когда пятновыводитель закончился. Вот почему так важно держать контейнер герметичным и как можно дальше от воздуха.

6. Щавелевая кислота
Использование щавелевой кислоты для очистки кристаллов кварца.
Что бы вы ни делали, я бы посоветовал вам использовать Iron Out, как описано выше, прежде чем использовать щавелевую кислоту.Щавелевая кислота — это ядовитый белый кристаллический порошок, который растворяется в воде и обладает способностью растворять различные мелкозернистые минералы железа и очищать коричневый кварц. Щавелевая кислота — это токсичное вещество, которое делает листья ревеня ядовитыми для употребления в пищу. В течение многих лет он продавался, а иногда и продается в хозяйственных магазинах для различных целей, возможно, самой распространенной из которых было отбеливание древесины. Когда вы работаете с этим химическим веществом, вы должны носить пластиковые перчатки и следить за тем, чтобы не вдыхать его пыль и не попадать в рот.Не оставляйте растворы из этого материала, потому что они ядовиты. Прежде чем начать, зайдите в Википедию в сети и прочтите, что там говорится о щавелевой кислоте. Обязательно используйте технический или промышленный сорт, потому что он намного дешевле, чем более чистый, и отлично подойдет для очистки ваших кристаллов. Лучше всего использовать это химическое вещество в пластиковых или керамических емкостях, а не в металлических, потому что щавелевая кислота разрушает большинство металлов. Вы можете обойтись без железного контейнера, такого как бочка на 55 галлонов, но кислота постепенно съест его и, как правило, создаст беспорядок.Чтобы дать вам хорошее представление о том, с чем вы сталкиваетесь, от фунта до полутора фунтов щавелевой кислоты в пятигаллонном ведре воды будет хорошим решением для очистки кварца. Для растворения щавелевой кислоты потребуется несколько минут, и вы должны продолжать помешивать, пока она не растворится. Если вы воспользуетесь теплой водой, он растворится быстрее. Щавелевая кислота в течение многих лет использовалась для очистки кварца в Арканзасе и до сих пор остается химическим веществом, предпочитаемым горняками, которые очищают большое количество кварца. Они используют большие стальные резервуары из стали Т1, которую нагревают газовыми горелками почти до кипения.Таким образом, они могут очистить большое количество кристаллов кварца за ночь, хотя иногда для полной очистки образцов требуется второй проход через кислоту. Они покупают свою щавелевую кислоту (в основном китайского производства) в биг-бегах на поддонах. Это оказался наиболее экономичный способ очистки кварца.

Большинство мелких землекопов, которым необходимо удалить пятна железа с образцов, перешли на воду под высоким давлением и Iron Out. Когда вы закончите промывать образцы раствором щавелевой кислоты, не выбрасывайте его в канализацию.Вы можете нейтрализовать оставшуюся в растворе щавелевую кислоту с помощью известняковой крошки, в результате чего образуется белый относительно нерастворимый осадок оксалата кальция, одного из компонентов многих камней в почках. Большинство людей, которые используют это химическое вещество, просто хранят использованный раствор, чтобы использовать его снова, и иногда добавляют больше щавелевой кислоты по мере необходимости. Некоторые дают испариться досуха. Я много раз использовал щавелевую кислоту для удаления «железных пятен» с кристаллов кварца и кристаллов синей разновидности микроклина, называемой амазонитом.При желании можно нагреть его, и это ускорит очистку образцов. Я проделал это в мультиварках и украл первую из своей кухни! Альтернативой применению электрического или огневого тепла является то, что вы можете поместить раствор щавелевой кислоты в черные емкости или накрыть емкости черным пластиком и позволить солнцу нагреть раствор за вас. Если вы используете пластиковый контейнер, вы можете нагреть их примерно до 55 градусов по Цельсию, прежде чем они станут мягкими и начнут деформироваться. Пятьдесят пять градусов по Цельсию — это примерно столько, сколько вы можете выдержать, и при этом оставаться на пластике без излишней боли.Накрыв их черным пластиком на солнце, они не должны деформироваться. Практическое правило химических реакций состоит в том, что с увеличением температуры на каждые 10 градусов по Цельсию скорость реакции удваивается. Когда ваши образцы станут чистыми, вы должны промыть их и дать им впитаться в чистой воде на несколько часов. Вы можете повторить этот процесс полоскания несколько раз. Замачивание на ночь — это хорошо. Иногда, если на ваших кварцевых образцах есть минералы, содержащие кальций или железо, или вода, которую вы используете, содержит много кальция или железа, это приведет к выпадению оксалата кальция или железа из раствора и покрытию ваших образцов большим количеством грязи. после этого вам придется смыть соляной кислотой.

7. Соляная кислота
Соляная кислота, или соляная кислота, или «бассейновая кислота» — это газообразный хлористый водород, HCl, растворенный в воде. Иногда его используют для очистки кварца, но описанные выше методы лучше, безопаснее и менее проблематичны. Но если у вас их нет, вы можете использовать соляную кислоту. Прежде чем пытаться использовать эту кислоту, зайдите в Википедию в сети и прочтите, что там говорится об этой кислоте. http://en.wikipedia.org/wiki/Hydrochloric_acid Вы можете купить эту кислоту в хозяйственных магазинах и в таких местах, как домашние склады.Когда вы работаете с соляной кислотой, вы должны использовать резиновые перчатки и защитные очки и НЕ должны использовать ее в замкнутом пространстве, например, в вашем доме или гараже. Должно быть много вентиляции. У вас также должен быть под рукой садовый шланг, который можно использовать, чтобы залить водой случайно пролитую кислоту. Соляная кислота — сильная кислота, к которой следует относиться с уважением. Концентрированная соляная кислота, которую вы покупаете в хозяйственных магазинах, имеет содержание около 32% и выделяет сильный кислый пар. ЗАПРЕЩАЕТСЯ засовывать нос в горлышко бутылки и пытаться понюхать ее.Вы почувствуете достаточно запаха, просто вылив концентрированную кислоту из бутылки. Задержка дыхания — хорошая идея. Используйте пластиковую или керамическую емкость для очистки кварца. Пластик намного лучше, потому что он не такой хрупкий. Убедитесь, что у вас есть плотно прилегающая крышка для вашего контейнера. Не используйте металлические емкости при работе с этой кислотой. Поместите кристаллы кварца в емкость и залейте их соляной кислотой. Из-за паров, выделяемых этой кислотой, убедитесь, что вы закрыли контейнер.Периодически проверяйте кристаллы кварца, чтобы узнать, нужно ли им дольше оставаться в кислоте. Не рекомендуется нагревать этот раствор из-за образовавшихся паров. ОБЯЗАТЕЛЬНО НЕ ИСПОЛЬЗУЙТЕ ГОЛЫЕ РУКИ И НЕ ИСПОЛЬЗУЙТЕ ЗАЩИТЫ ГЛАЗ. Когда окажется, что кристаллы кварца чистые, удалите их из кислоты и смойте водой. Затем поместите их в другую емкость с чистой водой и дайте им впитаться в течение часа или двух. Замачивание на ночь не повредит им, и это хорошая идея. Вы делаете это, чтобы удалить всю кислоту, которая могла попасть в трещины образца.Если на образце много трещин или он состоит из пористого материала, возможно, вам придется оставить образец в промывочной воде на несколько дней и несколько раз замочить его в чистой воде, чтобы вся кислота была удалена с образца. Если вы не удалите кислоту из образца полностью, он может пожелтеть позже, и вам придется повторить кислотную обработку и нейтрализацию. Когда вы закончите, вы можете сохранить раствор для будущего использования или нейтрализовать его. Вы не захотите хранить соляную кислоту или растворы соляной кислоты долгое время в своем гараже или где-либо рядом с металлом.Пластиковые бутылки с соляной кислотой имеют свойство со временем трескаться и протекать. Растворы имеют тенденцию выделять хлористый водород, который трудно удержать, и он разъест ржавчину до каждого кусочка железа в любом месте рядом с веществом. Я бы порекомендовал не хранить его больше недели или двух, если вы можете с этим справиться. Для нейтрализации кислоты можно использовать известняковую или мраморную крошку. Когда вы добавляете в кислоту известняковую или мраморную крошку (кальцит), она будет пузыриться и пениться при выделении углекислого газа. Когда вы добавляете больше известняка и больше не образуется пузырьков, раствор нейтрализуется, и его можно утилизировать.Разбавленная соляная кислота — это то, что ваш желудок использует для переваривания пищи. Если вы пролили кислоту на пол или одежду, просто промойте это место или одежду большим количеством воды и, чтобы убедиться, что кислота ушла, промокните это место бикарбонатом соды. Если шипение не происходит или шипение прекращается, значит, вы успешно нейтрализовали кислоту. Когда я работаю с этой кислотой и случайно проливаю немного на руку или кожу, я промываю это место большим количеством воды, а затем пробую на вкус.Если на коже останется кислота, она будет кислым на вкус. Если это так, вам нужно будет еще немного вымыть область и, возможно, промокнуть ее бикарбонатом соды, чтобы обеспечить нейтрализацию. Дегустировать другие химические реагенты категорически не рекомендуется, потому что они могут быть отравляющими или даже смертельными. Старые химические тексты рассказывали вам, какой вкус у различных химикатов, потому что это был очень быстрый и простой способ дать вам представление о том, какое химическое вещество у вас есть. Некоторые химики отравились.

8.Фосфорная кислота
Вы также можете использовать фосфорную кислоту для очистки пятен от железа, но обычно эта кислота стоит дороже и занимает немного больше времени. Я не рекомендую вам использовать этот метод. Иногда при использовании этого метода на образцах осаждается фосфатные минералы, которые часто очень трудно удалить.

Очистка образцов аметиста из Тандер-Бей, Канада.
Химические вещества можно использовать для удаления тонких пленок минералов оксида железа, которые обычно присутствуют на образцах аметистов из Тандер-Бей.Однако в этом месте минералы оксида железа могут быть довольно толстыми, и их удаление химическими веществами вызывает изменения, которых нет в кварце из других мест. Для очистки этих образцов используются химические вещества, но это не так просто, как очистка образцов из других мест, а также трудоемкая задача. Смеси реагентов могут включать как соляную кислоту, так и щавелевую кислоту и нагревание. Формулы используемых реагентов являются собственными и получены только после значительных экспериментов. Поскольку они дают экономическое преимущество майнерам, которые их используют, они обычно не хотят отказываться от методов очистки, которые приносили им деньги на протяжении многих лет.Если кто-то захочет поделиться подробной информацией об очистке таких образцов кварца, мы будем рады рассказать о них здесь.

9. Очистка кварца с помощью коммерческих реагентов, содержащих небольшое количество бифторида.
Вот метод очистки кварца, который был предложен и задокументирован членом Mindat Никифороу, который кажется настолько практичным и хорошим, что мы представляем его здесь для всех членов Mindat. Если вы знакомы с тем, как выглядят образцы, особенно кристаллы кварца, когда они только что собраны из пегматитового кармана, вам следует знать об этом методе их очистки, особенно если в вашем распоряжении нет абразивных инструментов. .Даже если вы это сделаете, вы можете попробовать этот метод. Посмотрите на это изображение кварца и сподумена до и после. образец кунцита внизу. Хотя предыдущий снимок не очень резкий и сделан не из того же положения, что и после снимка, очевидно, что очистка образца была очень эффективной.

«Очистка вышеуказанного образца с помощью Whink заняла у меня около трех недель с использованием трех полных циклов, чтобы довести изделие до того состояния, в котором я был удовлетворен тем, что сделал достаточно.Обратите внимание, что даже несмотря на то, что большая часть белого вещества отделилась от кристалла кварца, после очистки он все еще оставался «под угрозой блеска». Тем не менее, я вполне доволен результатами (хотя я тоже кое-что испортил).

Хотя вы, вероятно, можете приготовить аналогичный реагент, используя гидрофлюорит аммония (бифторид аммония, белый ядовитый порошок), мы рекомендуем вам использовать коммерчески подготовленный реагент под названием Whink. Это тот, который легко получить здесь, в Соединенных Штатах.Наверное, есть и другие. Возможно, кто-то предложит формулу и процедуру приготовления реагента для аналогичного реагента, но до тех пор этот вариант подойдет. Тем из вас, кто живет в зарубежных странах и у кого нет доступа к этой конкретной марке бифторидного реагента, я бы посоветовал вам попросить или заплатить промышленному химику, чтобы он разработал формулу для той, которую вы можете использовать и, возможно, даже продать другим, которые могут захотеть для очистки своих кварцевых образцов. А пока давайте послушаем, что говорит Ник Никифороу:

У меня были хорошие результаты по удалению силикатного «белого вещества» из кварца и других минералов с помощью легко получаемого продукта под названием Whink Rust Stain Remover.Это жидкость, выпускаемая в коричневых пластиковых бутылках размером 6, 10, 16 и 32 унции, и ее можно купить во многих хозяйственных магазинах или в Интернете. Прежде чем я продолжу, вам необходимо знать, что этот продукт содержит от 2% до 3% фтористоводородной кислоты, одной из, если не САМЫХ, известных коррозионных кислот, и необходимо принять крайние меры предосторожности, в том числе работать с ним ТОЛЬКО НА ОТКРЫТОМ ВОЗДУХЕ, БЕЗОПАСНОСТЬ ОЧКИ и химически стойкие ПЕРЧАТКИ. Вы НЕ должны позволять жидкости касаться вашей кожи, и вы не должны вдыхать пары.Я не могу этого особо подчеркнуть.

Сказав это, я использую его, помещая очищаемый образец в СЛЕДУЮЩИЙ закрытый пластиковый контейнер (не стеклянный, так как он будет разъедать стекло!), Наливая достаточное количество жидкости, чтобы покрыть образец, и оставляя его снаружи. солнце в течение нескольких дней и дольше. Если мне нужно обработать большой кусок, я разбавлю его достаточным количеством воды, чтобы покрыть образец, хотя это продлит время, необходимое для его работы. Я буду проверять его каждые пару дней, аккуратно соскребая белое вещество, чтобы увидеть, не стало ли оно размягчаться.На этом этапе я сниму его с Whink и замочу в воде на несколько дней, ежедневно меняя воду, чтобы удалить остатки HF, особенно если кусок пористый. Затем я аккуратно соскребаю как можно больше материала стоматологическим инструментом или ножом, и, если кусок физически может противостоять ему, ударяю по нему из водяного пистолета. Мне часто приходится ПОВТОРИТЬ этот процесс два или три раза (обработка винком, замачивание водой, механическое удаление), чтобы удалить остатки белого вещества. Работы много, так что стоит делать это только с лучшими деталями.

ПЕЧАТИ:
1. Это не самый дешевый способ использования HF — вы можете получить больше прибыли, используя HF, полученный от химических предприятий, который является гораздо более концентрированным и может быть разбавлен в соответствии с вашими потребностями. Это НЕ вариант для меня или для большинства коллекционеров — я видел фотографии серьезного повреждения тканей, вызванного даже коротким воздействием этой кислоты, и не хочу, чтобы она находилась поблизости от меня.

2. Whink может УГЛУШИТЬ блеск кварца и других силикатов, если использовать его в течение длительного времени.Он также разрушит некоторые другие минералы (не используйте его на апатите!) И постепенно начнет травить полевой шпат и слюду. Сначала проведите небольшое исследование или тест на меньших предметах.

3. Частое снятие белого вещества НЕ улучшает внешний вид образца. По моему опыту, большинство кристаллических граней, находящихся под белым слоем, в любом случае имеют тенденцию быть тусклыми; это одна из причин того, что белое вещество так прочно прикреплено к кристаллу, поскольку у него множество микроповерхностей, за которые можно «ухватиться».

4. У меня было несколько случаев, когда образец либо терял несколько кристаллов, либо распадался, потому что «белое вещество» на самом деле скрепляло его. Вам необходимо внимательно изучить образец, чтобы определить, может ли это случиться.

Если вы планируете использовать Whink, ПОЖАЛУЙСТА, ПРИНИМАЙТЕ ПРАВИЛЬНЫЕ МЕРЫ БЕЗОПАСНОСТИ!
[Nik Nikiforou 2009]

В Бразилии дилеры, которые часто покупают и продают кристаллы кварца, используют коммерческую чистящую жидкость под названием Chispas, очищающий эффект которой обеспечивается бифторидом аммония и другими ингредиентами.Он используется для удаления пятен железа с кристаллов кварца, и они считают, что это также делает кварц ярче, и это может быть результатом слабого раствора HF, удаляющего очень микрокристаллический кварц с поверхности кристаллов кварца. Но я не уверен в этом. Я видел, как его использовали в Риу-Гранди-ду-Сул среди производителей образцов аметиста, чтобы сделать кристаллы аметиста яркими и чистыми, и мне говорили, что если аметист оставить слишком долго в Чиспасе, особенно свежем Чиспасе, он потускнеет кристаллы аметиста.Я видел образцы аметиста, где кристаллы аметиста все еще яркие и блестящие, но лежащий под ним агат стал белым снаружи, и мне сказали, что это результат даже короткой очистки в Чиспасе. Фтор в растворе очень быстро разрушает халцедон / агат по сравнению с кристаллизованным аметистом. Существует ряд коммерческих чистящих средств, в составе которых используется бифторид; Среди них те, которые регулярно используются в коммерческих автомойках. Растворы бифторида аммония следует нейтрализовать, бросив в раствор мраморную или известняковую крошку.Это вызовет образование пузырьков и белый осадок фторида кальция (флюорита). Чтобы убедиться, что реакция нейтрализации завершилась, продолжайте добавлять мраморную или известняковую крошку, пока не перестанет пузыриться. Это может занять некоторое время.

10. Возможная очистка и / или удаление кварца плавиковой кислотой (HF)
Я не собираюсь рассказывать вам, как использовать это ужасное химическое вещество, но я расскажу вам кое-что о нем, что, я надеюсь, может убедить вас не делать этого. попробуй использовать это. Я также расскажу вам о некоторых вещах, которые он может и не может делать.Опасность, связанная с использованием этой кислоты, настолько велика, что даже эксперты Миндата рекомендуют не использовать ее. Кроме того, здесь, на Миндате, существует неформальная политика, согласно которой эксперты не будут указывать людям на доске объявлений, как использовать эту кислоту. Есть несколько применений, которые нельзя заменить менее опасными химическими веществами, и если вам действительно нужно использовать HF для этих целей, вам нужно найти кого-то, кто знает, как обращаться с HF. Хорошо подойдет старый химик или учитель химии; попросите его / ее обучить вас безопасному использованию этого реагента.Пытаться научить вас этой кислоте письменно — это не то, на что я охотно поступлю. Это навлекло бы на себя всевозможные неприятности, особенно в этом спорном обществе.

Плавиковая кислота — это в основном ядовитый газ фтористый водород, растворенный в воде. В концентрированном виде это прозрачная жидкость, которую нельзя хранить в стекле, потому что она растворяет емкость. Мне рассказывали, что в старых химических лабораториях его хранили во флаконах из парафина. Это было до появления современных пластиковых контейнеров.В небольших количествах он теперь продается в бутылках из толстого пластика, например, из полипропилена. Когда вы откроете баллон, газ начнет выходить, и в влажный день вы его увидите. Он поднимется немного вверх, как пар, и позвольте мне заверить вас, что вы действительно не хотите дышать этим веществом. Так что, если день не влажный, и у вас нет вытяжного шкафа, чтобы избавиться от паров из HF, вы действительно не можете сказать, собираетесь ли вы дышать чем-либо, пока не станет слишком поздно. Немного, вероятно, не убьет вас, но если вы почувствуете его запах, вы убежите в укрытие.Даже самые бездушные из нас, которые много раз использовали этот реагент, относятся к этому зверю с уважением. Я собираюсь вести вас так далеко по этому пути. Позвольте мне также сказать, что многие люди, даже те, кто обучался химии, получили краткосрочные и долгосрочные травмы из-за этого химического вещества. Он может творить с вашим телом неприятные вещи. Если, несмотря на то, что я сказал здесь, вы упорствуете в попытках использовать HF, что бы вы ни делали, зайдите в Википедию в сети и прочтите о кислоте и сопутствующих опасностях. http: //en.wikipedia.org / wiki / Hydrofluoric_acid. Ужасные эффекты, вызываемые контактом с этим химическим веществом, известны уже более 100 лет. Шесть капель убьют собаку. http://lateralscience.blogspot.co.uk/2013/06/little-dog-hydrofluoric-acid.html

Все сказанное выше, если вы хотите удалить кварц или другие кремнеземсодержащие минералы из золота, серебра или других материалов. минералы, на которые HF не влияет, то другого выхода может и не быть. В некоторых случаях кристаллы кварца могут быть покрыты тонкой друзами вторичных микрокристаллов кварца, которые могут быть удалены с помощью плавиковой кислоты.Чтобы растворить или частично растворить кварц из золота, серебра и т. Д., Вам нужно использовать довольно концентрированный HF, и тогда процесс будет медленным, день или два или больше, в зависимости от того, сколько вы хотите удалить. Массивный кварц (бычий кварц), халцедон и опал поражаются гораздо быстрее, чем поверхность обычных кристаллов кварца. Растворение хорошо сформированного кристалла кварца с помощью HF при комнатной температуре может занять несколько дней, и он не полирует кварц и не делает его блестящим, как обычное стекло, а скорее неровным и тусклым.Я видел довольно тусклый кварцевый крученый кварц из Вашингтон-Кэмп, штат Аризона, обработанный в довольно концентрированной HF в течение нескольких часов с удивительным результатом: очень мелкий рыхлый кварц, который придавал двойникам матовый оттенок, был в основном удален, а поверхности оставались блестящими. . Мы забыли положить их в кислоту и вернулись примерно через четыре часа, чтобы найти этот удивительный результат. Не думаю, что у нас хватило бы смелости бросить их так долго, если бы мы раньше вспомнили, что они были на ВЧ.Некоторые кристаллы имели небольшие трещинки, и хотя HF оставлял поверхность кристаллов кварца блестящей, она действительно немного атаковала ее по краям трещин и оставляла небольшие белые следы вдоль трещин. Мой совет: если кварц, который вы пытаетесь очистить, действительно исключительный, не пытайтесь очищать его с помощью HF. Это просто не стоит затрат и риска для вашего здоровья. Последняя цитата, которую я получил от химической компании за галлон HF, составляла более 100 долларов. Если вам действительно нужно использовать HF, вы можете прочитать различные статьи и комментарии в Интернете.Один из них приведен здесь: http://www.minsocam.org/ammin/AM46/AM46_1498.pdf

Гидроксид натрия
Альфредо Петров добился успеха в очистке покрытий скородита от кварцевых двойников по закону Японии из Ками, Боливия. . Это были группы близнецов японского права. Он поместил кварц в концентрированный раствор гидроксида натрия. Это превратило скородит в гетит, который затем можно было удалить с помощью раствора Валлера, щавелевой или соляной кислоты. Вероятно, вы не захотите оставлять кварц в растворе крепкого щелока (гидроксида натрия) надолго, потому что кварц медленно растворяется в этом реагенте при комнатной температуре.Сильные растворы гидроксида натрия очень едкие и растворяют кожу, превращая жир на коже в мыло. При работе с этим материалом надевайте пластиковые перчатки. Вы можете нейтрализовать щелочные растворы уксусом или соляной кислотой.

Перекись водорода
Иногда перекись водорода можно использовать для удаления определенных черных минералов марганца, таких как тодорокит, из кристаллов кварца. Некоторые люди сообщают об успешном удалении органических материалов, таких как лишайник, глинистые минералы или мелкозернистые минералы, из кварца и других минералов с помощью растворов перекиси водорода.Когда раствор вступает в реакцию с минералами марганца, образуются пузырьки газообразного кислорода. Перекись водорода с чистотой реагента может бурно реагировать с такими вещами, как асфальт. Перед использованием концентрированной перекиси водорода убедитесь, что вы действительно знаете, что делаете. Мы добились достаточного первоначального успеха в удалении глины из китайского азурита с помощью стандартного слабого раствора перекиси водорода (для отбеливания волос), и мы собираемся больше экспериментировать с этим реагентом. Неясно, что именно перекись водорода делает при разрыхлении глины и других мелкозернистых материалов, но она оказывает благотворное влияние.У меня не было большого опыта использования этого реагента на кварце, поэтому любая помощь более знающих, чем я, будет приветствоваться. Результаты использования этого реагента на оксидах марганца могут быть очень драматичными. Вы бросаете образец в раствор, и когда пузырьки рассеиваются через минуту или две, образец становится волшебно чистым, без всякого черного оксида марганца. Однажды мне удалось всего за несколько минут очистить многие образцы пренитовых слепков после ломонтита, который, казалось, был безнадежно покрыт черным тодорокитом.Это было похоже на волшебство.

При работе с химическими веществами, подобными указанным выше, не следует смешивать разные растворы вместе. Иногда это будет вызывать нежелательные осадки или реагировать нежелательными или опасными способами.

Мы хотели бы попросить Миндатеров, которые имеют опыт очистки кварца различных типов, поделиться своим опытом с различными химическими веществами, отправив нам электронное письмо, чтобы мы могли сделать эту статью более полной и полезной. Если вы чего-то не понимаете или хотите пояснить подробнее, опубликуйте заметку на доске ниже, и вы почти наверняка сможете получить дополнительную помощь.Меня время от времени называли экспертом, но как только я начинаю чувствовать себя немного самодовольным, старый друг напоминает мне, что бывший — это бывало, а рывок — это капля под давлением.

Rock Currier
Проверено и проверено Джорджем Холлоуэем

Баритовая белая акварельная краска — натуральные пигменты

Детали

Рублёв Цвета Белый барит — это непрозрачный белый цвет, изготовленный с использованием природного минерального барита (британский английский baryte ; химическое название: сульфат бария).Сульфат бария использовался британскими акварелистами в восемнадцатом и девятнадцатом веках в качестве краски для тела. Синтетический минерал часто назывался «Постоянно-белый» или «Постоянный белый», занимая второе место среди цветных пигментов белой воды после китайского белого (оксида цинка). Он состоит из осажденного сульфата бария или блан фикс. В двадцатом веке это название также использовалось для цинковой белки или смесей сульфата бария с цинковой белкой.

Chinese White или Zinc White имеет очень неприятное свойство растрескивания при использовании в толстых покрытиях; То есть после высыхания покрытие распадается на мелкие блоки.В этом отношении барит белый намного лучше. Единственным недостатком Barite White является то, что он кажется более темным во влажном состоянии и с меньшей укрывистостью после высыхания, когда вода испарилась. Поскольку художнику иногда трудно использовать цвета, которые сохнут визуально, отличные от тех, которые были получены при первом нанесении, лучше всего использовать Barite White с достаточным количеством акварельной среды, чтобы создать условия, которые приведут к окончательным различиям. как можно меньше.

В девятнадцатом веке в состав Barite White часто входили как трагакантовая камедь, так и гуммиарабик из-за ранее обсуждавшегося качества и из-за очень высокого удельного веса пигмента.Rublev Colours Barite White производится на основе тех же формул девятнадцатого века, с использованием обеих камедей в связующей среде.

Рублёв Цвета Барит Белый Акварель

Примечание : Образцы цветов показаны массовым тоном вверху, смешаны с равным количеством воды и заштрихованы к низу. Все изображения образцов цвета на этом веб-сайте являются приблизительными к фактическому цвету акварельной краски. Мы приложили все усилия, чтобы цвета на этих изображениях на откалиброванных цветных мониторах соответствовали фактическому цвету.Однако из-за большой разницы в цвете мониторов результаты могут отличаться.

Состав и стойкость

Хелатирующий агент Сокращение
N — (3-гидроксипропил) имино- N , N -диауксусная кислота
2-HPIDA
N -глицерилимино- N , N -диауксусная кислота GLIDrois

-диауксусная кислота GLIDA DHPIDA
Метилимино- N , N -диуксусная кислота MIDA
2-метоксиэтилиминоцевая кислота- -N00 кислота MEIDA
Амидоиминодиуксусная кислота (= амидонитрилтриуксусная кислота натрия) SAND
Ацетамидоиминодиуксусная кислота AIDA
3-Метоксипропилимино- N , N -диуксусная кислота MEPIDA
Трис (гидроксиметил) метилимино- N , N -диауксусная кислота TRIDA 9016

.

Белый барит
Цвет: Белый
Переплет: Гуммиарабик
Информация о пигменте
Пигмент: Барит (барит)
Классификация пигментов: Природное неорганическое
Индекс цвета: Белый пигмент 22
Химическое название: Сульфат бария
Химическая формула: BaS 4
№ CAS 13462-86-7 (7727-43-7)
Недвижимость
Код: 809
Серия: 3
Непрозрачность: непрозрачный
Прочность тонирования: Умеренная
Окрашивание: Не применимо
Гранулирование: Не применимо
Флокуляция: Не применимо
ASTM Светостойкость: I
Стойкость: А
Информация по безопасности: На основании токсикологического обзора нет острых или известных хронических опасностей для здоровья при предполагаемом использовании этого продукта.Всегда защищайте себя от потенциально неизвестных хронических опасностей, связанных с этим и другими химическими продуктами, избегая проглатывания, чрезмерного контакта с кожей и вдыхания распыляемого тумана, шлифовальной пыли и концентрированных паров. Свяжитесь с нами для получения дополнительной информации или обратитесь к паспорту безопасности материалов для получения дополнительной информации.

Для подробного объяснения терминов в таблице выше, пожалуйста, посетите Состав и стойкость .

Банкноты

Примечание : Некоторое разделение пигмента и среды может происходить в акварельных красках «Рублев Колорс» и является естественным процессом, когда в краску не добавляются стабилизаторы, чтобы предотвратить это.

Идентификация опасностей

Классификация в соответствии с Регламентом ЕС 1272/2008: Этот продукт не классифицируется как опасный в соответствии с Директивой CLP / GHS.

Классификация в соответствии с Регламентом ЕС № 67/548 или № 1999/45: Материал не подлежит классификации в соответствии со списками ЕС. Этот продукт не требует классификации и маркировки как опасный в соответствии с CLP / GHS.

Обозначение безопасности

P280 Пользоваться защитными перчатками / одеждой / средствами защиты глаз / лица.

P281 При необходимости используйте средства индивидуальной защиты.

P501 Утилизируйте содержимое / контейнер в соответствии с региональными, национальными и международными правилами.

Произошла ошибка при настройке пользовательского файла cookie

Этот сайт использует файлы cookie для повышения производительности. Если ваш браузер не принимает файлы cookie, вы не можете просматривать этот сайт.


Настройка вашего браузера для приема файлов cookie

Существует множество причин, по которым cookie не может быть установлен правильно.Ниже приведены наиболее частые причины:

  • В вашем браузере отключены файлы cookie. Вам необходимо сбросить настройки своего браузера, чтобы он принимал файлы cookie, или чтобы спросить вас, хотите ли вы принимать файлы cookie.
  • Ваш браузер спрашивает вас, хотите ли вы принимать файлы cookie, и вы отказались.
    Чтобы принять файлы cookie с этого сайта, нажмите кнопку «Назад» и примите файлы cookie.
  • Ваш браузер не поддерживает файлы cookie. Если вы подозреваете это, попробуйте другой браузер.
  • Дата на вашем компьютере в прошлом.Если часы вашего компьютера показывают дату до 1 января 1970 г.,
    браузер автоматически забудет файл cookie. Чтобы исправить это, установите правильное время и дату на своем компьютере.
  • Вы установили приложение, которое отслеживает или блокирует установку файлов cookie.
    Вы должны отключить приложение при входе в систему или проконсультироваться с системным администратором.

Почему этому сайту требуются файлы cookie?

Этот сайт использует файлы cookie для повышения производительности, запоминая, что вы вошли в систему, когда переходите со страницы на страницу.Чтобы предоставить доступ без файлов cookie
потребует, чтобы сайт создавал новый сеанс для каждой посещаемой страницы, что замедляет работу системы до неприемлемого уровня.


Что сохраняется в файле cookie?

Этот сайт не хранит ничего, кроме автоматически сгенерированного идентификатора сеанса в cookie; никакая другая информация не фиксируется.

Как правило, в файлах cookie может храниться только информация, которую вы предоставляете, или выбор, который вы делаете при посещении веб-сайта.Например, сайт
не может определить ваше имя электронной почты, пока вы не введете его. Разрешение веб-сайту создавать файлы cookie не дает этому или любому другому сайту доступа к
остальной части вашего компьютера, и только сайт, который создал файл cookie, может его прочитать.

.

Добавить комментарий

Ваш адрес email не будет опубликован. Обязательные поля помечены *